Sie sind auf Seite 1von 289

ACCA P2

Corporate Reporting (INT)


Basic Package Note
2015
© Lesco Group Limited, April 2016
All rights reserved. No part of this publication may be reproduced, stored in
a retrieval system, or transmitted, in any form or by any means, electronic,
mechanical, photocopying, recording or otherwise, without the prior written
permission of Lesco Group Limited.
Course Note Structure

The course will be divided into 4 parts:

First, you need to go through the tuition pre-recorded video lectures on the study
platform together with our tuition note.

Second, you need to watch the super summary course lectures and the notes will be
detailed here as well.

Third, after you have done the previous two steps, you also need to go through case
study course lectures as well.

Finally, join our final super revision sessions to axes your study.
Content

CHAPTER1: INTRODUCTION TO P2 ............................................................................................................... 6

CHAPTER2 ACCOUNTING STANDARDS ......................................................................................................... 7


IAS 1PRESENTATION OF FINANCIAL STATEMENTS ......................................................................................... 8
IAS 2 INVENTORY ...................................................................................................................................... 14
IAS 8 ACCOUNTING POLICIES, CHANGES IN ACCOUNTING ESTIMATES AND ERRORS ................................... 17
IAS10 EVENTS AFTER THE REPORTING PERIOD........................................................................................... 22
IAS 12 INCOME TAXES ............................................................................................................................... 24
IAS16 PROPERTY, PLANT &EQUIPMENT ..................................................................................................... 39
IAS 17 LEASES ........................................................................................................................................... 46
IFRS 15 — REVENUE FROM CONTRACTS WITH CUSTOMERS ....................................................................... 52
IAS 19 EMPLOYEE BENEFITS AND IFRS 2 SHARE BASED PAYMENT ............................................................... 58
IAS 20 GOVERNMENT GRANTS .................................................................................................................. 70
IAS 23 BORROWING COSTS ....................................................................................................................... 76
IAS 24 RELATED PARTY DISCLOSURES ......................................................................................................... 81
IAS 34 INTERIM FINANCIAL REPORTING..................................................................................................... 85
IAS 36 IMPAIRMENT OF ASSETS ................................................................................................................. 87
IAS 37 PROVISIONS, CONTINGENT LIABILITIES AND CONTINGENT ASSETS .................................................. 96
IAS 38 INTANGIBLE ASSETS ...................................................................................................................... 103
IAS 40 INVESTMENT PROPERTY ............................................................................................................... 106
IFRS 1 FIST TIME ADOPTION OF INTERNATIONAL FINANCIAL REPORTING STANDARDS ............................. 112
IFRS 2 SHARE BASED PAYMENT ................................................................................................................ 114
IFRS 5 NON-CURRENT ASSETS HELD FOR SALE AND DISCONTINUED OPERATIONS .................................... 120
IFRS 8 OPERATING SEGMENTS ................................................................................................................. 128
IFRS 10 CONSOLIDATED FINANCIAL STATEMENTS ..................................................................................... 170
IFRS 11 JOINT ARRANGEMENTS ............................................................................................................... 172
IFRS 12 DISCLOSURES OF INTERESTS IN OTHER ENTITIES .......................................................................... 173
IFRS 13 FAIR VALUE MEASUREMENT ........................................................................................................ 176
IAS 41 AGRICULTURE ............................................................................................................................... 178
IFRS FOR SME’S ....................................................................................................................................... 186
MANAGEMENT COMMENTARY ............................................................................................................... 187
CONCEPTUAL AND REGULATORY FRAMEWORK ....................................................................................... 188
INTEGRATED REPORTING ........................................................................................................................ 194

CHAPTER 3 CONSOLIDATION ................................................................................................................... 195


Chapter1: Introduction to P2
P2 is all about application of accounting standards to consolidation FS + solve client’s
specific problems.

You need to know the behind idea and theory of accounting standards outlined in the study
note.

You will also need to practice lots of past exam questions to reinforce your understanding
about each accounting standards.

Make sure you do those and certainly you can pass this exam!

Exam Structure:

Q1: Consolidation (35marks relating to numbers + 15 marks relating to ethics)

Q2/ Q3: Accounting Standard Question with 25 marks each

Q4: Current Issues


Chapter2 Accounting Standards
IAS 1Presentation of Financial Statements

The objective of FS is to provide information about the financial position, performance


and cash flows of an enterprise that is useful in making economic decisions.

The FS will show how effectively management has looked after the resources of the
entity to help users assess the stewardship of management.

Contents of FS:

1: Statement of financial position


2: Statement of profit or loss and other comprehensive income
3: Statement of changes in equity
4: Statement of cash flow
5: Accounting policies note and other explanatory notes.

Principle Based:

IAS 1 is just principle based accounting standard and management can depart from it.
For example:

 Management can use other titles such as Balance Sheet instead of statement of
financial position.
 Management can also provide a financial review describing the main features of
entities’ financial position and performance.
 Management can depart from the IFRS if they feel that compliance with an IFRS
would be misleading and to depart from the IFRS requirements would make the
FS show a fair presentation(This is quite rare). If so, management needs to make
the following disclosures:

Disclosure of departure:

 What has been departed and why?


 The departure would result in a fair presentation of financial performance and
position as well as cash flow.
 The financial impact of the departure on net profit or loss, assets, liabilities, equity
and cash flows.
Example of departure:

A reporting entity operates in a jurisdiction where deliberate departure from the


requirements of IFRS is not permitted. Management has considered the requirements
in IAS 2 Inventories which prohibits the use of the last-in first-out (LIFO) method of
cost flow assumption when valuing inventories. Management feels that due to the high
tech industry in which they operate, the LIFO method of cost flow assumption is
applicable. However, they are prohibited from departing from the requirements of IAS
2.

Some requirements by IAS1:

1: Underlying assumptions: (Going concern + Accrual basis)

 Going concern VS break up basis


IAS 1 states that FS should be prepared on the going concern basis if the company can
trade it businesses at least 12 months after the FS year end.

If not, company should prepare its FS under break up basis.

Going concern basis means that the FS would include non-current assets and
non-current liabilities.

Break up basis means that the FS would not include non-current assets and
non-current liabilities.

 Accrual basis VS Cash basis


IAS 1 requires the entity to prepare their FS using accruals basis for statement of
financial position, statement of profit or loss and other comprehensive income.

But for statement of cash flow, it should be prepared using cash basis.

Accrual basis means that the transactions should be recorded in the accounting period
in which they relate regardless of whether or not cash has been received or paid.
For example, we recognize sales revenue after we signed the contract with customer
even though we haven’t received cash from them.

Accrual concept also means “Matching Principle”. This means expenses should be
recognized in the P/L to match against the income that entity earns.

2:Materiality and aggregation

Materiality means that if the item is omitted or misstated then it would affect users’
economic decisions.

For material items, they should be presented separately in the FS.


Immaterial amounts should be aggregated with the amounts of a similar nature and
need not be disclosed separately.

Materiality would be determined either by nature (Fraud) or amount: 1% of total


assets; 5%of total profit; 0.5% of total revenue.
3. Offsetting

Assets, liabilities, income and expenses should note be offset against each other
unless this is allowed by IFRS.

For example, under IFRS 5 non current assets held for sale and discontinued
operations, it allows the following to be offsetting against each other:

Net cash flow detailing operating, investing and financing activities.

Single line in the statement of profit or loss and other comprehensive income showing
post tax profit or loss on discontinued operation.

4. Comparative information

Comparative information should be disclosed regarding previous period for all


amounts reported in the FS.

5. Timeliness

FS should be presented at least annually and should be issued on a timely basis


(within 6 months of the end of reporting period and 9 months for private entities.)

6. Consistency

Presentation and classification of items should be consistent from one period to the
next.
Example of Statement of Financial Position:

Example of statement of financial position as at 31 DEC 2015:

$000 $000
Assets
Non-current assets
Property plant & equipment 200,000
Intangible assets 187,999
387,999
Current assets
Inventory 88,432
Trade receivables 97,455
Bank 13,400
199,287
Total assets 587,286

Equity and liabilities


Equity*
Share capital 50,000
Share premium 50,000
Revaluation reserve 38,000
Retained earnings 220,497
358,497

Non-current liabilities
8% loan note 75,000
Redeemable preference shares 25,000
100,000

Current liabilities
Trade payables 77,789
Taxation 51,000
128,789

Total equity and liabilities 587,286


Example of statement of profit or loss for the year ended 31 DEC 2015:

$’000
Revenue 385,000
Cost of sales (188,000)
Gross profit 197,000
Other income 2,000
Distribution costs (38,500)
Administration expenses (37,700)
Profit before interest and tax 122,800
Finance costs (8,000)
Profit before tax 114,800
Income tax expense (53,000)
Profit for the year(profit after tax) 61,800

Other comprehensive income:


Gains on property revaluations 36,000
Remeasurement components 2,000
Total comprehensive income for the year 99,800

Other comprehensive income is income and expense that are not recognized in profit
or loss (recognize in reserve rather than actual profit, because they are not realized.)

For material items, they need to be presented either:


 1: on the face of P/L before operating profit
 2: in the notes

Examples:
 Inventory write offs
 Impairment losses
 Restructuring costs
 Disposals of property, plant and equipment
Proposed Amendments

Effective for annual periods beginning on or after 1 January 2016, early adoption
permitted

 materiality;
Clarifying that materiality applies to the whole financial statements and that
information which is not material need not be presented in the primary financial
statements or disclosed in the notes

 order of the notes;


Remove the perception of a 'normal order of presentation' of financial statements,
making it easier for entities to provide more contextual and holistic information

 subtotals;
Adding additional explanations with examples of how IAS 1 requirements are designed
to shape financial statements instead of specifying precise terms that must be used,
including whether subtotals of IFRS numbers such as earnings before interest and tax
(EBIT) and earnings before interest, tax, depreciation and amortisation (EBITDA)
should be acknowledged in IAS 1

 accounting policies;
Reducing restrictions on how accounting policies should be presented, allowing
important accounting policies to be given greater prominence in financial reports
IAS 2 Inventory

1, Initial measurement

Inventory= quality X value


(Number of inventory purchased X historical cost)*

*Historical cost:
1, Cost of purchase: purchase price, import duties but excluding discounts
2, Cost of conversion relating to production (direct/variable overheads),
E.g., labour costs in factory; (but labor costs relating to marketing department
is not) machinery depreciation
3, Other costs happened necessary to bring the inventory to its intended location
and condition.
e.g., Carriage inwards can be cost. But carriage outwards is expense

2, Subsequent Measurement (valuing closing inventory)

Inventory= quality X value


(Closing inventory X (using FIFO, WAC))

Lower of cost and net realizable value

Aim: not to overstate the asset (inventory) figure, i.e., be prudent.

For statement of financial position, closing inventory will appear under


current assets and if there’s more closing inventory then it will make
statement of financial position look better.

For statement of profit or loss and other comprehensive income, if there’s


more closing inventory, then there’ll be less cost of sales then overstate the
profit.
3, Where does inventory fit into?

Statement of financial position as at 31 DEC 2014 for Manny company:


Current assets $
Inventory 8,990

Statement of profit or loss and other comprehensive income (extract) for the year
ended 31 DEC 2014 for Manny company:
$ $
Sales revenue $78,559
Cost of sales
Opening inventory 8,009
Purchase 5,889
-closing inventory (8,990)
(4,908)
Gross profit 73,651

Journals:

Opening inventory:
DR cost of sales 8,009
CR closing inventory 8,009

Purchases
DR cost of sales 5,889
CR cash/payable 5,889

Closing inventory
DR closing inventory 8,990
CR cost of sales 8,990
Q housing department [DEC 2012 Q3 extract]

The local government organization supplements its income by buying and selling
property. The housing department regularly sells part of its housing inventory in the
ordinary course of its operations as a result of changing demographics. Part of the
inventory, which is not held for sale, is to provide housing to low-income employees at
below market rental. The rent paid by employees covers the cost of maintenance of
the property.

Required: Comment on the above accounting treatment. (2 marks)

Answer:

Supplement
Sales of property are in the ordinary course of its operations and are routinely
occurring, then the housing stock held for sale will be classified as inventory per IAS2
inventory.

Low-income employees
The part of the inventory held to provide housing to low-income employees at below
market rental and this is held to provide housing services rather than rentals so can’t
be classified as investment property but as property, plant and equipment per IAS16.

Q Internal plc [DEC 2012 Q2 modified]

Internal plc receives lots of certificate from government (free) which can be sold to
other companies. But these certificates are not sold as at the year end.

Required:
How to account for them?
(2 marks)

Answer:
Treatment:
Initial measurement [government grant]
DR inventory
CR deferred income

On sale of certificate
DR deferred income
CR cost of sales (become inventory)

Then
DR cash/receivable
CR inventory
DR/CR P/L (balancing figure)
IAS 8 Accounting policies, changes in accounting
estimates and errors
If the company is going to use another accounting policy this year and find an error
relating to last year’s account then the company should adjust for this year and last
year’s financial statements. (Retrospective adjusting)

If the company is going to use another accounting estimate this year and the company
should adjust for current year financial statements and future one.(prospective
adjusting)

But how to determine whether this is a change in accounting policy or estimate?


Well, if there’s a change in

 Measurement basis of the figure, e.g., value the inventory using FIFO but now
use weighted average method; use replacement cost rather than historic cost.
 Recognition basis of the figure, e.g., recognize as an expense before but now for
asset(e.g., IAS 23 borrowing costs)
 Presentation basis of the figure, e.g., recognize the depreciation expense into
cost of sales now rather than in administrative expenses before.

You are going to change in the accounting policy only if:


1, a change in laws / accounting standards and you are required to do so;
2, gives a fairer presentation to the users of FS.

And anything that is not changing the measurement, recognition or presentation of


figures are deemed to be a change in accounting estimate such as:
 Allowance for receivables;
 Useful life/ depreciation method of the non-current assets;
 Warranty provision relating to return of goods from customers.

An error may happen if there’s a


 Misuse of the accounting standard last year;
 Fraud happened last year;
 Omit some figures in last year’s account.

Accounting Summary:

Changes in accounting policy this year:


Assume it happens in last year as well and of course this year happens;
Adjust for last year closing retained earnings taken into account in the changes to be
brought forward in this year’s statement of changes in equity.

Material prior period errors found:


Correct last year’s material errors;
Adjust for last year closing retained earnings taken into account in the error effect to
be brought forward in this year’s statement of changes in equity.

Changes in accounting estimate:


Use the new one to continue the calculation.
Q Account Ltd (accounting policy and accounting estimate)

1, Account Ltd charged interest expenses incurred from the construction of tangible
non-current assets to the income statement before but now it capitalizes the interest
as an addition to the cost of tangible non-current asset as per IAS 23 borrowing costs.

2, Account Ltd depreciate the machine using the reducing balance basis method at 30%
but now it use the new depreciation method over 10 years.

3, Account Ltd shows overhead expenses within cost of sales before but now it shows
under administrative expense.

4, Account Ltd has previously measured inventory at weighted average cost but now
it uses FIFO method.

Required:
Whether the above transactions are a change in accounting policy or accounting
estimate.

Answer:

1. Change in accounting policy because of a change in recognition basis.


2. Change in accounting estimate because it’s related to experience.
3. Change in accounting policy because of a change in presentation basis.
4. Change in accounting policy because of a change in measurement basis.
Q Martin Construction (change in accounting policy)

Martin Construction incurs significant finance costs on its financing for the
construction of supermarkets. Its chosen accounting policy to date has been
expense the finance costs as incurred. The final accounts for the year ended 31
December 2012, and the 2013 draft accounts, reflect this policy and show the
following.

2013 2012
$000 $000
Profit before interest and tax 8,700 6,200
Finance costs (2,500) (1,750)
Profit before tax 6,200 4,450
Income tax expense (1,900) (1,400)
Profit after tax 4,300 3,050

Retained earnings B/F: 26,050 23,000

The directors of Martin Construction have now decided to change the accounting
policy in 2013 to 19 capitalization of finance costs per IAS23. Martin Construction
incurs no finance costs other than those related to the construction of the
supermarkets.

Martin Construction paid a dividend of $1m during the year ended 31 December
2013.

Required:
Show how the change in accounting policy will be reflected in the income
statement and statement of changes in equity for the year ended 31 December
2013.

Answer:
2013 2012
$000 $000
Profit before interest and tax 8,700 6,200
Finance costs (-) (-)
Profit before tax 8,700 6,200
Income tax expense (1,900) (1,400)
Profit after tax 6,800 4,800

Statement of changes in equity:


Opening retained earnings 23,000
PAT (2012 revised ): +4,800
Revised opening retained earnings 27,800
PAT 6,800
Dividend paid: (1,000)
Closing retained earnings 33,600
Q JJK (prior period errors)

During the year 2013 JJK Ltd discovered certain items that had been included in
inventory at 31 DEC 2012 at a value of $2.5m but they had been in fact sold before the
year end.

The income statement below for JJK for 2012 and 2013 are as follows:
2013 2012
Sales 52,100 48,300
Cost of sales (33,500) (30,200)
Gross profit 18,600 18,100
Tax expense (4,600) (4,300)
Profit after tax 14,000 13,800

The retained earnings at 1 Jan 2012 were $11.2million.

Required:
Show the 2013 income statement with comparative figures and the retained earnings
for each year.

Answer:

P/L:
2013 2012
Sales 52,100 48,300
Cost of sales (33,500) (30,200+2,500)
Gross profit 18,600 15,600
Tax expense (4,600) (4,300)
Profit after tax 14,000 11,300

Statement of changes in equity:


Opening retained earnings in 2012: $11,200
PAT revised (2012) $11,300
Closing retained earnings 2012: $22,500

PAT (2013) $14,000


Closing retained earnings 2013: $36,500
Q Giant (changes in accounting estimate)

Giant Ltd has an asset which was purchased for $80,000 on 1 January 2005 when its
useful life was estimated to be ten years with a residual value of $10,000. A straight
line depreciation policy was selected. On 1 January 2011 the directors reviewed the
useful life of the asset and found that it had a remaining life of eight years.

Required:
Calculate the NBV as at 31 December 2011?

Answer:
$
Cost (1.1.2005) 80,000
Depreciation ($80,000-$10,000)/10yearsX6years (42,000)
Carrying value at 1.1.2011 38,000
Depreciation (new estimate): (38,000-10,000)/8years (4,625)
Carrying value at 1.1.2012 33,375
IAS10 events after the reporting period

Time line:

Audit FS
report authorized
signed to issue
YR start YR end

This is the event happened between financial statement year end and the financial
statements are authorized to be issued to the shareholders to be discussed at the AGM
(annual general meeting).

They will be either adjusting events or non-adjusting events

Magical way to distinguish the adjusting events and non-adjusting events:


Is it because of this event then it will affect the figure as at the year end?

-Adjusting events

Change in judgments, estimate or assumptions after the year end.


e.g., 1, inventory sold at a loss? Change in assumptions that closing inventory should
be valued at the lower of cost and net realizable value (IAS 2);
2, Customers go bankruptcy so that recoverability of the receivable balance at the
year-end has been changed.
3, If company is involved in going concern problems after the year end and because
the financial statement should be prepared under going concern basis and now
this is changed.

-Non-adjusting events

There’s no link between financial statement figures at the year end and events after the FS
year end.
e.g., 1, fire destroyed the inventory after the year end (cant’s predict!)
2, dividends are declared after the year end or share issues after the year end (no
link between figures and events)
Example: (Orange Ltd) (IAS 10 events after the reporting period)

John has asked you to identify the following events happened in orange Ltd of whether
they’re adjusting or non-adjusting events between the accounting year end of 31
December 2013 and 31 March 2014:

1. Major acquisition of a competitor announced on 17 January 2014.


Answer: Non adjusting event

2. Inventory is sold for a price significantly lower than the original cost on 5
January 2014.
Answer:Adjusting event

3. The bankruptcy of a major customer on 9 February 2014.


Answer:Adjusting event

4. A Major fire happened in a warehouse, destroying two thirds of the company’s


inventory on 27 February 2014.
Answer:Non adjusting event

5. You discovered a material sales ledger fraud on 29 January 2014 that took place
throughout the financial year.
Answer:Adjusting event

6. 100,000 ordinary shares issued on 1 March 2014.


Answer:Non adjusting event

7 Dividends were announced on 30 January 2014.


Answer:Non adjusting event
IAS 12 Income taxes

In this session, we’ll be looking at

Current tax
Deferred tax
Basic Idea:

In the statement of profit or loss and other comprehensive income:

$
Sale revenue 1,000
Cost of sales (300)
Gross profit 700
Expenses (100)
Profit before tax 600
Tax expense (@30%) (50)
Profit after tax 550

You can see although tax rate is 30% but we use 30%Xprofit
before tax which does not equal to 50, why?

The reason being within the tax expense there are 3 components:
(mnemonics: CPD)

Current tax payable (based on last year taxable profit)


Provision (under/ (over))
Deferred tax movement

Because of permanent and temporary difference which leads to the


difference in taxable profit calculation and accounting profit calculation.

Permanent differences are the amounts which represent income or expense


for accounting purposes but are not taxable/allowable for tax purposes.
Example: client entertaining.

Temporary differences are amounts which represent income or expense for


accounting purposes and tax purposes but in difference periods. Example:
depreciation and capital allowances.

Notice: The deferred tax transfer is not cash flow!!!

Before we look at deferred tax, why not start off by looking at


current taxation?
Current tax:

Companies have to pay tax on taxable profits. The tax charge is normally ESTIMATED
at the end of the financial year and charged to the statement of profit or loss and other
comprehensive income, and paid in the following year.

The double entry for taxation would be:


DR Taxation expense (Statement of profit or loss)
CR Taxation liability (Statement of financial position)

The double entry for when the tax is paid a few months later:
DR Taxation liability (Statement of financial position)
CR Bank (Statement of financial position)

Since the amount paid is likely to differ from the estimated tax charge originally
recognized, a balance will be left on the taxation liability account being an under or
over provision of the tax charge.

Example of Current tax: (Tracy Ltd)

Tracy Ltd estimated last year’s tax charge to be $250,000 at on December 2011.
On 1 March 2012, Tracy Ltd settled their income tax bill and paid cash to the tax
authorities of $255,000.

On 31 December 2012 Tracy Ltd estimated that this year’s income tax charge to be
$270,000.
On 1 March 2013 Tracy Ltd settled their income tax bill and paid cash to the tax
authorities of $265,000.

Required:
Show how this should be accounted for in the books of The Tracy Ltd,
showing Journal entries.

Answer:
2011: (expense don't have to recognize in this year’s SOCI)
DR tax expense 250,000
CR tax liability 250,000

2012:
DR tax expense 5,000
CR tax liability 5,000 (under provision)

DR tax liability 255,000


CR bank 255,000

DR tax expense 270,000


CR tax liability 270,000

2013:
DR tax liability 5,000 (over provision)
CR tax expense 5,000

DR tax liability 265,000


CR tax expense 265,000
Deferred tax

What is deferred tax?

Illustrate with an example:


Imagine you have a building with a carrying value of $1000. During the year you have
revalued this building to $1,100 then you make a profit from it of $100 which is not
realized yet.

DR NCA 100
CR revaluation reserve 100

So for the tax man’s perspective, because you will somehow in the future realized this
profit when sold so they may require you to provide for a future tax obligation
(deferred tax) of $100Xtax rate although you are not paying money now but you will
in the future.

Concept:
So we know that deferred tax is a future obligation to be settled by company
depending on the future tax law. So deferred tax does not necessarily fulfill the liability
definition (present obligation).

Deferred tax arises because of temporary differences (TD). Temporary difference is


the difference between CV and TB.

DT=TD* X CT%
*TD=CV - TB

TD: Temporary difference between carrying value and tax base


CV: Carrying value of asset/liability.
TB: tax base in the tax man’s book. (In real practice we will try to refer to
different tax regulations to calculate the tax base)
DT: Deferred tax liability/asset
CT%: Corporation tax rate

Deferred tax is a future liability recognized today. And deferred tax is based on
temporary difference (timing difference between accounting and tax law). So the
amount we owe to the tax authority will be finally paid back to them in the subsequent
years.
Q: (formula)
Opening deferred tax (DT) 60
Closing temporary difference (TD) 200
CT rate 35%

Required:
Deferred tax movement.

Answer:
Deferred tax movement= closing DT-opening DT
=200X35%-60
=10
Why do we need deferred tax?

1, Accruals Concept

Because we know assets represent present value of future economic inflows from
using it. That means from using this asset company can generate into future operating
profits. And we know profits are taxed by tax authority. So we need to match the
future operating profits with future tax expenses. And the future tax expenses being
discounted back to today’s value is called deferred tax. So this is according to Accruals
concept (Matching principle).

For liability which represents present value of future economic cash outflows from
settling the obligation. That means when settle the liability the company has to incur
cash outflows which will save us tax. In order to match the future cash outflow to tax
future tax savings we need to provide for present value of future tax savings. Again,
according to Accruals concept (matching principle).

2, Reduce distortion in profit figure

See Example in Thomas.


Basic Deferred tax Example: (Thomas)

Thomas acquired a non-current asset costing $2,000 at the start of year 1. It is being
depreciated straight line over 4 years. Corporation tax rate is 25%. The capital
allowances granted on this asset are:

Year Capital allowances


1 800
2 600
3 360
4 240

The company expects to make a profit before tax of 3,000 in these 4 years.

Required:
1, calculate PAT for these 4 years (ignoring deferred tax)
2, calculate PAT for these 4 years (including deferred tax)
3, Show double entry for deferred tax movement and where do they fit into statement
of financial position.
4, Show how deferred tax will eventually become 0.
5, Explain the conceptual basis within deferred tax.
How to determine the carrying value and tax base?

DT=TDXCT%

DT=CV-TB

Carrying value: (CV)


Value in the statement of financial position (asset or liability)

Asset: positive
Liability: negative

If you are given income and expenses then substitute them in asset and liability
Income: positive
Liability: negative

And if you are given equity figure, then its CV=0.

Tax Base: (TB)


If it’s:
Taxed in future= 0
Not taxed in future= CV
Expansion of tax base:
This would be very difficult as you see the IAS 12 has listed out all of the procedures
to determine tax base for the item. In the real practice which is quite different from the
exam, depending which country you are in, you will be given lots of rules of the future
amount which is deductible or taxable.
But in the exam, your examiner will clearly state whether:

“It’ll have future tax consequences or will be taxed in the future”, or allow
cost in full
Will be taxed= 0

“It has been included in the taxable profit or loss or has been taxed or will not
be taxed”
Will not be taxed= CV

Table:

Asset:

Tax base of an asset is the amount that will be deductible for tax purposes against any
taxable economic benefits that will flow to the entity when it recovers the carrying value of
the asset. Where those economic benefits are not taxable, the tax base of the asset is the
same as its carrying amount.

Taxed future? -0
Not taxed future?-CV

Liability

The tax base of liability will be its carrying amount, less any amount that will be deducted
for tax purposes in relation to the liability in future periods

Taxed future? –[CV-CV=0]


Not taxed future? –[CV-0]

In summary, irrespective of asset or liability, if it’s: (exam trick)


Taxed in future= 0
Not taxed in future= CV
Q practice (Deferred tax)

Required:
Calculate the deferred tax.

1. Accelerated capital allowances (ACA)


LEKO Ltd purchased a building at the year start for $200,000. The life of the building
is 20 years and there is no residual value and one year has gone. Capital allowances
are allowed at 40% for the year. Corporation tax rate is 30%.

2. Revaluation
Jim purchased a building at the year start for $900,000. The life of the building is 10
years and there is no residual value. Capital allowances are allowed at 40% for the
year. At the year end the building is revalued to $950,000. Corporation tax rate is
30%.

3. Development
Feikon Company spends $15million on a development project which will lead to three
years of sales including the current year. The tax man allows this cost in full as
incurred. Corporation tax rate is 30%.

4. Provisions
Environmental provision $60million
Corporation tax rate 30%
Tax man recognizes environmental costs as the cash flow.

5. Tax losses
The tax man is carrying forward losses of $100million. Corporation tax rate is 30% and
the company is considered to be generating into profits in the future.
6. Trade receivables
Trade receivables have a carrying amount of $10,000. The related revenue has
already been included in taxable profit. Corporation tax rate is 30%
A loan receivable has a carrying amount of $1m. The repayment of the loan will have
no tax consequences. Corporation tax rate is 30%.

7. Accrued expense
① Current liabilities include accrued expenses with a carrying amount of $1,000. The
related expense will be deducted for tax purposes on a cash basis.

② Current liabilities include accrued expenses with a carrying amount of $2,000. The
related expense has already been deducted for tax purposes.

③Current liabilities include accrued fines and penalties with a carrying amount of $100.
Fines and penalties are not deductible for tax purposes.

8. Loan payable
A loan payable has a carrying amount of $1m. The repayment of the loan will have no
tax consequences.
Recognition of deferred tax assets for unrealised losses(Current Issue)

Background
The proposed amendments would clarify that unrealised losses on debt instruments
measured at fair value and measured at cost for tax purposes can give rise to
deductible temporary differences.

The proposed amendments would also clarify that the carrying amopunt of an asset
does not limit the estimation of probable future taxble profits. It would specify that
when comparing deductible temporary differences with future taxbale profits, the
future taxable profits would exclude tax deductions resulting from the reversal of
those deductible temporary differences.

Why amendments?

The IFRS interpretation committee revirewed a request to clarify the application of IAS
12 for the recognition of a deferred tax in the following situations:

 An entity holds a debt instrument classfieid as avaialbel for sale with gains and
losses being reognised in the other comprehensive income;
 Changes in market interest rates cause the fair value of the debt instrument to
be below its costs;
 The entity expects to recover all of the contractual cash flows by holding the
instrument until maturity;
 The entity does not consider the debt instruemnt to be impaired;
 The tax base of the debt instrument remains cost;
 The tax law doesn’t allow a loss to be subtracted until it is realised;
 The entity has insufficient taxable temporaray differences and no other
probable future taxable profits against which the entity can use up the
deductible temporary differences.

What are amendments?

The IASB proposes to clarify that the unrealised losses resulting from the sitation
described above give rise to a deductible tax difference irrespective of whether the
holder expects to recover the carrying amount by holding the debt instrument until
maturity or by selling the debt instrument.

If tax law restricts the utilisation of tax losses so that an entity can only subtract tax
losses against income of a specified type, an entity would assess a deferred tax asset
together with other deferred tax assets of the same type.

The proposed amendments would also clarify that when estimating taxable profit of
future periods, and entity can assume that an asset will be recovered for more than its
carrying amount if that recovery is probable and the asset is not impaired. All relevant
facts and situations should be assessed when making this assessment.

In evaluating whether sufficient future taxable profits are available, an entity should
compare the deductable temporary differences with the future taxable profits
excluding tax deductions resulting from the reversal of those deductible temporary
differences.
Q Kasare (Deferred tax)
This question comes from Q2 of you P2 pilot paper. Note this version has been
updated to reflect the latest version of accounting standards.

The following statement of financial position relates to Kesare Group, a public limited
company, at 30 June 2006.
$000
Assets
Non-current assets:
Property, plant and equipment 10,000
Goodwill 6,000
Other intangible assets 5,000
Financial assets (cost) 9,000
30,000
Current assets
Trade receivables 7,000
Other receivables 4,600
Cash and cash equivalents 6,700
18,300
Total assets 48,300

Equity and liabilities


Equity
Share capital 9,000
Other reserves 4,500
Retained earnings 9,130
Total equity 22,630

Non-current liabilities
Long term borrowings 10,000
Deferred tax liability 3,600
Employee benefit liability 4,000
Total non-current liabilities 17,600
Current liabilities
Current tax liability 3,070
Trade and other payables 5,000
Total current liabilities 8,070
Total liabilities 25,670
Total equity and liabilities 48,300
The following information is relevant to the above statement of financial
position:
(i) The financial assets are classified as 'investments in equity instruments' but are
shown in the above statement of financial position at their cost on 1 July 2005. The
market value of the assets is $10.5 million on 30 June 2006. Taxation is payable on
the sale of the assets. As allowed by IFRS 9, an irrevocable election was made for
changes in fair value to go through other comprehensive income (not reclassified to
profit or loss).

(ii) The stated interest rate for the long term borrowing is 8 per cent. The loan of $10
million represents a convertible bond which has a liability component of $9.6 million
and an equity component of $0.4 million.

The bond was issued on 30 June 2006.

(iii) The defined benefit plan had a rule change on 1 July 2005, giving rise to past
service costs of $520,000. The past service costs have not been accounted for.

(iv) The tax bases of the assets and liabilities are the same as their carrying amounts
in the draft statement of financial position above as at 30 June 2006 except
for the following:
(1)
$000
Property, plant and equipment 2,400
Trade receivables 7,500
Other receivables 5,000
Employee benefits 5,000

(2) Other intangible assets were development costs which were all allowed for tax
purposes when the cost was incurred in 2005.

(3) Trade and other payables includes an accrual for compensation to be paid to
employees. This amounts to $1 million and is allowed for taxation when paid.

(v) Goodwill is not allowable for tax purposes in this jurisdiction.


(vi) Other adjustments
①Kesare buys a sub just after year end. The sub has inventory as at cost of $60,000,
but fair value of $65,000. All the inventory is still in inventory a few days later at the
year end.

②Kesare bought a foreign sub for $500,000, measured in the parental home currency
in the second year. The subsidiary has grown to $570,000 because of net profits of
$70,000 that the sub has retained. The withholding tax rate is 60%. Show the
deferred tax implication at 30June2007.

③The company had granted 20million options in the scheme and they are all expected
to vest at the end of the fourth year. The fair value of the options at the grant date was
$10 and the current intrinsic value of each is $8. Show the deferred tax implication at
30June2007.

④Kesare is leasing plant under a finance lease over a five year period. The asset was
recorded at the present value of the minimum lease payments of $12 million at the
inception of the lease which was 31 May 2005. The asset is depreciated on a straight
line basis over the five years and has no residual value. The annual lease payments
are $3 million payable in arrears on 1 June and the effective interest rate is 8% per
annum. The directors have not leased an asset under a finance lease before and are
unsure as to its treatment for deferred taxation. The company can claim a tax
deduction for the annual rental payment as the finance lease does not qualify for tax
relief.

(vii) Assume taxation is payable at 30%.

Required
(a) Discuss the conceptual basis for the recognition of deferred taxation using the
temporary difference approach to deferred taxation. (5 marks including)

(b) Calculate the deferred tax liability at 30 June 2006 after any necessary
adjustments to the financial statements showing how the deferred tax liability would
be dealt with in the financial statements. (Assume that any adjustments do not affect
current tax. Candidates should briefly discuss the adjustments required to calculate
deferred tax liability.) (30 marks)
(Total = 35 marks)
IAS16 property, plant &equipment

1, Initial measurement:

Capital expenditure
Capital expenditure is the costs of acquiring non-current assets.
According to IAS 16 the following costs may be capitalized in the statement of financial
position on acquisition of a non-current asset:
(Mnemonic: IIIID)
Initial cost (purchase price)
Import duty not refundable (if asset is bought from other country)
Installation costs
Intended use relating costs (lawyer, surveyor costs)
Delivery costs

Finance cost (IAS 23 see F7 & P2)

Revenue expenditure
Revenue expenditure is expenditure on maintaining the capacity of noncurrent assets.
Costs that are regarded as revenue expenditure should be expensed in the statement
of profit or loss and other comprehensive income and may not be capitalized according
to IAS 16 are:
(Mnemonic: RIM)
Repairs expenses
Insurance expenses
Maintenance expense

After we’ve purchased the non-current asset the accountant needs to record that
non-current asset into the non- current asset register.
A non-current asset register is generally maintained in the finance department.
Companies can purchase specifically designed packages or a register can simply be
maintained on an Excel spreadsheet.
And this is used to reconcile the NCA in the NCA register to the individual asset in place,
ie, an example of control procedure by company.

Sample of Non-current asset register:


Asset type Date purchased Description Cost Depreciation Carrying Disposal Disposal date
value proceeds
Machine 1 July 2013 Drink $7m
machine
Year ended 31 DEC 2013 $700,000 $6.3m
Year ended 31 DEC 2014 $3m Jan-2014
2, Subsequent measurement

Cost model: cost-accumulated depreciation*=carrying value

Depreciation method should be reviewed each year to see whether or not it is


reasonable. A change in depreciation method should be treated as a change in
accounting estimate and prospective adjusting method according to IAS 8 should be
applied. I.e., disclose the depreciation method in the note of the financial statements.

Revaluation Model: revalued amount

*Depreciation
*Revaluation
*Disposal
*Impairment [IAS 36]
*Non-current asset held for sale & discontinued operations [IFRS5]

[Note]:

IAS 16 the test was whether the expenditure was Capital or Revenue e.g. an
improvement could be capitalized but maintenance or repair could not be capitalized.

The following circumstances should be capitalized:

*Revaluation Model:
(Mnemonics: LOSE)
L: Life extension
O: major overhaul cost
S: separate component, e.g., new engine for an aircraft
E: energy saving, e.g., improving production capacity

Basic idea:
1, economic benefits are excessed
2, component treated separately
3, major overhaul cost
1, Depreciation:
Basic Idea:
Depreciation is the charge to the statement of profit or loss and other comprehensive
income to reflect the use of an asset in a period and this is based on ACCRUALS
CONCEPT which applies the cost of using the asset to the statement of profit or loss
and other comprehensive income for the same period as the revenue earned from the
asset.

Methods of depreciation

1, straight line basis depreciation:

Idea: An equal amount is charged in every accounting period over the life of the
asset.

Calculation:
Depreciation per year = original cost – estimated residual value
Estimated useful life

Or = % X cost

2, reducing balance basis depreciation

Idea: at the start of year we charge more depreciation and at the end of the year we
charge less depreciation given the fact that machine will be less efficient at the end of
its life, i.e., less revenue can be earned so less expenses matched against with it.

Calculation:
Depreciation per year = % X carrying value

Journal
Dr Depreciation expense (Statement of profit or loss and other
comprehensive income)
Cr Accumulated depreciation (Statement of financial position)

Financial statements
$
Non–current assets
Property, plant & equipment(note1) 184,490
Note1:
Cost Accumulated Carrying value
depreciation
Non-current assets:
Property 150,000 (12,000) 138,000
Motor vehicles 45,000 (11,250) 33,750
Plant & Machinery 26,000 (13,260) 12,740
2, revaluation
Basic Idea:

As time goes by initial costs of asset may be very different from their market value.

Eg, if a company purchased a property 35 years ago and therefore subsequently


charged depreciation for 35 years, it would be safe to assume that the carrying value
of the asset would be significantly different from today’s market value.

If revaluation policy per IAS 16 may be adopted (i.e. the business has a choice), and
if so the following rules must be applied per the standard: (mnemonic: CRRR)

1, No Cherry picking (If a company chooses to revalue an asset they must revalue all assets in that category.)

2, Regular (Revaluations must be regular but IAS 16 doesn't specify how often)

3, Revalued amount (Subsequent depreciation must be based on the revalued amounts.)

4, Revaluation Reserve (Gains from revaluations are taken to revaluation reserve rather than retained earnings
unless they are sold)

Calculation:
$
Revalued amount X
CV of asset on revaluation date (X)
Revaluation gain/(loss) X/(X)

Journal
DR Asset cost (Statement of financial position)
DR Accumulated depreciation (Statement of financial position)
CR Revaluation reserve (Statement of financial position)
3, Disposal

Basic Idea

An asset should be removed from the statement of financial position on disposal or


when it is withdrawn from use and no future economic benefits are expected from its
disposal

Calculation

Cash sale/part exchange X

CV of asset at disposal date (X)

Profit/(loss) on disposal X/(X)

Journal
(Mnemonic: CAP)
1, C: Cost Removal

DR Disposals
CR Non-current asset Cost

2, A: Accumulated Depreciation Removal

DR Accumulated Depreciation
CR Disposals

3, P: Proceeds to be dealt with in cash/part exchange

In cash:

DR Bank
CR Disposals

Part exchange:

DR Asset Cost
CR Bank
CR disposals
Q June2010 Q1 (VI) (IAS16)

(vi) Ashanti owned a piece of property, plant and equipment (PPE) which cost $12
million and was purchased on 1 May 2012. It is being depreciated over 10 years on the
straight-line basis with zero residual value. On 30 April 2013, it was revalued to $13
million and on 30 April 2014, the PPE was revalued to $8 million. The whole of the
revaluation loss had been posted to other comprehensive income and depreciation has
been charged for the year. It is Ashanti’s company policy to make all necessary
transfers for excess depreciation following revaluation.

Answer:

Cost [1.5.2012] 12
Depreciation(12/10yrs) (1.2)
CV @1.5.2013 10.8
Revaluation reserve [bal] 2.2
Revalued at 1.5.2013 13
Depreciation[13/9yrs] (1.4)
CV@1.5.2014 11.6
Impairment of PP&E (3.6)
Revalued @1.5.2014 8
DR RR (2.2-0.2) 2
DR P/L (bal) 1.6
CR PP&E 3.6

Excess depreciation=excess RR (2.2)


9years
=0.2
DR RR 0.2
CR RE 0.2
Q June2011 Q1 (5) extract

Rose purchased plant for $20 million on 1 May 2007 with an estimated useful life of six
years. Its estimated residual value at that date was $1·4 million. At 1 May 2010, the
estimated residual value changed to $2·6 million. The change in the residual value has not
been taken into account when preparing the financial statements as at 30 April 2011.

Answer:

Cost (1.5.07) 20
Cum depreciation (1.5.10) (9.3)
(20-1.4)/6 X3
10.7

New depreciation: (10.7-2.6) /3 =2.7


Old depreciation (10.7-1.4)/3= 3.1
Save depreciation =0.4

DR PPE 0.4
CR RE 0.4

Q Dec2011 Q1 extract

Traveler acquired a new factory on 1 December 2010. The cost of the factory was $50
million and it has a residual value of $2 million. The factory has a flat roof, which needs
replacing every five years. The cost of the roof was $5 million. The useful economic life
of the factory is 25 years. No depreciation has been charged for the year. Traveler
wishes to account for the factory and roof as a single asset and depreciate the whole
factory over its economic life. Traveler uses straight-line depreciation.

Answer:

Depreciation

Roof (5m/5yrs) 1m
Factory(50-5-2)/25yrs 1.72m

2.72m

DR P/L 2.72
CR NCA 2.72
IAS 17 leases

Introduction

You want to have a photocopier and you have two choices:


1, you can buy it and then you become the owner of the photocopier;
2, you can lease it from the lessor and then you would become the lessee.
Long term-finance lease
Short term-operating lease

But the key to differentiate between them is not just the time length it takes but rather
“substance over form”.

IAS 17 leases describes two types (forms) of leases:

*Finance lease: lease that transfers the risks and rewards of the asset from the
lessor to the lessee.
*Operating lease: any leases other than finance lease.

5 scenarios

So the substance over form concept behind it can be summarized as follows:


IAS 17 prescribes there are 5 common scenarios that the lease is a finance lese.
(one of them fulfilled then it’s a finance lease and if none of them fulfills then it’s an
operating lease.)

1, ownership of asset has been transferred from lessor to lessee.


2, lessee has the option to purchase asset at a price which is sufficiently lower than its
FV.
3, lease term is almost the same as the major part of economic life of asset.
(IFRS doesn't specify the period but US GAAP has given us guidance of >75 %.)
4, at the start of the lease, PV of minimum lease payment is close to FV of asset.
(again, IFRS doesn't specify the percentage but US GAAP has given us a guidance
of >90%.)
5, leased assets are specified nature and can only be used by lessee and they can be
used by others if any significant modification to assets occurs.
Risks and rewards

But the idea behind it is when the majority risks and rewards has been transferred
from the lessor to lessee then it’s considered to be a finance lease.
So the typical risks and rewards may include:

Risks:
Costs of repairing, maintaining and insuring the assets.
Risk of obsolescence
Risks of losses from idle capacity of the asset (if machine breaks down then lessee
bears the loss)

Rewards:
Use of assets for almost all of its useful life.
Use of the assets is not disrupted.

Accounting Treatment:
In P2, you are required to know the accounting treatment for both lessor and
lessee.

Lessee Lessor
Finance lease:
Initial measurement DR PPE DR lease receivable
CR lease liability CR lease asset
Subsequent measurement PPE:
DR P/L-depre expense
CR accumulated
depreciation

Lease liability:
DR lease liability DR cash (from lessee)
DR P/L-finance cost CR lease receivable
CR cash CR P/L-interest income
Operating lease:
Expense the lease Expense the lease revenue
payment on a straight line received on a straight line basis
basis DR cash
CR P/L
DR P/L Keep the assets in FS and
CR cash depreciates it.
DR P/L-depreciation expense
CR accumulated depreciation
Sale and leaseback transaction:

Sale and finance leaseback:


 Sell asset and lease for most/all of useful life
 ‘Profit’ on sale is spread over the lease period
 ‘Sale’ element of transaction

DR cash (sale proceeds)


CR PPE (CV)
CR/DR deferred income (gain on sale-release over lease term)

 Lease element of transaction


DR PPE (PV of min lease payment)
CR lease liability

Sale and operating lease back:


(a) Sale as fair value:
 Derecognize the asset at sale date and take the profit immediately to income
statement.
 SP=FV>CA

(b) Sale above fair value:


 Defer the ‘additional’ gain and spread over the lease period.
 SP>FV>CV

(c) Sell below fair value:


 If the seller is short of cash and therefore has to sell at a loss take the WHOLE
loss to the income statement on disposal.
 If the seller has the benefit of below market rentals for the lease of the asset
then “defer” the loss over the lease period.
 SP<FV>CV
Q: Finano (finance lease)

Finano has the option of buying a drinking machine for a cash price of $14,276 or
leasing it on a financial lease paying $5,000 at the end of every year for 4 years
and the present value of minimum lease payment is $14,275 as well. The rate of
interest implicit in the lease arrangement is 15% per annum.

Finano plans to buy a very sophisticated egg machine to expand its business. Two
advertisements appear in the local newspaper for the egg machine at $11,000.
Finano negotiates the price down to $10,425, and it will pay $2,500 immediately
on 1 January 2013, with 4 more instalments on the anniversary of signing the
agreement. The implicit rate of interest is 10% per annum.

Required:
Show how the company should account for this lease in its Income Statement and
Statement of Financial Position for the drinking machine and egg machine for the year
ended 31 Dec2013.
Q: Opera (operating lease)

Opera has taken out an operating lease on its photocopier paying a non-refundable
deposit of $3,000 to John. The lease is for three years with annual payments of $5,000
after which the photocopier goes back to the lessor John. The photocopier has a useful
economic life of five years.

Required:
Show how the photocopier will be accounted for in the Statement of Financial Position
and Income Statement of Opera, at end of year 1.
Q: Finco Ltd [sale and leaseback]

Finco Ltd has 4 sale and leaseback transactions during the year which can be shown as
follows:

Description Sale Fair value Book(carrying)


proceeds $m value
$m $m
1, sale and finance lease back 50 50 32
2, sale at fair value operating lease back 80 80 55
3, sale at overvalue and operating lease back 85 65 70
4, sale at undervalue and operating lease 65 85 60
back

Required:
Show how to deal with the above transactions.
IFRS 15 — Revenue from Contracts with Customers

1: Definition of contract

 All parties should have approved the contract


 All parties are committed to perform their obligations, but not all of the
obligations.
 Rights and payment terms are identified. The price does not need to be
agreed (e.g. construction contracts).
 Must have commercial substance.
 Probable that revenue will be collected.
2: Five step model

(Step 1) Identify contract (can combine two contracts entered into at same
time).

(Step 2) Identify performance obligations.

 Unbundle the elements


 Distinct obligation if customer can benefit from good or service on its
own.
 Series of distinct goods or services that are substantially the same
treated as single performance obligation.

(Step 3) Determine price

 Amount expected to be received in exchange for


goods/services
 Exclude amounts receivable for third parties (such as taxes).

 Includes variable/contingent consideration (use probability weighted


approach).
Q Jargar
Example: Jargar Inc enters into a contract on 1 January 2015 to sell cans of beans
to Dodger Inc. The contract is for one year at an agreed price of $2 per can if
Dodger buys less than 300,000 cans over the period, $1.80 per can if Dodger buys
300-400,000 cans and $1.50 per can if Dodger buys more than 400,000.

During January 2015 Jargar sells 28,000 cans of beans to Dodger. It believes
there is a 20% probability of selling less than 300,000 cans, a 50% probability
of selling 300-400,000 and a 30% probability of selling more than 400,000.

Requirement: Calculate the revenue figure Jargar should include in its statement
of profit or loss for January 2015 .
 Take into account performance bonuses/penalties.
 Include non-cash consideration, such as a financing component.

Q FVM & Tommy


FVM Inc enters into a contract to sell a machine to Tommy Inc on 1 January 2015
for $800,000. FVM's usual terms are payment within 30 days, but to incentivise
Tommy to sign the contract FVM stated that Tommy could pay on 1 January 2017.
FVM has a risk adjusted cost of capital of 12%.

Requirement: Calculate how FVM should account for the contract in its
financial statements for the year ended 31 December 2015.
(Step 4) Allocate transaction price to performance obligations.

 Use relative standalone selling prices of goods and services.


 Calculate at inception of contract.
 Allocate discounts and variable consideration
proportionately.

(Step 5) Recognise revenue as each performance obligation is satisfied.

 Goods----when ownership risks and rewards transferred to customer


 Services----satisfied over a period of time unless
o Customer simultaneously receives and consumes
benefits provided by the entity
o Entity's performance creates/enhances an asset that customer
controls

Recognise revenue in line with pattern of transfer.


Q BOBO Inc
Example: BOBO Inc sells satellite TV set top boxes and TV channel subscriptions.
The set top box usually retails for $560, and the subscription $60 a month.
Customers can however pay for a two year deal which includes both the box and
the TV channels for $62.50 a month. BOBO Inc has a year end of 31 December.

Requirement:
Show how BOBO should account for a two year contract signed with a
customer on 1 April 2015.

57
IAS 19 Employee Benefits and IFRS 2 Share Based Payment

Some companies will offer benefit to its employees. These benefits may include:

1, Short term benefit: (accounting: expense and liability)


Monetary benefit:
Wages and salary
Paid sick leave
Compensated absence.
Non-monetary salary:
Medical care, housing, cars etc

2, Long term benefit: shares (IFRS2), bonus, etc. (Accounting: easier than pension
accounting. Only to show costs, asset and liability if company has invested their money
in other instrument before paying for cash)

3, Termination benefit: redundancy payments; early retirement payments etc.


(accounting: expense and liability)

4, Post-employment benefit: pensions etc. (EXAM POINT!)


Talking about pensions there’re two types of pensions within p2 exam.
They are defined contribution scheme and defined benefit scheme (we’ll talk about
those below)

58
1, Defined contribution scheme

The company doesn't guarantee the final payment to employees.

Company and employee will put the money into the scheme run by the trustee each
year and when employees retire then they get the money which is not fixed (defined).

If the trustee did a bad job, e.g., invest money into shares and suffered a loss then
company didn't have enough money to pay off to the employee then company will have
no further obligation for those money.

Risk is not born by the company but by the employee.

The accounting for this is to DR P/L CR cash each year.

Accounting: DR P/L CR Cash (income statement charge would equal to cash paid.)

Company does not recognize the asset and liability in their account because they have
transferred all the risks and rewards of these to the trustee or agent.

Example: Doc Ltd

Doc ltd makes contributions to the pension fund of employees at a rate of 10% of gross
salary. The gross salary total is $3m for the year.
Total contributions made by Doc Ltd is $200,000.

Required: Show the double entry relating to the above transaction.

Answer:

DR P/L-expense (10%X$3m) $300,000


CR cash $200,000
CR Accrual $100,000

59
2, Define benefit scheme

The company will guarantee the amount of money paid to employees when they retire
and this will be based on number of years that employee has worked for company and
the final salary as well.

The company will put money into the pension scheme each year to create pension
asset (e.g., buying shares etc.) to be paid off to employees (settle pension liability)
when employees get retired.

The question is whether company will have sufficient pension asset to pay off the
pension liability? So the company will employ an actuary to value the pension asset and
liability each money and any deficit occurred would require the company to put money
into it again.

Dose the company recognize the asset and liability in its financial statements? Well the
answer is yes because the company hasn't transferred the risks and rewards to the
trustee because company has to paid off to settle the pension liability even if the
trustee has done a bad job (e.g., lose money into its assets.)

And this is according to substance over form.

The actuary would value the pension asset and liability based on a number of
assumptions:
 Level of investment return from pension assets
 Number of income/outgoing employees etc.

60
The accounting for this is to separate assets and liabilities. (Remain in company’s
account)

Disclosure:

Asset Liability
b/f bal b/f bal
Return on asset Interest cost
Contributions in Service cost
Benefits out Benefits out
Actuarial gains/losses Actuarial gains/losses
c/f bal c/f bal

Accounting:

 b/f bal (c/f from last year by actuary)

 Return on asset(discount rate X b/f): DR asset CR P/L

 Interest cost (discount rate X b/f): DR P/L CR liability

 Contributions in (company putting money in): DR asset CR cash (only cash item)

 Service cost (including current & past service cost: employees work for you and you
have to pay for them): DR P/L CR liability
 Current service cost: This is expense you need to pay for because employee has
provided service to you in the current year.
 Past service cost: This is expense you need to pay for because there is
amendment or curtailment of the pension plan.
 Curtailment and settlement gains/losses: This arises when significant
reductions are made to the number of employees covered by the plan or the
benefits promised to them. DR/CR P/L
 CR/DR Pension Liability

 Benefits out (money paid to those retired): DR liability CR asset

 c/f(by actuary then b/f to next year)

 Actuarial gains/losses:
Gain: DR liability CR OCI
Loss: DR OCI CR liability

61
Q Mini Ltd (short term benefit, similar to termination benefit)

The employees of Mini Ltd are entitled to a 10 days compensated absence in the year
costing $10,000.

Required:
So how does the company deal with this transaction?
(i) Supposing the cash is not paid at the year end
(ii) Company pays for the employees in the next year.

Answer to Mini Ltd:

(i)
DR P/L $10,000
CR Liability $10,000

(ii)
DR Liability $10,000
CR Cash $10,000

62
Q BBQ [Introductory Q on defined benefit scheme]

This is a defined benefit scheme relating to BBQ Ltd.

$m
Opening PV of pension obligation 990
Closing PV of pension obligation 1100
Opening FV of pension asset 1000
Closing FV of pension asset 1190
Current service cost 130
Pension benefits paid 150
Contribution paid by company 90
Discount rate 10%

BBQ Ltd has anther defined contribution scheme in which it pays $30m each year end.

Required:
Show how to account for the above pension schemes:
1, showing the disclosures of the defined benefit scheme and financial statement
effects relating to it.
(5marks)
2, Journals relating to it.
(2marks)

3, show the treatment relating to defined contribution scheme. (1mark)

63
Answer to BBQ:

(i)
P/L:
Return on asset (100)
Unwinding expense 99
Service Cost 130
129

OCI: Net actuarial Gains (150-39) 111

SFP:
Net pension asset(1,190-1,100) 90

Disclosures:
Asset Liability
Opening balance 1,000 Opening liability 990
Return on asset 100 Unwinding expense 99
Benefits paid (150) Benefits paid (150)
Contribution in 90 Service cost 130
Expected closing balance 1,040 Expected closing 1,069
Remeasurement component 150 Remeasurement component 39
Actual closing 1,190 Actual Closing 1,100

2 Journals:
Return on assets: DR asset CR P/L 100
Unwinding expense: DR P/L CR Liability 99
Benefits paid DR liability CR Asset 150
Contributions in: DR asset CR Cash 90
Service Cost: DR P/L CR liability 130
Net Actuarial gain: DR asset CR OCI 111

3: DR P/L CR cash 300

64
Q Mal (past Q Macaljoy Rewritten [IAS 19])

Mal, a public limited company, is a leading support services company which focuses on
the Human Resource Management industry.

The company would like advice on how to treat certain items under IAS 19 Employee
benefits. The company operates the Mal Pension Plan B which commenced on 1
November 2012 and the Mal Pension Plan A, which was closed to new entrants from
31 October 2012, but which was open to future service accrual for the employees
already in the scheme. The assets of the schemes are held separately from those of the
company in funds under the control of trustees. The following information relates to the
two schemes.

Mal Pension Plan A


The terms of the plan are as follows.
(i)Employees contribute 6% of their salaries to the plan.
(ii)Mal contributes, currently, the same amount to the plan for the benefit of the
employees.
(iii)On retirement, employees are guaranteed a pension which is based upon the
number of years’ service with the company and their final salary.

The following details relate to the plan in the year to 31 October 2013:
$m
Present value of obligation at 1 November 2012 200
Present value of obligation at 31 October 2013 240
Fair value of plan assets at 1 November 2012 190
Fair value of plan assets at 31 October 2013 225
Current service cost 20
Pension benefits paid 19
Total contributions paid to the scheme for year to 31 October 2013 17

Mal Pension Plan B


Under the terms of the plan, Mal does not guarantee any return on the contributions
paid into the fund. The company’s legal and constructive obligation is limited to the
amount that is contributed to the fund. The following details relate to this scheme:
$m
Fair value of plan assets at 31 October 2013 21
Contributions paid by company for year to 31 October 2013 10
Contributions paid by employees for year to 31 October 2013 10

The interest rate for Plan A is 5%.

65
The Mal Pension Plan A is wound up at the year end. The market value of the plan
assets is unchanged by the curtailment. But the liability is affected. The employees
departing the scheme agree to receive the plan assets in full plus a further payment of
$16m. The cash was paid just before the year end.

1 year later, Mal has a new defined benefit pension scheme with new employees. This
scheme is in surplus with an asset value of $100m and a liability value of $85m. And
because the asset exceeds the liability, it is expected that in the future it will be
possible to reduce contributions into the scheme.
The present value of the reductions in future contributions is only $10m.

Required:
(a) Discusses the nature of and differences between a defined contribution plan and
a defined benefit plan with specific reference to the company’s two schemes.
(10 marks)

(b) Shows the accounting treatment for the two Mal pension plans for the year ended
31 October 2013 under IAS 19 Employee benefits (revised 2011). (7 marks)

(c)Show how to account for the curtailment in the financial statements. (3marks)

(d) Show the effect of the above asset ceiling on the current financial statements.
(3marks)

66
Answer to Mal:

(a)

Nature
1. Defined contribution scheme
 Employer and employee pay in set amounts.
 Pension received is determined by the success of pension fund manager.
 Accounting: DR P/L CR cash

2. Defined Benefit Scheme


 Employer contributes vary due to the need of the pension scheme to cover future
liabilities.
 Pension paid is lined to salary and length of service. Eg, 2% of final salaries for
each complete service.
 The company will employ an actuary to value the pension asset and liability at the
year end and actuarial gains and losses are taken to OCI.

Difference
The main difference between the two is:
1. Risks
The risks associated with defined contribution plan are with the employees.
The risks associated with defined benefit plan are with the employers.

2. Amounts
The future amounts paid by employer in the defined contribution scheme is not fixed
but in the defined benefit scheme is fixed.

Reference

1. Mal Plan A:
 This is a defined benefit scheme because employees hav e a guaranteed pension.
 The pension A closure is not a curtailment cost because it doen’t affect the current
employees but just the new ones.

2. Mal Plan B:
 This is a defined contribution scheme because there is no guarantee to the
employee and Mal pays a fixed amount to the scheme.

67
(b)

Mal Pension A:

P/L:
Return on asset (10)
Unwinding expense 10
Service Cost 20
20

OCI: Net actuarial Losses (27-29) (2)

SFP:
Net pension liabilities(225-240) 15

Disclosures:
Asset Liability
Opening balance 190 Opening liability 200
Return on asset 10 Unwinding expense 10
Benefits paid (19) Benefits paid (19)
Contribution in 17 Service cost 20
Expected closing balance 198 Expected closing 211
Remeasurement component 27 Remeasurement component 29
Actual closing 225 Actual Closing 240

Mal Pension B:

$10 goes into operating costs


$10 goes into wages and salaries costs

DR operating costs $10


CR cash $10

DR Wages and Salaries costs $10


CR Cash $10

68
(C) Curtailment:

FV of assets at year end 225


PV of obligation at year end (240)
Net obligation 15
Further Payment (16)
Settlement losses (1)

(d)

Asset 100
Liability (85)
Net pension asset 15
De-recognition loss (5)
Asset Ceiling(PV of future reductions in the scheme) 10

DR P/L 5 CR Asset 5

69
IAS 20 Government grants

What is government grant?

Government grant is the cash or asset given by government to help company if it fulfills
the conditions set by government.

This may be categorized as:


Capital grants- grants which are made to contribute towards the acquisition of asset
Revenue grants- grants which are made for other purposes like paying wages.

When recognized?

A grant can be recognized in the FS when:


1, entity complies with the condition set by government
2, the grants will be received.

Usually we will use the deferred income method to reverse the deferred income over
the useful life of asset.
And this is based on “Accrual” concept or Matching principle.

Disclosure:

Accounting policy adopted, including method of presentation (net off or separate


method?)

Nature and extent of government grants recognized and other forms of assistance
received (e.g., buy a machine?)

Unfulfilled conditions and other contingencies attached to recognized


government assistance (eg, repayment?)

70
Accounting treatment:

Capital grant (grants related to assets):

2 treatments both acceptable in IAS but treatment 1 is banned in UK

treatment1, (net off method)


Step1: Decrease grant from NCA which becomes net cost
DR cash
CR non-current asset

Step2: Depreciate the net cost


DR I/S-depreciation expense
CR accumulated depreciation

treatment2, (separate method)


Step1: Treat the grant separately
DR cash
CR deferred income

Step2: Release deferred income matched with depreciation expense of


asset: (Split between current and non-current liability)
DR deferred income (over life of asset)
CR I/S(revenue)

71
Revenue grant(grants related to income)

 If the grant is paid when evidence is produced that certain expenditure has been
incurred, the grant should be marched with that expenditure.

 If the grant is paid on a different basis, eg, achievement of a non-financial


objective, such as the creation of a specified number of new jobs, the grant should
be matched with the identifiable costs of achieving that objective.

Treatment
DR cash
CR deferred income

And subsequently release the deferred income matched with expenses as it is


occurred.

Repayment of grant:

After entity has received grant from government, if it doesn't fulfill certain conditions
then the entity would need to repay the grant and hence the following journals need to
be created.

If a grant becomes repayable, it should be treated as a change in estimate.

Capital Grant Revenue Grant


DR Deferred income/ non-current asset(if net off is used) DR Deferred income
DR P/L DR P/L
CR Bank CR Bank

72
Question: Gran

Gran receives a grant from government of $300,000 towards the acquisition of a


sophisticated machine to manufacture drugs costing $500,000. And the machine has a
useful life of 5 years.

Required:
Show the statement of financial position extract and statement of profit or loss extract
for Gran using method1: netting off and method 2: separate method.

Answer to Gran:

Method1: Net off:

SFP:
NCA
PPE ($500-$300-$40) $160

P/L:
Depreciation expense $40 $500-$300)/5years

Method2: Separate Method:

SFP:
NCA
PPE 400 ($500-$100)

Liability (300-60=240)
Non-current Liability 180
Current Liability 60

P/L:
Grant income 60

Start:
DR cash 300 CR deferred income 300

Year end:
DR deferred income ($300/5years) 60
CR Grant income 60

73
Question: Repay (repayment of revenue grant)

Repay ltd has a balance on the deferred income account of $5,000 relating to the
balance of a grant given to them in order to produce 200 extra jobs. Due to a downturn
in the economic climate, the entity only managed to produce 20 extra jobs and
government asks for a repayment of $12,000.

Required:
Show the double entry about the above transaction.

Answer:
DR Deferred income (decrease) $5,000
DR P/L loss $7,000
CR Bank $12,000

74
Question: Capay (repayment of capital grant)

Capay opens a new factory and receive government grant of $15,000 regarding the
capital expenditure of $100,000. Capay has used “net off” method to deal with the
grant.

At the end of 1st year, because of the external environment changes, Capay doesn't
fulfill conditions set by government regarding job opportunities and so Capay has to
repay $15,000 the grant to govnerment.

The depreciation charged in the current year is $5,000.

Required:
Show the double entry about the above transaction supposing Capay has
used the net off method.

Answer:

DR PP&E $15,000-$5,000= $10,000


DR P/L loss $5,000
CR Bank $15,000

75
IAS 23 borrowing costs

When you’re trying to build a building then you may borrow money from the bank and
the bank may charge you interest. Maybe you should expense them in the P/L? or you
can capitalise them as cost to the building and depreciate them over the useful life?

Well IAS 23 borrowing costs specifies that in some circumstances that these interest
expense can be capitalised as cost to the building.

1, it should be a qualifying asset:

the asset takes a substantial period of time to get ready for its intended use or sale.
Example:
*Inventories that require a substantial period of time to bring them to a
saleable condition, e.g., a big ship
*Manufacturing plants
*Power generation facilities
*Investment properties

2, the amount to capitalize?

Borrowing costs – temporary investement income


General funds raised not specific for the asset? (use weighted average
borrowing costXasset value)

3, when to capitalize?

Start capititalisation:
Later of ABB(mncmonic)
A: activity begins (start building)
B:Borrowing costs incurred (take loan)
B: Buy something(buy the land)

Pause to be capitalised
When the activity is disruppted, eg, strike

Ceased to be capitalised
When the asset is intented for use not necessarily actually for use.

4. Disclosure:

Amount of borrowing cost capitalised during the period


Capitalisation rate used

76
The amount to capitalize:

Borol (Temporary investment income)

Borol corrowed $10m at 6.5% to construct a supermarket. But these $10m is not all
needed at once and the unused cash was reinvested in a short term financial
instrument and Borol receives interest from it was $25,000.

Required:
Calculate the amount of borrowing costs to be capitalised.

Answer to Borol:

Borrowing cost ($10m X6.5%) $650,000


 Temporary Invstment Income ($25,000)
Amount to be capitalized $625,000

BOW (general funds raised)


Bow has a $2m 5% loan and a $3m 6% loan. Bow builds a factory costing $500,000
and it takes 15months.

Required:
Calculate the amount of borrowing costs to be capitalised.

Answer to BOW:

Amount to be capitalized: $500,000 X 5.6% interst rate (w) X15/12=$35,000

W: Interest Rate (Average)


Borrowing Cost Total Cost
2,000 5% 100
3,000 6% 180
5,000 280
Interest rate (average) = 280/5,000 =5.6%

77
The period to capitalize:

Boad:
Boad buys a building on 1st DEC 2012 and Boad take a loan from the bank on 2nd Feb
2013 but due to weather conditions the commencement of work has delayed until 1th
April 2013.

On 1th July 2013 to 1th August 2013 there is a strike in the country disrupting the
building process.

The building completes on 1th August 2014 and actually put into use on 8th September
2014.

Required:
Calculate how many months that Boad should capitalize its borrowing cost?

Answer:

Start:
Later of:
Buy a building: 1st DEC2013
Borrow: 1st DEC 2012
Activity begins 1st April 2013

Pause:
1st July 2013- 1st August 2013

Cease:
1st August 2014

So total months= 15months

78
Bocci
Bocci, a supermarket, is planning to construct a new store, and so it issues a $10m
unsecured loan with a coupon (nominal) interest rate of 6% on 1st April 2009. The loan
is redeemable at a premium which means the loan has an effective finance cost of 7.5%
per year.

The loan was specifically issued for this purpose only.

Construction started on 1st May 2009 and it was completed and ready for use on 28th
Feb 2010, but did not open for trading until 1st April 2010.

Because of the recession during the year trading at Bocci’s other stores was below
expectations so the construction of the new store was suspended for 2 months during
July and August 2009.

The proceeds of the loan were temporarily invested for the month of April 2009 and
earned interest of $40,000.

Required:
Calculate the net borrowing cost that should be capitalized as part of the cost of Bocci’s
new store and the finance cost that should be reported in the Income Statement for the
year ended 31st March 2010.

79
Answer:

SFP:

NCA 500,000

CA: Cash 40,000

P/L:
Finance cost 250
Investment Income 40,000

W1: Period to capitalize:


Start:
Later of:
Activity begins 1st May 2009
Borrow: 1st April 2009

Pause:
2months (July & August 2009)

Cease:
28th Feb 2010

W2: Amounts to be capitalized

$10,000 X7.5% X8/12= $500


$10,000 X7.5% X 4/12= $250

80
IAS 24 Related Party Disclosures

The IAS 24 related party disclosures is just to do with disclosure of the related parties
not the accounting treatment of it.

IAS 24 just discloses there is a possibility that results may be affected by buying or
selling things from/to related parties.

81
What?

 Parties are related if one party has control or significant influence over the other
party.

 Related party transactions are transactions between related parties.

B C

Scenarios:

 If A controls (>50%) or joint controls (=50%) B and have significant influence


over C then A&B are related parties, A&C are related parties as well. Also B&C are
related parties because A could have power to force one sub to do something
against another.

 If A have significant influence over B&C then A&B, A&C are related parties but B&C
are not related parties because A can’t control over B or C to do something.

 If a person has significant influence or control over A then this person &A are
related parties. (particularly if this person is a member of the key management
team in A or close family)

 IAS 24 states there are particularly some situations which may be related parties
transactions:
 Associate and subsidiary
 Key management
 Post-employment benefit: pension plan
 Close family

 So what should we disclose under IAS 24 related party disclosures?

 Transaction: purchase/sale of goods?


 Parties: X Company; Y Company.
 Relationship: eg, parent and subsidiary
 Value: $
 Date

82
Q Engina

Engina, a foreign company, has approached a partner in your firm to assist in obtaining
a local Stock Exchange listing for the company. Engina is registered in a country where
transactions between related parties are considered to be normal but where such
transactions are not disclosed. The directors of Engina are reluctant to disclose the
nature of their related party transactions as they feel that although they are a normal
feature of business in their part of the world, it could cause significant problems
politically and culturally to disclose such transactions.

The partner in your firm has requested a list of all transactions with parties connected
with the company and the directors of Engina have produced the following summary:

(a) Every month, Engina sells $50,000 of goods per month to Mr Satay, the financial
director. The financial director has set up a small retailing business for his son and the
goods are purchased at cost price for Mr Satay.

The annual turnover of Engina is $300 million. Additionally Mr Satay has purchased his
company car from the company for $45,000 (market value $80,000). The director, Mr
Satay, earns a salary of $500,000 a year, and has a personal fortune of many millions
of pounds.

(b) A hotel property had been sold to a brother of Mr Soy, the Managing Director of
Engina, for $4 million (net of selling cost of $0.2 million). The market value of the
property was $4.3 million but in the foreign country, property prices were falling rapidly.
The carrying value of the hotel was $5 million and its value in use was $3.6 million.
There was an oversupply of hotel accommodation due to government subsidies in an
attempt to encourage hotel development and the tourist industry.

(c) Mr Satay owns several companies and the structure of the group is as follows.

Mr Satay

100% ownership 80% ownership


of Car Limited of Wheel Limited

100% ownership
of Engina Limited

Engina earns 60% of its profits from transactions with Car and 40% of its profits from
transactions from Wheel. All the above companies are incorporated in the same
country.

83
Required
1, setting out the reasons why it is important to disclose related party transactions;
(5 marks)
2, Explain the nature of any disclosure required for the above transactions under IAS
24 Related party disclosures.
(15 marks)
(25marks)

84
IAS 34 Interim Financial Reporting

Basic Idea:
 Companies are not required to prepare the interim financial reporting.
 But if you want to do so then you can follow the guidance given by IAS 34.
 Interim financial reporting period is the period that is less than 1 year.
 Many companies do this every six months and then they should issue the financial
statements within 60 days following the guidance.
 The content within the account may include:
SOFP,
P/L,
SOCIE,
SOC,
Explanatory note: related party transactions; litigation settlements; acquisition and
disposals of PPE

Periods cover:
 SOFP:
End of current interim date + end of last financial year date

 P/L, SOCIE, statement of cash flow:


Current interim period + last year interim period+ cumulative figure to date

85
Q Interim Ltd (periods cover)

Interim Ltd chooses to prepare its interim financial reporting because they can show
this to shareholders and let them keep track of the development of the company.
The date to date is 31st July 2014 and the financial statement date is 31 DEC.
Interim Ltd wants to prepare the interim financial report at 30th June 2014.

Required:
Show what periods that these financial statements should cover if it were to do so?

Answer:

SOFP: 30th June2014 and 31 DEC 2013.

P/L, SOCIE, statement of cash flow:


30th June2014 + 30th June2013 + 31st July 2014

86
IAS 36 impairment of assets

Basic Idea:

Impairment means decrease. According to prudence concept we cannot overstate the


asset value. And in order to ensure that amount is prudent we can use a test called
impairment test.

This standard sets out that when the carrying value of an asset in the FS is greater than
its recoverable amount (company can recover money from the asset at the end of its
useful life) then the carrying value is reduced to its recoverable amount. (choose the
lower amount).

Reduction in carrying value is called impairment loss. Defined by IAS 36:


The amount by which the carrying amount of an asset or cash-generating
unit exceeds its recoverable amount.

The question is when do we do the impairment test and how can we do that?

87
When:

1, Indication of impairment at the reporting date


Internal:
-Asset obsolete or damage;
-Operating losses for the current period;
-Loss of key employees;
-Reconstructions.
External:
-Adverse change in the commercial environment(decrease demand for the asset)

2, Test annually for certain assets:


- Intangible assets with an infinite useful life;
-Goodwill acquired in a business combination.

88
How:

1, Steps for individual asset:

Step1: Compare CV with RV*(higher-step2)


Step2: Take the higher of VIU*(future cash flow discounted) and NRV (price-cost to
sell)
Step3: CV-RV=impairment loss

*RV=recoverable amount: how much money can I get if I’m going to sell it (NRV) or
use it (VIU).
*VIU=value in use (using this asset then it will generate into future cash flow)
And in the real practice this is from the most recent budget and a maximum of 5 years.

What can be included in future cash flows? [recent budgets or forecasts]:


 Cash inflow from continuing use the asset.
 Necessary and directly attributable expenses such as overhead expenses; day
to day servicing.
 Net cash flows from disposal from selling the assets or paying for any expenses
etc at the end of the useful life of assets.
 You can include inflation or you don’t need to include inflation and it’s up to you.
But you should make sure you do this consistently.

What can’t be included? [Your estimate process (Extrapolate)]:


 Expenses improving assets further performance but for cash outflow incurred to
maintain assets current conditions are included.
 From future restructuring costs which not committed.
 Any cash flows from financing activities are not considered because they have
been considered in the discount rate stated above.
 Any income tax receipt or paid is not considered because discount rate is
determined on a pretax basis.
 Exclude any financial assets and liabilities such as receivables, payables.

How to determine Discount rate?

Should be pre tax rate and reflect time value of money and risks specific to assets.
You should use market rate or if there is no market rate of interest then use WACC for
example.

89
2, accounting entries:

Asset at historical cost Revalued asset


DR P/L DR revaluation reserve
CR NCA DR P/L(balancing figure with any excess
impairment)
CR NCA

3, Impairment for cash generating units (CGU)


What CGU actually means is that we cannot estimate how much cash flow that a
component can generate but if we were to put all these components all together then
they will generate into cash flows.

The steps to measure impairment in CGU:


Step1: Specific assets
Step2: Goodwill
Step3: Remaining
Note: we can’t allocate impairment expenses to monetary assets such as cash,
receivable and payables.

4, impairment reversal

Step1: see whether impairment indicators are reversed? Ie, reverse internal and
external impairment indicators.

Step2: reverse the impairment losses.

Situation1: [revaluation and then impairment]


If you have revalued an asset and after that an impairment happens and you can
reverse those revaluation reserve in the first place. (see example IMA ltd below
second(b))

Situation2: [impairment and then revaluation]


The maximum of expenses recovered to statement of profit or loss is where you take
asset value without impairment and continue to charge depreciation expenses and
then compare with depreciated asset value charging impairment losses and this is the
amount you can recovered to statement of profit or loss. (See example IMA ltd below
second(c))

90
Q Cable &Gable
$000 $000
Cable Gable
Carrying value 400 500

Value in Use(VIU) 170 540


Net realisable value(NRV) 230 20

Required:
Show the impairment expense for Cable and Gable.

Answer:
Cable Gable
CV 400 500
Impairment (170) -
RA 230 540

91
Q IMA Ltd [impairment reversal]
On 31 DEC2011 the value in use of machine in IMA Ltd is $290m while the net
realisable value for it is $110m. Its carrying value at that time is $300m.

On 31 Jan 2012 the building of IMA Ltd with a NBV of $57m has been revalued to $60m.
And at the year end the recoverable amount of the building is $55m.

On 1st june2011 asset value was $90m and the useful life is 30 years. Impairment
happens for the year ended May2012 for this asset and its recoverable amout is down
to $75m. On 31st may 2013 the asset is revalued up to $105m.

Required:
(a)Show the doubel entry for the machine of IMA Ltd. (single asset impairment)

(b)show the double entry for the building of IMA Ltd. (impairment reversal)

(c)show the double entry for the building of IMA Ltd. (impairment reversal)

92
Answer:

(a):
CV-RV(higher of VIU 290 and NRV110)=300-290=10
DR P/L-impairment expense 10
CR NCA 10

(b):
Revaluation: DR NCA 3 CR RR 3
Impariment:
1, DR RR 3 CR NCA 3
2, DR P/L-impairment loss 2 CR NCA 2

(c)
1st June 2011 PPE 90
Depreciation (2012) 90 (3)
30
CV @ 31st may 2012 87
Impairment losses(balancing) (12)
Revalued amount @31st May 2012 75
Depreciation(75/29yrs)(2013) (2.58)
CV @31st may 2013 72.42
Revaluation gain 32.58
Revalued at 31st may 2013 105

Way1:
CV @ 31st may 2012 87 CV @ 31st may 2012 87
Impairment losses(balancing) (12)
75
Depreciation(75/29yrs)(2013) (2.58) Depreciation(87/29) (3)
CV @31st may 2013 72.42 Maximum value to 84
be reversed

So maximum P/L reversal is 84-72.42=11.58

So:
DR PPE 32.58
CR P/L 11.58
CR RR(bal)21

Way2:
DR PPE 32.58
CR P/L 11.58 (reversal of impairment losses 12 – adj for depreciation (3-2.58))
CR RR(bal)21

93
Q Farma –CGU impairment
Farma acquired a truck business on 1 January 2014 for $230,000. The values of the
assets of the business at that date based on book values were as follows:

$(‘000)
Garage 20
Vehicles(9 vehicles) 90
Computers 10
Intangible assets 30
Trade receivables 10
cash 50
Trade payables (20)
190

On 1 February 2014 a rival truck company commenced business in the same area. It is
anticipated that the business revenue of Farma will be reduced leading to a decline in
the present value in use of the business, which is calculated at $140,000. It is unlikely
the business could be sold as a going concern.

On 1 February 2014, the truck company had three of its vehicles stolen. The net selling
value and net book value of each vehicle was $10,000. Because of non-disclosure of
certain risks to the insurance company, the vehicles were uninsured.

The net selling value of the truck licence has fallen to $25,000 as a result of the rival
truck operator.

Required:
Calculate the carrying value of assets of Farma after considering impairment losses.

94
Answer:

Before Impairment After impairment


impairment expense
Garage 20 (10) 10
Vehicles(9 vehicles) 90 (30) 60
Computers 10 (5) 5
Intangible assets 30 (5) 25
Trade receivables 10 - 10
cash 50 - 50
Trade payables (20) - (20)
Goodwill 40 (40) 0
230 140

Step1: Specific Assets:


Vehicles:3 X $10,000=$ 30,000
Intangible asset: 5

Step2: Goodwill: (remaining 90-30-5-40=$15)

Step3: Pro rata to the remaining assets excluding Cash, Payable and Receivable:

Remaining expense: $15 : Garage: 20/30 = $10


Computer:10/30 = $5

95
IAS 37 provisions, contingent liabilities and contingent assets

1, Provision:

A provision is an uncertain future obligation that the business may or may not have to
settle.

You can only recognize the provision if these 3 criteria are met: (mnemonics: POR)

P: probable that resources will be transferred to settle the liability (asset/other


resources);
O: present obligation whether it’s legal (law) or constructive (published information)
from past event;
R: reliable estimate of the amount of payment can be made.

Double entry:
DR Relevant expense a/c (Statement of profit or loss and other comprehensive
income)
CR Provision (Statement of financial position)

Disclosure:
To show how the opening provision may be reconciled to the closing provision

Opening provision $55m


Provision $20m
Closing provision $75m

96
2, Contingent liabilities

A contingent liability exists when:

Situation1:
A possible obligation that arises from past events and existence will only be confirmed
by the occurrence or non-occurrence of one or more uncertain future events not wholly
within the control of the entity.

Situation2:
A present obligation that arises from past events but it fails criteria P and R (above) of
a provision.

Disclosure:
1, nature of contingent liability
2, likely financial effect
3, uncertainty of the amount and timing

3, Contingent assets

A contingent asset arises from probable future income.

Situation:
It is a probable/possible asset that arises from past events whose existence in
confirmed by the occurrence or non-occurrence of uncertain future events not wholly
within the control of the entity.

If it becomes virtually certain(>95%) that the company can receive the asset rather
than just a contingent asset so that they can recognize the asset in the financial
statements rather than disclose it.

Disclosure: (when it’s probable)


1, nature of contingent asset
2, likely financial effect

97
To sum up:

Liability (outflow) Asset


(inflow)

Probable(>50%) Provide (provision) Disclose


Possible(20%-50%) Disclose Ignore
Remote(<20%) Ignore Ignore

[Background:]

Prior to IAS 37, the management of business can manipulate its financial statements
through providing for this provision.

So for example in the year that company makes a profit of $55m but the market
estimate of this is to be $25m so the management may try to provide for a future
intention to invest its money into the project which costs them money of $29m so they
increase the expense and liability for this year of $29m and its profit may become
$26m which is above the market expectation. In next year the management may find
the company did a bad job of only having profit of $5m so the management may try to
reverse the $29m spent originally (which is not spent at all) so that the profit may
increase by another $29m to become $34m which is well above the market expectation
of $29m so company is doing a good job again.

Provision for restructuring:

P: probable that resources will be transferred to settle the liability (asset/other


resources);

O: present constructive obligation (published information) from past event;


 Detailed formal plan
 This plan should identify business is being restructured; locations affected;
staff compensated etc.
 Valid expectation
 It should create valid expectation that company either starts to restructure
or announce its plans.

R: reliable estimate of the amount of payment can be made.


 It should not include the following:
 Retraining or relocating existing staff
 marketing expense

98
 Investment in new systems and distribution network.
Example: [Bosco Ltd]

1, Bosco Ltd sells high fashion clothing and accessories and has an established policy of
allowing customers to return goods if customer finds the goods unsuitable or they have
a change of mind. Bosco Ltd has no legal obligation to do this. The directors have
estimated that sales returns next year would be $250,000 based on past trading
experience.

2, Bosco Ltd is also being taken to court by a customer for injury due to one of the
products Bosco Ltd sells. The customer is claiming damages of $75,000. Bosco Ltd.’s
lawyers said that there is a 35% chance that the company would have to settle the
claim as the court case is in its final stages.

3, The directors of Bosco Ltd would like to refurbish some of its supermarkets in the
next accounting period. They estimate that this would cost $500,000 and would like to
provide for this amount in their yearend financial statements.

4, MJY Ltd signed a contract with Bosco Ltd to do the refurbishment work for MJY ltd.
Bosco Ltd sub-contracted a company to provide part of refurbishment of the retail
stores of MJY Ltd. But after the refurbishment service has been provided that MJY Ltd
found there’re certain areas the sub-contractor did not meet the standard. So MJY Ltd
decides to sue Bosco Ltd for damages of $45,000. Then Bosco Ltd counter sue MJY Ltd
for this issue. And from the lawyer of Bosco that there’s a 80% of chance that the
sub-contractor will have to pay $35,000 regarding this issue to Bosco Ltd. And the
lawyer also estimates that there’s a 60% of chance that Bosco Ltd will have to pay
$45,000 to MJY Ltd as well.

, Bosco Ltd owns 28% shares of another company Musico Ltd so Bosco Ltd is deemed
to have significant influence over the financial and operational policy making process of
Musico Ltd. Now Musico Ltd is having financial problem so Bosco Ltd decides to borrow
money from the bank of $500,000 and lend this money to Musico Ltd. Because the
uncertainty about its financial performance within Musico Ltd which means that Musico
Ltd may default on payment so the Bosco Ltd may have to recover the amount to bank.

Required:
Show how Bosco Ltd will deal with the above transactions.
[including your judgment + possible double entry/disclosures]

99
Answer to Bosco:

1. Judgment:

Probable [yes]- based on past experience


With constructive obligation
Costs of $25,000 can be reliably measured

So a provision should be provided for.


Double entry:
DR P/L $250,000
CR Provision $25,000

2. Judgment:

Not probable because it’s only 35% change


With legal obligation because this is required by law
The amount is reliably measured because it’s $75,000

So we should disclose this as a contingent liability.


Action:
Disclose the:
1. Nature of the event: Court case
2. Financial Effect: $75,000 cash outflow from Bosco Ltd
3. Uncertainty: Amount is only possible and the timing would be in last stage of the
court case.

3. Judgment:

It’s probable
There is no legal obligation for this because it’s just an intention
The cost can be reliably measured because it’s $500,000

So there would be no actions for this.

100
4. Judgment:

It’s probable because it’s 80% chance of cash outflow


There is legal obligation
The cost can be reliably measured with $35,000

So a contingent asset should be disclosed for Bosco Ltd.


But for sub-contractor, there should be a provision being provided for:
DR P/L CR provision $35,000
The following disclosure should be made by Bosco Ltd as well:
1. Nature of event: Court Case
2. Financial effect: cash inflow of $35,000 will be received by Bosco ltd.

5. Judgment:

Not probable because it’s not known. (Due to future uncertainties)


There is legal obligation
The cost is reliably measured with $500,000

There is no control by Bosco Ltd to Musico Ltd because it’s only having 28% shares
(significant influence)

So a contingent liability should be disclosed.

There should be following disclosures:


1. Nature of event: default on payment by Musico ltd potentially.
2. Financial effect: Recover $500,000 of cash flow.
3. Uncertainty of amount and timing: only possible and for timing-unkown.

101
Restructuring (June2013 Q1 (6))

6. Trailer has announced two major restructuring plans. The first plan is to reduce its
capacity by the closure of some of its smaller factories, which have already been
identified. This will lead to the redundancy of 500 employees, who have all individually
been selected and communicated with. The costs of this plan are $9 million in
redundancy costs, $4 million in retraining costs and $5 million in lease termination
costs. The second plan is to re-organize the finance and information technology
department over a one-year period but
it does not commence for two years. The plan results in 20% of finance staff losing
their jobs during the restructuring. The costs of this plan are $10 million in redundancy
costs, $6 million in retraining costs and $7 million in equipment lease termination costs.
No entries have been made in the financial statements for the above plans.

Answer:

1st plan: closure:


Redundancy costs 9
Lease termination costs 5
14: DR RE 14 CR provision 14

Retraining costs[not direct costs] 4 [no provision]

2nd plan-reorganization:
Can’t provide for future decision.

102
IAS 38 Intangible Assets

Basic idea:

Intangible assets are something you can’t touch, i.e., without physical substance.
Things like Goodwill, Patents, Brands / trademarks, Copyrights and customer lists etc.

You can recognize the intangible assets as a non-current asset in the SOFP if they are
externally generated, ie, you purchase them and they have a fair value for this.

You cannot recognize the intangible assets as a non-current asset in the SOFP if they
are internally generated, ie, you can’t reliably measure their value is because even
though they engage an expert to put a value onto the asset but everybody has different
opinion on the assets as well.
An exception for this is the development costs.*

Initial recognition:

Identifiable: You purchase it and you’ve got a contract.


Asset definition: Control by company- for human assets which are not controllable by
business.
Also it’s probable that future economic benefit will flow into entity.
Cost: Can be reliably measured.

103
Subsequent measurement:

Amortize it over its useful life using straight line method.

Double entry:
DR Amortization expense (Statement of profit or loss and other comprehensive
income)
CR Accumulated amortization (Statement of financial position)

*Exception [R&D]

Research and development costs (R&D)

Research expenses: this means that you search for the internet and other
information to see whether the plan is workable. So it should be expensed to P/L not
capitalize as asset because it’s not probable that this can help company generate into
future economic benefit.

Development costs: when company sees that the plan is workable then it starts to
invest money into developing the asset, eg, design and test for the product. Then if the
development costs meets the following criteria then it should be capitalized as an
intangible asset.
(Mnemonic: USER: TIM)

The asset can be used or sold


Economic benefit will be probable to flow into entity
Enough resource to complete the process
The process is technically feasible
There’s management intention to complete the process
The costs of this can be measured reliably.

104
Q Tommican

Tommican is a leader in high fashion clothes manufacturing business and it now wants to
diversify its activity. Tommican purchases an incorporated football club towards the end of
their accounting year. In the football club are 30 football players all trained by the club from
school boys. Therefore, the subsidiary attaches no carrying value to the players. However,
the 30 players are worth a combined $25million at acquisition.

Immediately after the subsidiary acquisition the football club purchases a star striker for
$18million.

Tommican starts to use the slogan “JUST DO THAT” and they argue that because of the
belief in their new venture, the slogan has a genuine value of $7million at the year end.

Required:
What is the total amount of intangible assets to be capitalized at purchase?

Q Intan

On 31th DEC2010 Intan has incurred expenditure of $100,000 in investigating a new


process. It is hoped that the new process can eventually be adapted and used to
manufacture product A more efficiently than at present, resulting in considerable cost
savings.

The project is at a very early stage and the outcome is uncertain.

On 31th DEC2011 Intan decides to develop this product firstly by designing the
prototype of it. After designing the prototype of this product which incurs $50,000 then
it is found that this product will not meet the scientific test although management has
confidence that this product can be completed.

Required:
How should the above transactions be treated in the financial statements?

105
IAS 40 investment property

Basic Idea:

If you are going to not invest your money into buying shares but instead investing your
money into some land and buildings to earn capital gain(non-cash item) or let it out to
earn rental income(cash item), well this is investment property.

Classification of investment property:

Investment purpose (earns capital gain or let it out to earn rental income and if not:
Use by company: IAS 16; held for sale: IAS2)
Complete (asset has been completed and if not, IAS 16 until it’s finished)
Empty (the asset is not occupied by the business and if not: IAS16.
Or if lessee leases property from lessor but from a group’s
perspective this is not an investment property)

And they should be recognized at cost initially.

106
Subsequent measurement:

1, Fair value model (widely used)

Get the fair value:


From Price
From Similar asset within the area
From Value from institution for similar assets
From Discount future cash flow

Then any gain/losses should be recognized into the Income statement.


Gain: DR Investment property (NCA)
CR P/L
Loss: DR P/L
CR investment property (NCA)

If the fair value cannot be obtained by the company then it can use cost model
(IAS16)

2, Cost Model (Rarely used)

Recognized the investment property at cost and depreciate over its useful economic
life.

Disclosures:

Fair value model

An entity that adopts this must also disclose a reconciliation of the carrying amount of
the investment property at the beginning and end of the period.

Opening IP value 100


Increase 50
Closing IP value 150

Cost model

These relate mainly to the depreciation method. In addition, an entity which adopts the
cost model must disclose the fair value of the investment property.

107
Transfer

Maybe due to some reasons that company may try to reclassify the PPE into investment
property then any gains/losses should be recognized appropriately as well.

At date of transfer:

1, From PPE to IP
CV>FV: DR IP CR RR
CV<FV: DR P/L CR IP

2, From IP to PPE
Use fair value as the value to asset and depreciate under IAS16

3, From IP to inventory
Use fair value as the value to asset

4, From inventory to IP
Difference goes into P/L.

108
Q Zu

Zu has 2 investment properties: A and B.

Investment property A is held to earn capital gain and investment property B is letting
out to others to earn rental income.

The net book value of A is $1000 and at the year-end a qualified valuer value this
property to be $1,500.

During the year Zu earns $500 from letting out investment property B.

Required:
How to account for investment A and B in Zu in the financial statements?

Answer to Zu:

A:
SFP:
I.P. 1000+500=1500

P/L:
Capital appreciation(gain) 500

B:
P/L:
Rental income 500

109
Q Investeria

Investeria purchases a property for $10m on 1 January 2013 with a view to earning
rentals and for its capital appreciation. The property is expected to have a useful life of
50 years. At 31 December 2013, market conditions indicated that the fair value of the
property has risen to $12m.

At 1 January 2013 the proportion of the property that would be considered to be land
rather than buildings is $1m

Required:
Show how the property would be presented in the financial statements as at 31 December
2013 if Investeria follows the:

(a) Fair value model


(b) Cost model

Answer to Investeria:

FV Model
P/L SFP
Other income $2m Investment property (FV) $12m

Cost Model
P/L SFP
Depreciation $180,000 Investment property
($10m-$1m[land]) X1/50 years ($10m-$180,000=$9,820,000)

110
Q Trans

Trans Ltd uses a head office in area A and because of rising prices in Area A then Trans
Ltd decides to move his office in area B and to reclassify area A into investment
property and letting it out to others to enjoy the rental income. The carrying value of
the office at date of reclassification is $60m and the fair value for this is $65m.

Trans Ltd has another building in area B. Because Trans wants to have more investment
opportunities and found this building can be classified into investment property to earn
capital gain and it decides to do so. The carrying value of the building at date of
reclassification is $65m and the fair value for this is $60m.

Required:
Show the journal entries to record the above transactions.

111
IFRS 1 fist time adoption of international financial reporting standards

Basic Idea:

If you’re going to adopt the IFRS for the first time then you can do that but the question
is there’s lots of difference between the financial statements you prepared under the
OLD local GAAP and the financial statements you are going to prepare under IFRS.

So we would like to restate the balance that is the closing balance in last year’s account
using old LOCAL GAAP which is the opening balance in this year’s account and any
difference of them will be taken into Equity.

So how can we present the financial statements if you were to adopt the IFRS first
time?
Here’s the picture for you:

2004 2005

First time
IFRS Comparative
adoption

Unpublished
Old GAAP
but disclose
the note

112
Note to be disclosed:

OLD GAAP X
Difference1 (SBP) (X)
Difference2 (provision) X
IFRS X

113
IFRS 2 Share Based Payment

Basic idea:

Share based payment really covers a lot of areas.

Senario1: If you are going to purchase something but you are not paying cash but
instead you are paying in shares or share options and you can use IFRS2.

Senario2: If you are going to give some incentive to the management of your company
saying to them if you work for me for the next 10 years then I will give you
shares/share options then you can also use IFRS2 to account for it.

The issue with senario1 is about measurement of the value. Because you are going to
pay in shares/options and if you can establish the fair value of the item you
bought(usually in selling price) then you should use the fair value of the item you
bought otherwise you can use the fair value of the shares.

The issue with senario2 is about recognition and measurement of the expense.
If you think about it that you are trying to give incentive to management by offering
them shares at the end of 5(say) years they have worked for you, the shares you are
going to give to them actually cost you nothing because you’re just giving shares to
them so does the company have to recognize the related expense to the financial
statement?

Well, IFRS2 says because management has worked for the company and the company
is going to give shares usually at a low price to the management but otherwise they
could trade it in the stock market at a higher price so company should recognize an
expense relating to it.

Some companies may also argue that recognizing the share based payment expense
will double hit the EPS because as expenses are recognized and shares are issued then
EPS will be twice lower. But as long as management has provided the service for you
and you earn the revenue then you should recognize the expense and also you are
going to give them shares and of course you have to take them into account into the FS
as well for PRUDENCE concept.

114
Calculation:

The question is how can we measure the expense?

Step1: identify the type of scheme. Pay (settle) in shares or cash?

Step2: follow the formula:


Obligation= number of rights expected to vest X FV X timing ratio

Obligation
The total expense we should recognize at the end of the vesting period.
There may be changes in the expense we recognize each year because of our estimates
and any changes in them would be a change in accounting estimate and this would be
accounted for under IAS8 by just using prospective adjusting method. In order words,
just provide for it.

number of rights expected to vest:


number of people left the company + no of people expected to leave next year

Fair value (FV)


If it’s settled in shares then FV should use the value at grant date because it has been
written into the contract.

If it’s settled in cash which means the company will pay cash to the employees based
on the future share price. So if the share price at the end of the vesting period(the end
that employee has worked for the company)is $50m then CR liability 50m. so the Fair
value here will be the FV of options at the end of each year.

Timing ratio=year end / vesting period

115
Q Fairy Ltd (fair value of items and shares)

Fairy Ltd issued share options on 1 June 2013 to pay for the purchase of inventory A. The
value of the inventory on 1 June 2013 was $15m and this value was unchanged up to the
date of sale. The shares issued have a market value of $7m.

Fairy Ltd also issued share capital on 1 July 2013 to pay for the purchase of inventory B and
the fair value of the inventory B cannot be estimated but the fair value of the shares is to be
$5.5m.

Required:
How to account for inventory A and inventory B for fairy ltd?

116
Q Samu (SBP introduction)

Samu offered a three year share based payment scheme to its directors. The volume
granted was 20m

Year Rights expected to vest Fair value of option


0 17m 20c
1 18m 27c
2 15m 33c
3 16m 29c

Required:
Suppose the share based payment uses:
(i) Options.
(ii) Share appreciation rights.

Show the financial statement effect over the three years.

117
Q (past exam Q rewritten) (equity settled)

Kethy granted 200 share options to each of its 10,000 employees on 1 December 2011. The
shares vest if the employees work for the Group for the next two years.

On 1 December 2011, Kethy estimated that there would be 1,000 eligible employees
leaving in each year up to the vesting date. At 30 November 2012, 600 eligible employees
had left the company. The estimate of the number of employees leaving in the year to 30
November 2013 was 500 at 30 November 2012. The fair value of each share option at the
grant date (1 December 2011) was $10. The share options have not been accounted for in
the financial statements.

Required:
How to account for the shares based payment for the year ended 30 November 2012 and
30 NOV 2013.
(4 marks)

118
Q Star (SARs)

Star, a public limited company has granted 700 share appreciation rights (SARs) to each of
its 400 employees on 1 January 2012. The rights are due to vest on 31 December 2014 with
payment being made on 31 December 2015. During 2012, 50 employees leave, and it is
anticipated that a further 50 employees will leave during the vesting period and during
2013 there were 50 employees left. Fair values of the SARs are as follows:

$
1 January 2012 15
31 December 2012 18
31 December 2013 20

Required:
Show how this transaction will be dealt with in the financial statements for the year ended
31 December 2013.

119
IFRS 5 Non-current Assets Held for Sale and Discontinued Operations

Non-current assets held for sale:

When the non-current asset within your company is about to be sold to the 3rd party
maybe because its falling in value then if some criteria are fulfilled then you can
reclassify this non-current asset into current asset as “NCA HFS and discontinued
operations” under IFRS5.

Classification:

The idea behind the criteria is that you should prove that this sale is probable:
Selling purposes by management
Available for sale under current condition
Locate a buyer actively
Expected to complete within 12 months from the year end
If the above criteria are proved then company can reclassify the non-current asset into
non-current asset held for sale under current asset.

120
Initial measurement:

Write down to net realizable value of the asset if it’s a non-current asset held for sale.
Same idea behind inventory (lower of cost and NRV)-impaired!

Trick: pick up the lower figure

Double entry: (NRV<CV)


DR non-current asset held for sale (current asset) (statement of financial position)
DR P/L(balancing figure) (statement of profit or loss and other
comprehensive income)
CR PPE (statement of financial position)

Double entry: (NRV>CV)


DR non-current asset held for sale (current asset) (statement of financial position)
CR PPE (statement of financial position)

Subsequent measurement:

1, no depreciation or amortization (because we are not consuming the asset any


more-not for continued use but for sale.)
2, further impairment losses
DR P/L
CR non-current asset held for sale

Discontinued operations

A discontinued operation is an operation if it’s closed or sold during the year or held for
sale at the year end.

A discontinued operation should:

1, Dispose of or plan to dispose of a separate major line of business or


geographical area of operations;
(Major line of business: eg, financial service industry; supermarket. geographical area: Canada
division)
2, A subsidiary acquired exclusively with a view to resale.

Note: it should be subject to impairment as well same as above. But the key to
discontinued operations is about “DISCLOSURE”. (to help users predict future
performance based on continuous operations.)

121
Disclosure:

Net cash flow detailing operating, investing and financing activities.

Single line in the statement of profit or loss and other comprehensive income showing
post tax profit or loss on discontinued operation.

Analysis of the profit or loss above in the note detailing how to arrive this figure
showing detailed:
Revenue $1,000
expense $50
Pre-tax profit $950
Income tax
Current tax ($15)
Deferred tax ($25)

Gains/losses on measurement to NRV

Recap:

1, Non-current assets held for sale is a single asset or a group of assets.


2, Discontinued operations can be non-current asset held for sale but this is big
assets!(criteria met)
3, For non-current asset held for sale (can be big or small)
4, Impairment happens within group of assets (cash generating units) should follow
the order of impairment in IAS 36.
5, Single asset impaired in non-current asset held for sale should follow IFRS5(write
down to NRV)

122
Q McDonald

On 1 January 2011 McDonald bought a factory for $200,000. It has an expected useful
life of 10 years and the residual value is 0.

On 31 December 2013, after three years of using the asset, McDonald decided to sell
the factory.

A plan was put in place and instructions given to locate a buyer. The factory is in great
demand so McDonald is confident that the factory will be sold quickly. And the factory
is available for sale.

Its current market value is $130,000. As the factory is of a considerable size


dismantling costs to make it available for sale will be incurred of $1,000.

Required:
1, Should the factory be reclassified as non-current asset held for sale? Please specify
the reasons for this.

2, Show how the asset should be presented in the Statement of Financial Position as at
31 DEC 2013 and in the statement of profit or loss and other comprehensive income for
the year ended 31 DEC 2013.

123
Answer:

1, Selling purpose: yes, plan;


Available for sale: yes;
Locate a buyer: yes;
Expected to complete within 1 year: yes, quickly.

2,
SFP as at 13 12 2013 P/L for the year ended 31 12.13
Current Assets Impairment losses (W2) 11,000
Non current assets held for sale(W:
140,000(W1)-11,000(W2)) 129,000

W1
Cost (1.1.2011) 200,000
-accum depre (200,000/10yrs X3) (60,000)
NBV 140,000
NRV= (130,000-1,000=129,000 choose this)
W2 Impairment losses
DR P/L (140,000-129,000) 11,000
CR non current asset held for sale 11,000

124
Q Impair

The cost of the machine is $0.6m but the net realizable value of it is $0.3m when the
machine is reclassified as non-current asset held for sale.

The cost of the factory is $0.6m but the net realizable value of it is $0.7m when the
factory is reclassified as non-current asset held for sale.

Required:
Show the double entry for the above transactions.

Answer:

1, CV>NRV (NCA HFS-NRV)


DR non-current asset held for sale 0.3(lower)
DR I/S 0.3(impairment loss)
CR PPE 0.6

2, CV<NRV (NCA HFS-CV)


DR non-current asset held for sale 0.6
CR PPE 0.6

125
Q Sunset (Discontinued operations Financial Statements)

Sunset is an entity that operates with 3 divisions, A, B C. During the year ended 31
March 2015, Division B is closed.

Here is the Trial Balance extract of Sunset as at 31 March 2015:

DR CR
Revenue (Division A+C) 2,400
Revenue (Division B) 650
Expense (Division A+C) 1,650
Expense (Division B) 525
Finance costs(All continuing activities) 70
Income tax 225

The income tax charge for the year is made up of a charge of $200,000 on continuing
activities and $25,000 for discontinued activities.

A loss of $50,000 was also incurred on the disposal of assets belonging to division B
and $80,000 was spent on restructuring divisions A+C following the termination of
division B.

Required:
Prepare the P/L for Sunset for the year ended 31 March 2015 per IFRS 5
Discontinued Operations.

126
Answer to Sunset:

Continued operations $
Revenue 2,400
Expenses (1,650)
Profit 750
Restructuring costs (80)
Operating profit 670
Finance cost (70)
Profit before tax 600
Income tax expense (200)
Profit for the year (continued operations) 400

Discontinued operations
Profit for the year (Discontinued Operations) **(note) 50
Profit for the year (Total Operations) 450

**note:

$
Revenue 650
Expenses (525)
Profit from Operations 125
Loss on disposal of assets (50)
Profit before tax 75
Tax expense (25)
Profit for the year 50

PS: Restructuring costs have been treated as a material item because this does not
occur on a regular basis and hence we treat it as a separate item on the P/L rather than
part of operating expenses.

127
IFRS 8 Operating Segments

Basic idea:

The aim of the IFRS8 is to give more information to the users of FS to make their
economic decision.

Think about it in this way, if you have a company which is operating in many industries
such as retail, mineral, financial services & education etc. If there’s a rise in price due
to increase in transportation fees then which industry will be mostly affected?

Well to some extent, the retail industry will be mostly affected and the financial service
and education will be least affected. So when investors try to invest their money into
these industries they want to know these segments (companies in different industries)
are operating effectively so we come to IFRS8.

Another example would be if a company has many subsidiaries all round the world such
as in Asia, America, Canada, Singapore etc. and if you want to invest your money into
these companies say in China and you want to know whether the company operating in
China will be good and maybe you will then take into account of political reasons etc.

128
Reporting:

So IFRS8 here just gives us some guidance of when trying to show the results of
different companies, how to do that?
Firstly, we should decide whether this is an operating segment?
An operating segment would have the following features:
1, It has business activities earning revenue and incurring expenses.
2, The operating results will be reviewed by CEO to make economic decisions.
3, There’s separate financial information for each segments showing assets, liabilities,
revenue, expenses and profits etc.

Secondly, once it fulfills the definition of operating segment then you will need to
decide whether this would be reportable?
An operating segment would be reportable if:
 It’s more than 10%of Revenue, Assets or Profits of all segments;
 If there’s a loss then we need to decide whether the loss is higher than the higher
of 10% of total profits and 10% of total loss and if no then it doesn't fulfill this
criteria.
Only one of the criteria needs to be fulfilled.

Thirdly, once the operating segments are classified but they do not add up to 75% in
total then we need to break the other operating segments down in order to make the
total up to 75%.
If other operating segment doesn't fulfill the definition of operating segment then we
can bring them together if they have similar products/types of customers/distribution
methods or regulatory environment.

Fourthly, we need to decide how to disclose the operating segments.


Revenue, total assets & liabilities, interest income & expense, tax & depreciation
should be disclosed.

Fifthly, some segments would have similar natures and can be combined together. We
need to decide whether those small segments would be combined together. These
segments should have the following similar characteristics: [MR PPC]

M**
Regulatory requirements
Products/services
Production process
Customer base

Sixthly, we need to decide if there are any central incurred costs, ie, head office costs,
how do they be allocated to each segments?
Per IFRS8, this is all up to management’s discretion. Let’s take a look at an example of
this in the later questions.

129
Criticisms:

1. Operating segments disclosures are entirely up to management’s discretions and if


management thinks that this is good for their profits, then they may disclose this.

2. In operating segments, there would be internal revenue generated by each segment


and being disclosed. But in the consolidated financial statements, this would be
eliminated and hence quite a lot of adjustments need to be made in order to show the
consistency of information to users.

130
Q Laughing plc (Disclosure)

Laughing plc is a multinational business which operates in Canada and Singapore. The
results of laughing plc are as follows:

$m
Revenue 5,000
Profit after tax 2,000
Net Assets 3,000

High fashion clothes Education


Revenue 40% 60%
Profit after tax 35% 65%
Net Assets 40% 60%

Canada Singapore
Revenue 70% 30%
Profit after tax 60% 40%
Net Assets 30% 70%

Required:
Prepare a segmental note for laughing plc.

Answer:

Based on industry:
Total High fashion clothes Education
Revenue 5,000 2,000 3,000
Profit after tax 2,000 700 1300
Net Assets 3,000 1,200 1,800

Based on area:
Total Canada Sigapore
Revenue 5,000 3500 1500
Profit after tax 2,000 1200 800
Net Assets 3,000 900 2100

131
Q Norman plc (June2008) (a)

(a) Norman, a public limited company, has three business segments which are currently
reported in its financial statements. Norman is an international hotel group which reports to
management on the basis of region. It does not currently report segmental information
under IFRS 8 Operating segments. The results of the regional segments for the year ended
31 May 2008 are as follows.

Revenue Segment Segment Segment


results Assets Liabilities
Internal External Profit/loss
$m $m $m $m $m
European 200 3 (10) 300 200
South East Asia 300 2 60 800 300
Other regions 500 5 105 2,000 1,400

There were no significant intra-group balances in the segment assets and liabilities. The
hotels are located in capital cities in the various regions, and the company sets individual
performance indicators for each hotel based on its city location.

Required:
Discuss the principles in IFRS 8 Operating segments for the determination of a
company’s reportable operating segments and how these principles would be
applied for Norman plc using the information given above. (11 marks)

132
Answer to Norman plc:

(a)
The aim of the IFRS8 is to show users information in a similar manner to help the
company reviewed by the key decision makers.

Impact on Norman:
The decision maker reviews information on a geographical basis.

The European Region would be a reportable segment because its total revenue ($203m)
are more than 10% of Norman’s internal and external revenue of $1,010m.

Europe doesn't satisfy the rules regarding profit because the losses of $10m are less
than 10% of the higher of total profits ($165m) or total losses($10m).

Because only one criteria needs to be satisfied so the profit condition is irrelevant.

Asia:
Asia satisfies the operating segment threshold on
1, revenue :302/1010
2, results: 60/165
3, assets: 800/3100

Other regions:
Norman may need to break down the other regions into smaller components.

This is because Europe and Asia only contribute 50% (500/1000) of external revenue.

Per IFRS8 reportable segment must contribute 75% of this figure.

So we may need to separate out the results of (eg, Canada, Singapore etc from other
region.)

133
Q Traveler

Traveler has three distinct business segments. The management has calculated the net
assets, turnover and profit before common costs, which are to be allocated to these
segments. However, they are unsure as to how they should allocate certain common costs
and whether they can exercise judgement in the allocation process. They wish to allocate
head office management expenses; pension expense; the cost of managing properties and
interest and related interest bearing assets. They also are uncertain as to whether the
allocation of costs has to be in conformity with the accounting policies used in the financial
statements.

Required:
Advise the management of Traveler on the points raised in the above paragraph.
(7 marks)

134
Q Verge

In its annual financial statements for the year ended 31 March 2013, Verge, a public
limited company, had identified the following operating segments:
(i) Segment 1 local train operations
(ii) Segment 2 inter-city train operations
(iii) Segment 3 railway constructions

The company disclosed two reportable segments. Segments 1 and 2 were aggregated
into a single reportable operating segment. Operating segments 1 and 2 have been
aggregated on the basis of their similar business characteristics, and the nature of their
products and services. In the local train market, it is the local transport authority which
awards the contract and pays Verge for its services. In the local train market, contracts
are awarded following a competitive tender process, and the ticket prices paid by
passengers are set by and paid to the transport authority. In the inter-city train market,
ticket prices are set by Verge and the passengers pay Verge for the service provided.
(5 marks)

Answer:

Reportable segment
An operating segment would be reportable if:
 It’s more than 10%of revenue, profits or assets of all segments;
 If there’s a loss then we need to decide whether the loss is higher than the higher
of total profits and loss and if no then it doesn't fulfill this criteria.
Only one of the criteria needs to be fulfilled.

Aggregation:
Aggregation of one or more operating segments into a single reportable segment is
permitted if it meets certain conditions.

Conditions:
Aggregated operating segment should have:
Similar products or services;
Similar business processes;
Similar types of customers.

Scenario:
Segment 1 and 2 customers are different because contract is awarded by local
transport authority in segment1 whilst contract is awarded in the tender process in
segment2 and also they would have different risks so their business processes are not
similar.

Conclusion:
So sgement1 and 2 can’t be aggregated into 1.

135
Financial Instrument (IAS32; IAS 39; IFRS 9; IFRS 7)

In this session we will be covering financial instrument.

Accounting standards summary:

IAS 32 Financial Instruments: Presentation


This deals with classification and presentation in the Financial Statements

IAS 39: Financial Instruments: Recognition and Measurement


This deals with how financial instruments are measured and when they should be
recognized in the Financial Statements.

IFRS 7: Financial Instruments: Disclosures


This deals with the disclosure of financial instruments in the financial statements.

IFRS 9: Financial Instruments

136
IAS32 How to classify financial instruments

Financial instrument:
Any contract that gives rise to both a financial asset of one entity and a financial liability
or equity instrument of another entity.

A Contract is an agreement between two or more parties with clear economic impact
and parties have to exercise this contract and this is usually enforceable through law.

Financial asset:
This is a contract if a party is holding then it can give benefit to the other.

A Financial Asset is any asset:


 Cash (eg, currency in the safe);
 Equity instruments of another party (Eg, investment in ordinary shares);
 Contractual right to receive cash or financial assets (Eg, Receivables);
 Contractual right to exchange financial instrument under favourable conditions
(Eg, call/put option).

Financial liability:
This is a contract if a party is holding then it will deliver cash to other party or cost us
something when exchanging financial instrument.

Eg,
Contractual obligation to deliver cash or another financial asset, ie, trade payables;
redeemable preference shares.

Contractual obligation to exchange financial instrument under unfavorable conditions,


ie, written option.

Equity instrument
Something not for cash or any other assets but they are settled in shares.
Eg, Shares; irredeemable preference shares.

Contract

Financial Asset OR Financial Liability


Equity Instrument

137
Interest, Dividends, losses and gains:

The accounting treatment of them relating to a financial instrument follows the


treatment of the instrument itself.

Eg, dividends paid regarding preference shares classified as a liability would be


charged as a finance expense through profit or loss.

Dividends paid on shares classified as equity would be reported in the statement of


changes in equity.

Transaction costs of an equity transaction should be accounted for as a deduction from


equity, usually debited to the share premium account.

Offsetting a financial asset and liability:

IAS 32 Financial Instruments: presentation sets guidance that in the following


situations, the assets and liabilities would be offset against each other when:

1. The entity has a legally enforceable right to set off the amounts
2. The entity wans either to settle on a net basis, or to realize the asset and settle the
liability at the same time.

138
Presentation of Financial Instrument:

Non-Current Assets
Property plant & equipment
Financial Assets
At fair value through OCI
At amortized cost

Current Assets
Inventories
Financial Assets
Cash & Cash Equivalents
Loans
Receivables
At fair value through P/L

Non-current liabilities
Financial Liabilities
At Amortized Cost

Current Liabilities
Financial Liabilities
At fair value through P/L

139
Initial Recognition: [IAS 39 Financial Instruments: Recognition and measurement]

Criteria: Become party to contract to the contractual provisions of the instrument.

*Financial Assets Designation:

Business model test NO

(Hold it till maturity?)

FVTPL
yes

Contractual cash flow


characteristic(CCC) test
(Does debt contain principle and NO
interest only? ie, only receive
cash?)
yes

Amortized
Cost

Financial liability test (base on intention)

Intention

Keep Trade

Amortized
FV
Cost

140
Accounting Treatment:

1, Financial Asset
Amortized cost (OIOIC)
1, in arrears (quite common)
Years Opening Interest Outstanding Installment Closing
balance (at effective interest rate) (repayment) balance

DR financial asset DR cash

CR P/L(interest receivable) CR financial asset

2, in advance
Years Opening Installment Outstanding Interest Closing
balance (repayment) (at effective interest rate) balance

DR cash DR financial asset


CR financial asset CR P/L(interest receivable)

FVTPL
Gains/losses goes into P/L.

2, financial liability (not held for trading)


Amortized cost (OIOIC)
1, in arrears (quite common)
Years Opening Interest Outstanding Installment Closing
balance (at effective interest rate) (payment) balance

DR P/L(interest payable) DR financial liability

CR financial liability CR cash

2, in advance
Years Opening Installment Outstanding Interest Closing
balance (repayment) (at effective interest rate) balance

DR financial liability DR P/L(interest payable)


CR cash CR financial liability

141
Measurements:

Situation 1: Company accepts Finance.

Notice: A company issues shares and B company bought it.

If this is the case:

To A Company:

DR Cash [Financial Assets]


CR Share capital
CR Share premium

And also if company A pays transaction cost, then:


DR Share premium
CR Cash.

To B Company:
Since B Company has bought the shares from A company:

DR Financial Asset ( Fair value + Transaction cost if FVTOCI )


Or (Fair value only (if FVTPL)
CR Cash

Subsequent measurement:
Take the gains or losses of fair value directly to the OCI or P/L.

142
Financial Liabilities:

Not for trading Financial Liability:

Initial Measurement (Not for trading Financial Liability):

 Not for trading: (Bank loan): Fair value – Costs

Subsequent measurement: At Amortized Costs

FVTPL Financial Liabilities:

Initial Measurement (FVTPL):


 Fair value AND expense the transaction costs

Subsequent measurement:

 Restate to fair value at each reporting date; gains or losses would go into
statement of profit or loss.

143
Accounting Mismatch [Fair Value Option]

 A debt instrument may be classified as measured at either amortised cost or fair


value depending on whether it meets the criteria above. Even where the criteria
are met at initial recognition, a debt instrument may be classified as measured
at fair value through profit or loss if doing so eliminates or significantly reduces
a measurement or recognition inconsistency (sometimes referred to as an
'accounting mismatch') that would otherwise arise from measuring assets or
liabilities or recognising the gains and losses on them on different bases. An
example of this may be where an entity holds a fixed rate loan receivable that it
hedges with an interest rate swap that swaps the fixed rates for floating rates.
Measuring the loan asset at amortised cost would create a measurement
mismatch, as the interest rate swap would be held at FVTPL. In this case, the
loan receivable could be designated at FVTPL under the fair value option to
reduce the accounting mismatch that arises from measuring the loan at
amortised cost.

 IFRS 9 stipulates that if a liability is designated as being at fair value through


profit or loss, changes in the fair value that are due to changes in the liability's
credit risk must be recognised directly in other comprehensive income rather
than profit or loss. Such changes may not be re-classified to profit or loss in
subsequent years, although a reserves transfer is permitted from other
components of equity to retained earnings, ie, statement of changes in equity
would reflect this rather than the P/L.

 On the other hand, if changes in the fair value attributable to the credit risk of
the liability create or enlarge an accounting mismatch in profit or loss, then all
fair value movements are recognised in profit or loss.

 For example, you can argue that the basis of measurement of the debt and the
investment properties is similar, particularly as regards interest rates. This
argument holds good in respect of the interest, and so the fair value option
would be allowed.

144
Compound financial instrument (IAS 32)

Convertible debt

Initial Measurement:

DR Cash
CR Loan (Fair value of future cash flows)
CR Equity (Balancing Figure)

: PV of future cash flow


Debt element
(use effective interest rate on
comparable bond if there’s no
conversion)
Convertibled
ebt

: value of the future conversion


Equity element As a balancing figure
Or (total value-debt-issue costs
from debt + equity pro-rata)

Subsequent Measurement:

 The equity component is not remeasured.


 The liability component is measured at amortized cost where the effective rate
would be the rate of similar non-convertible liability (ie, the rate used to
discount at the initial recognition stage).

145
Situation2: Company provides Finance.

Financial Assets

Financial Assets Initial Measurement Subsequent


Measurement

Fair value through P/L Fair value (With transaction cost P/L
(If it doesn’t belong to the below being expensed into P/L) (Transaction cost is not
categories) included)

Investment in equity instrument at Fair value + Transaction Costs OCI


fair value through OCI (Transaction cost being capitalized)
(Eg, Ordinary shares not for trading
purposes)

If an equity investment is not held for


trading, an entity can make an
irrevocable election at initial recognition
to measure it at FVTOCI with only
dividend income recognised in profit or
loss.

The amounts recognised in other


comprehensive income (OCI) are not
recycled to profit or loss on disposal of
the investment although they may be
reclassified in equity.

Investment in debt instrument at Fair value + Transaction Costs OCI


fair value through OCI (Transaction cost being capitalized)
(Eg, Hold to collect contractual cash
flows but to sell the financial Assets. Ie,
Debentures that earn interest and are
sold before maturity date).

At Amortized Cost Fair value + Transaction Costs Using Amortized Table


(Eg, Hold to collect contractual Cash (Transaction cost being capitalized) (Transaction cost is
flows. Ie, hold the debentures and earn included when suing
interest until the maturity date.) amortized table)

146
Note:
For shares held for speculation purposes, ie, active and regular purchases and sales of
shares; Derivatives: They all fall into Fair value through profit or loss.

147
Disclosure about financial instrument (IFRS 7)

We are required to disclose (significance and risks)

1, Information about significance of financial instrument;


SOFP:
Financial performance and position for each class financial instrument;
Any reclassification, de-recognition, irrecoverable losses of financial
instrument, breach of loan agreements.

P/L:
Separate disclosures for each class of financial instrument;
If financial instrument is not carried at FVTPL then disclose interest expense
on that;
Disclose any impairment losses.

Other information:
Information about the nature of financial instrument in detail;
Accounting policy of how to treat those financial instrument;
Fair value of financial instrument (IFRS 13): how to determine and its value;
Its cash flow relating to the financial instrument.

2, Information about risks of financial instrument.


Qualitative of risks:
Risk exposure- risks included and what would happen?
Risk management-how to manage the risks?
Changes in risks from prior years-if risk has changed?

Quantitative of risks:
Data about exposure
Credit risk-collateral and the quality of it
Liquidity risk-how to manage this risk? (risk of default on payment of
interest?)
Market risks-market prices change? (fair value changes?)

148
IAS39 Financial instrument impairment

Under IAS 39, impairment of financial instrument applies to financial assets carried at
amortized cost.

What we do is to look for indicators of impairment.

If there’s Objective evidence that event has occurred (not happen in the future);
The impairment expense can be estimated reliably.

Then we should recognize an impairment loss to the P/L.

149
IAS39 Hedge Accounting

Meaning

Hedge means use something1 protecting something2.


Something1 means hedging instrument such as forward, future, SWAPs etc.
Something2 means hedging items such as underlying assets, interest rate, exchange
rate etc.

Why hedge?

To reduce existing or future business activities’ risks.

Hedging conditions: [DEM]

Designate at start: when we acquire the hedging instrument we must state it is relating
to a specific hedged item.

Highly effective (80-125% effectiveness)


[Change in fair value of hedged item/Change in fair value of hedging instrument] or
vice versa.

The effectiveness of hedge can be measured reliably


Types

There are 2 types of hedging.


1, Fair value hedge: can only be used to protect something already in the statement
of financial position, e.g., avoid the risk that there may be rise in prices in inventory.
The accounting for this hedge is to take any gains/losses of the hedging instrument
setting off against the gains/losses of hedging items.

2, Cash flow hedge: to protect something they may exist in the future, e.g., if you
want to buy a machine in the future and you are afraid of rising in prices due to
exchange rate movement then you can enter into a forward contract.

And it can be used to protect something which already exists in the FS.

The accounting for this hedge is that any gains and losses of hedging instrument will be
taken to other comprehensive income (OCI) and when the hedging item actually exists
in the financial statement then take this gain/losses to set off against it.

150
Summary in a table:

Types What Treatment

Fair value hedge Hedge item at FV Offset P/L on each in P/L


(Bet on fair value would fall) Hedging instrument at FVTPL

Recognize P/L on hedging


Cash flow hedge Hedged item is future cash flow instrument in OCI;
(Bet on prices arise) Hedging instrument at FVTPL
Offset in P/L till cash flow occur
or offset the original hedge item.

151
Q Aaron (financial asset carried at FVTPL)

Aaron purchased 15,000 shares at the market price of $1.62/share on 1st March 2013.
Transaction cost was $1,200. At 30th June 2014 the shares are trading at $1.4/share.
At 30th June 2015 the shares are trading at $1.85/share.

Required:
How should this be accounted for in Aaron’s account?

Answer:

Initial measurement:
DR Financial Asset [15,000shares X $1.62/share] $24,300
DR Finance Cost $1,200
CR Cash $25,500

At 30th June2014:
DR P/L ($1.4/share X 15,000 shares=$27,600 - $24,300 ) $3,300
CR Financial Asset $3,300

At 30th June 2015:


DR Financial Asset ($1.85/share X 15,000 shares -$27,600) $6,750
CR P/L $6,750

152
Q Peter (financial asset carried at amortized cost)

Peter invested $10,000, the nominal value, in 5% loan notes on 1st July 2014. Peter
had to pay transaction costs of $500. The loan notes will be redeemed in 3 years atime
at a premium of $1,255. The effective interest rate is 7%.

Required:
How to do the initial and subsequent measurement for the above shares for
Peter?

Answer to Peter:

Initial Measurement:
DR Financial Asset ($10,000 +$500) $10,500
CR Cash $10,500

Subsequent Measurement:
Years Opening Interest@7% Outstanding Installment Closing
5%X10,000
1st July 2014-30th June2015 $10,500 735 11,235 (500) 10,735
30th June2015-30th June 2016 10,735 751 11,486 (500) 10,986
30th June 2016-30th June 2017 10,986 769 11,755 (500+1255+10,000) 0

153
Q Morgan (FVTOCI)

Morgan invested in 20,000 shares of a listed company not for trading purpose. The
shares cost $1.7 each at 1 March 2015. Transaction costs of $2,000 were incurred. At
30th June 2014 their market price was $2.1/share. At 30th June2015 their market
price was $1.6/share.

Required:
How to do the initial and subsequent measurement for the above shares for
Morgan?

Answer:

Initial Recognition: Irrecoverable election to measure financial asset as FVTOCI.

Initial Measurement:
DR Financial Asset ($1.7/share X20,000shares) $36,000
CR Cash $36,000

Subsequent Measurement: [20th June 2014]


DR Financial Asset ($2.1/share X20,000 shares=$42,000- $36,000) $6,000
CR FVTOCI $6,000

Subsequent Measurement: [20th June 2015]

DR FVTOCI($1.6/share X 20,000shares=$32,000 -$42,000) $10,000


CR Financial Asset $10,000

154
Q Amy (financial liability carried at amortized cost)

Amy issues a $30,000 3 year 7% redeemable bond at a discount of 10% with issue
costs of $1,000.

The bond is redeemable at a premium of $1,297.

The effective interest rate is 14%.

Required:
Show the treatment for the bond over the 3 year period.

Answer to Amy:

Initial Recognition: Financial liability holding to maturity (Amortised Cost)

Initial Measurement:

DR Cash $26,000
CR Financial Liability ($30,000-$3,000-$1,000) $26,000

Subsequent Measurement:
Years Opening Interest@14% Outstanding Installment Closing
(30,000X7%)
1 26,000 3,640 29,640 (2,100) 27,540
2 27,540 3,856 31,396 (2,100) 29,296
3 29,296 4,101 33,397 (2,100) 31,297

155
Q Tom (convertible bond)

Tom issued one million convertible bonds on 1 June 2011. The bonds had a term of
three years and were issued with a total fair value of $100 million which is also the par
value.

Interest is paid annually in arrears at a rate of 6% per annum and bonds, without the
conversion option, attracted an interest rate of 9% per annum on 1 June 2011. The
company incurred issue costs of $1 million. If the investor did not convert to shares
they would have been redeemed at par. At maturity all of the bonds were converted
into 25 million ordinary shares of $1 of Tom. No bonds could be converted before that
date.

The directors are uncertain how the bonds should have been accounted for up to the
date of the conversion on 31 May 2014 and have been told that the impact of the issue
costs is to increase the effective interest rate to 9·38%.

Required:
Show the initial measurement and subsequent measurement for the convertible bond.

156
Answer to Tom:

Split the bond between Debt & Equity Element.


The debt is calculated on the issue date of 1/6/11 as cash flows (Assuming repayment
is made) discounted by the effective interest rate on a similar non-convertible bond.

Fair value 100


Debt
Equity

1st Year interest $5.5m


6% X $100m
(1+9%)

2nd Year interest $5.05m


6% X $100m
(1+9%)^2

3rd Year interest $81.85m


(6% X $100m +$100m)
(1+9%)^3
$92.4m
Equity Element(Bal) $7.6m
Total Fair Value = $100m

DR Cash $100m
CR Equity $7.6m
CR Liability $92.4m

The issue cost should be deducted from the proceeds of the issue on a pro-rata basis.

CR Cash $1m

DR Debt (92.4/100 X$1m) $0.924m

DR Equity (7.6/100 X$1m) $0.076m

The interest is charged on the net debt figure of ($92.4m-$0.924m)


=$91.476m on the effective rate of 9.38% on the amortised cost approach.

Subsequent measurement (Amortised Cost Table)

157
Years Opening Interest Outstanding Installment Closing
(9.38%) (6%X100)
1/11/12 91.476 8.58 100.056 (6) 94.056
1/11/13 94.056 8.822 102.878 (6) 96.978
1/11/14 96.879 9.021 105.9 (6) 100

Upon Conversion:

The debt and equity figures are converted into shares:

DR Debt $100m

DR Equity (7.6-0.076) $7.524m

CR Share capital ($1 X 25m) $25m

CR [Bal] Share premium $82.524m

158
Financial instrument de-recognition

This means to remove the financial asset or liability from the statement of financial
position.

Financial asset de-recognition:

1. No rights:
This means contractual rights have been expired.

I.e., we have paid for $20,000 and we have a right to purchase gold on 3rd march 2014.
And on 6th march 2014 the option has expired and hence we remove $20,000 of
financial asset from SFP.

2. No R&R (risks and rewards)


This means risks and rewards of the financial assets have been transferred.

For example, for FVTOCI and FVTPL, investments derecognition is when the investment
is sold.

Eg,

Proceeds- Carrying Value= Gains/losses

DR Cash
CR Financial Asset
DR/CR Finance Cost / Investment Income

Eg,
For FVTOCI investments, the gains/losses that have previously been recognized in the
OCI would be transferred to retained earnings in equity.

Because the financial asset is being removed from the SFP so should its associated
reserve in equity be removed. This is just to be a reclassification in equity so there is no
change in the net assets of the entity and hence no gains or losses (This is not shown
in the P/L but it would only be shown as a movement in the statement of changes in
equity) and the double entry would be:

DR FVTOCI
CR Retained Earnings

159
Eg, Receivable:

I.e., we have receivable balance of $20 on 3rd march 2014 and on 6th march 2014 we
have received it and hence:
DR cash
CR receivable (de-recognition)

If the receivable risks and rewards have not been transferred then we should:
DR cash
CR loan

Financial liability de-recognition:

No obligation:
This means that the entity has settled their contractual obligations to deliver cash and
in other words, when they pay the loan back.

I.e., we have borrowed $10m from the bank and we have repaid the loan.
So:

DR financial liability
CR cash

160
Q FVTOCI Derecognition

The carrying value of FVTOCI investment(Total) is $35,400. During the year an


investment which had originally cost $3,000 was sold for cash proceeds of $8,000.

Its fair value as at 30th June 2014 was $6,500.

The fair value of the remaining investments at 30th June 2015 was measured at
$36,700.

Required:
Show the accounting treatment.

Answer:

Proceeds – Carrying Value = Gains/Losses


$8,000 - $6,500 = $1,500

DR Cash $8,000
CR Financial Asset(Fair value @2014) $6,500
CR Investment Income $1,500

The OCI recognized ( FV @ 2014- Cost: $6,500- $3,000=$3,500) should be


transferred to retained earnings:

DR FVTOCI Reserve $3,500


CR Retained earnings $3,500

The remaining investments balance should be updated to reflect their fair value at the
reporting date of $36,700 and any gain/loss recognized:

Total Financial Asset $35,400


Disposal (FV@2014) $(6,500)
Remaining assets at FV @ 30th June 2014 $28,900
Gain $7,800
FV @ 30th June 2015 $36,700

Double entry: DR Financial Asset (FVTOCI) CR FVTOCI gain $7,800

161
Q Thomas (focus on de-recognition of financial assets)

Thomas held 3% holding of the shares in Smart, a public limited company. The investment
was classified as strategic equity and at 31 May 2011 was fair valued at $5 million. The
cumulative gain recognized in equity relating to the strategic equity was $400,000.

On the same day, the whole of the share capital of Smart was acquired by Given, a public
limited company, and as a result, Thomas received shares in Given with a fair value of $5·5
million in exchange for its holding in Smart.

Required:
Show the treatment for the transaction in the accounts to the 31 May 2011:
If the asset was classified as FVTOCI under IFRS 9

Answer:

 At 31st May 2011 we should derecognize the investment in Smart because we no


longer have any rights in the shares.

 Per IFRS9 the sale proceeds are the sum received of $5.5m.

 So a profit of $0.5m ($5.5m-$5m) arises on disposal on 31st May 2011.

DR shares in Given $5.5m


CR shares in Smart $5m
CR OCI $0.5m

 Total gain to OCI is $0.5m+$0.4m=$0.9m and this is transferred via reserve to


retained earnings:

DR OCI 0.9
CR RE 0.9

162
Q Tommy Financial instrument impairment (IAS 39)

(a) Tommy loaned $200,000 to Bromwich on 1 December 2010. The effective and stated
interest rate for this loan was 8 per cent. Interest is payable by Bromwich at the end of each
year and the loan is repayable on 30 November 2014. At 30 November 2012, the directors
of Tommy have heard that Bromwich is in financial difficulties and is undergoing a financial
reorganization. The directors feel that it is likely that they will only receive $100,000 on 30
November 2014 and no future interest payment. Interest for the year ended 30 November
2012 had been received. The financial year end of Tommy is 30 November 2012.

Required
(i) Outline the requirements of IAS 39 as regards the impairment of financial assets.
(6 marks)
(ii) Explain the accounting treatment under IAS 39 of the loan to Bromwich in the
financial statements of Tommy for the year ended 30 November 2012. (4 marks)

163
Q Balabala Ltd

Here is the extract from the trial balance for Balabala Ltd at 30th June2015:

DR CR
10% Bonds $95,000
Interest Paid $10,000

Balabala Ltd issued $100000 10% bonds at their par value on 1st July 2014 that are
being traded on their local exchange. Transaction costs of $5,000 were incurred which
have been debited to the bonds in the above trial balance.

The bonds have a 15 year term and will be redeemed at par value. But the terms of the
bond allows Balabala Ltd to repurchase them at any time at a price of $105 for each
$100 of nominal value so tht Balabala Ltd can benefit from anticipated falls in the
interest rates.

Market interest rates were 8% at the date of issue and 7% at the reporting date.

Balabala Ltd intends to repurchase them if the interest rates fall to 5%.

Required:
Show the accounting treatments.

164
Answer to Balabala Ltd

Initial measurement:
Should:
DR Finance Cost $5,000
CR Cash $5,000

Have done:
DR Financial liability $5,000
CR Cash $5,000

Correct the error:


DR Finance Cost $5,000
CR Financial Liability $5,000

Fair value at the reporting date:

Years Cash flow Discount Factor @ Present Value


7%
2-15 10,000 8.745 87,450
(remaining 14 years)
15 100,000 0.388 38,880
126,330

So the liability value has increased from $100m000 to $126,330 so a loss has
been incurred: [$126,330-$100,000=$26,330]

DR loss on Financial liability $26,330


CR Financial Liability $26,330

165
Hedge Accounting (IAS 39)

Q Fair Ltd (Fair value hedge)

On 1 August 2013 Fair Ltd bought 200 tons of copper at $1,400 a ton for inventory. Fair
Ltd was concerned about price fluctuations so sold all 200 tons on a futures contract for
delivery on 31 March 2014 at $1,480 a ton.

At 31 December 2013, the reporting date, the market value of copper was $1,500 a ton,
and the futures price for 31 March 2014 delivery was $1,568 a ton.

Required:
1, Show the treatment for the fair value hedge.
2, Show how this financial instrument should be disclosed under IFRS 7.

166
Answer:

1.
Hedged item: copper inventory

Hedged instrument=future contract

Change in FV of hedged item: (200X (1,500-1,400) = 20,000)

Change in FV of hedging instrument: (200 X (1,568-1,480) =17,600)

Effectiveness: 20,000/17,600 = 113.6%

And this is highly effective.

1. Revalue inventories:
DR inventory 20,000
CR P/L 20,000

2. Revalue hedging instrument:


DR P/L 17,600
CR financial liability 17,600

2.
Disclosures:
Significance of financial instrument:
1. FS (SFP&P/L) classification of financial instrument
2. Other information about future contract in detail
3. Accounting policy relating to this
4. Cash flow relating to future contract.

Risks of financial instrument:


Qualitative of risks
Risk exposure
Risk management

Quantitative of risks
Data about the exposure
Changes in future price resulting in cash flow changes.

167
Q Cashy Ltd (cash flow hedge)

Cashy Co signs a contract on 1 November 2011 to purchase an asset on 1 November


2012 for €60,000,000. Cashy reports in US$ and hedges this transaction by entering
into a forward contract to buy €60,000,000 on 1 November 2012 at US$1: €1.5.

Spot and forward exchange rates at the following dates are:

Spot Forward (for delivery on 1.11.X2)


1.11.2011 US$1: €1.45 US$1: €1.5
31.12.2011 US$1: €1.20 US$1: €1.24
1.11.2012 US$1: €1.0 US$1: €1.0 (actual)

Required:
Show the double entries relating to these transactions at 1 November 2011, 31
December 2011 and 1 November 2012.

Answer:

1 November 2011

Hedging instrument (forward contract):


$
at 31.12.2011 (€60,000,000/1.24) 48,387,096

at 1.11.2011 (€60,000,000/1.5) 40,000,000

Gain on contract 8,387,096

Hedged item:

at 31.12.2011 (€60,000,000/1.2) 50,000,000

at 1.11.2011 (€60,000,000/1.45) 41,379,310

Increase in cost 8,620,690

Effectiveness: 8,387,096/8,620,690=97% and hence this is effective: [hedged


accounting can be used]

DR Financial asset (Forward a/c) $8,387,096


CR Equity $8,387,096

168
2. 31 December 2011

Hedging instrument (forward contract):


$
at 31.12.2011 (€60,000,000/1) 60,000,000

at 1.11.2011 (€60,000,000/1.24) 48,387,096

Gain on contract 11,612,904

Hedged item:

at 31.12.2011 (€60,000,000/1) 60,000,000

at 1.11.2011 (€60,000,000/1.2) 50,000,000

Increase in cost 10,000,000

Effectiveness: 11,612,904/10,000,000=116% and hence this is effective: [hedged


accounting can be used]

Because hedging instrument changes > hedged item changes and hence:

DR Financial asset (Forward a/c) $11,612,904


CR Equity $10,000,000
CR Profit or loss $1,612,904

3. 1 November 2012

Purchase of asset at market price


DR Asset (€60,000,000/1.0) $60,000,000
CR Cash $60,000,000

Settlement of forward contract


DR Cash $20,000,000
CR Financial asset (Forward a/c) $20,000,000

Realization of gain on hedging instrument


The cumulative gain of $18,387,096 recognized in equity:
 Is transferred to profit or loss as the asset is used, i.e. over the asset's useful life;
or
 Adjusts the initial cost of the asset (reducing future depreciation).

169
IFRS 10 Consolidated Financial Statements

Background:

This is a result from the convergence of the FASB in USA and IASB.

IFRS 10 replaces the definition of control in IAS27 but the preparation of separate
financial statements in IAS27 remains unchanged and consolidation process does not
change per IFRS3.

The definition of control per IAS27: the power to govern financial and operational
policies of an entity to obtain benefit.

Definition:

But the definition of control under IFRS10:


When investor is exposed or has rights to variable returns(profit/loss) from its
involvement with the investee and has the ability to affect those returns through its
power over the investee.

170
3 steps

There are 3 steps to determine control and we use a mnemonic called “PAR”.

P: Power instrument:
Voting rights and potential voting rights; power to appoint directors on the board.

A: activities (relevant).
I like to think about relevant activities which are based on the purpose of the
organization. Such as if your company is going to manufacture high fashion clothes
then a relevant activity would be to determine the selling price of the high fashion
clothes in the whole market etc. An participation in preparation of accounts for the
company is just an irrelevant activity.

R: returns (either positive or negative)


The returns here could be positive or negative which means through the direction of
the activity within the company then the investor may be exposed to or have right to
profit or loss not necessarily benefit (profit.)

Also the returns are variable which means that if the company is doing a good job so it
earns a larger amount of profit then it will distribute larger amount of dividend to
shareholders and this is called variable which is against “fixed”.
Eg, a preference shareholder is just exposed to the fixed dividend received from the
company so it does not necessarily have control over the entity.

171
IFRS 11 Joint Arrangements

This is based on the concept of joint control.

Joint control just prevents any party who control the whole business.

Before setting up the joint control you need to have a look at whether the party has
control per IFRS10.

If yes then move on to see their agreements to establish whether there would be an
unanimous consent over the relevant activities by parties sharing control.

And if yes then you should decide whether this is a joint operation(JO) or joint
venture(JV). These are the two types of joint arrangements.

JOINT OPRATION (JO)

This means that parties do business together using their own assets and settling their
liabilities. The accounting for this follows the substance over form which means the
assets and liabilities remain in each parties’ FS and just a sharing of revenue, expense,
assets and liabilities.

JOINT VENTURE (JV)

Parties may set up a business together and put their assets and liabilities in then the
assets and liabilities belong to the business rather than belong to their own. The
accounting for this is to use equity accounting which means the growth of business
goes into the income statement and SOFP where it’s added to the cost.

172
IFRS 12 disclosures of interests in other entities

This is a new IFRS on disclosure of group relationships that requires the ultimate
parent to disclose all its relationship with other entities.

The parent company is required by IRS12 to list:


All of its subsidiaries and state the reason why it has control not significant influence.
All of its associates and state the reasons why it has significant influence not control.
All of its joint arrangements and state the reasons why it has joint controls.

173
Q Wallet (IFRS10, 11, 12)

Wallet is considering partnering with two other businesses, Danny and Louis. Wallet is a
high tech business and a large multinational group. Danny is a large manufacturer. Louis is
a small manufacturer in Ireland. The proposal is to build high tech manufacturing factory in
Ireland to produce large volumes of pad components for worldwide sales. Wallet will bring
machines and knowledge. Danny will build the factory. Louis will bring local knowledge, eg,
cultures and relationship with local government. There are four proposals being considered.
But in all cases each of the partners will have the right to one third of the profits in the new
business. The four proposals are:-

Proposal one
The new entity will be incorporated.
Each partner will hold one third of the shares.
But only Wallet shares will have voting rights. The Danny and Louis shares will have equal
ownership to the Wallet shares but will have no voting power.
(5 marks)
Proposal two
The new entity will be incorporated.
Each partner will hold one third of the shares.
All shares will have equal voting rights and equal ownership but all decisions must be made
by unanimous vote.
(5 marks)
Proposal three
The new entity will be incorporated.
Each partner will hold one third of the shares.
All shares will have equal voting rights and equal ownership and all decisions must be made
by majority vote.
(5 marks)
Proposal four
The new entity will not be incorporated.
Each partner will have the right to one third of the profit.
Assets are still belonging to each parity.
All the partners will have equal voting rights and equal ownership but all decisions must be
made by unanimous vote.
(5 marks)
Disclosures
The directors of Wallet would also like to know how the above would be disclosed regarding
the above 4 proposals.
(4 marks)
There’s 1 mark for the quality of your answer.
Required
Discuss the proposals, disclosure above and explain how each proposal would be dealt with
by Wallet group.
(25 marks)

174
Q Robby [June2012 Q1 (iii)]

(iii) Robby has a 40% share of a joint operation, a natural gas station. Assets, liabilities,
revenue and costs are apportioned on the basis of shareholding.

The following information relates to the joint arrangement activities:


– The natural gas station cost $15 million to construct and was completed on 1 June
2011 and is to be dismantled at the end of its life of 10 years. The present value of this
dismantling cost to the joint arrangement at 1 June 2011, using a discount rate of 5%,
was $2 million.

– In the year, gas with a direct cost of $16 million was sold for $20 million. Additionally,
the joint arrangement incurred operating costs of $0·5 million during the year.
The revenue and costs are receivable and payable by the other joint operator who
settles amounts outstanding with Robby after the year end.

Robby has only contributed and accounted for its share of the construction cost, paying
$6 million [PPE value on 1 june2011].

Required:
How to deal with joint operation in the consolidated statement of financial position?

Answer:

All of the followings are added to the CSFP:

PPE:
On 1 june2011= $6m
Dismantling costs ($2m X40%) = 0.8
Depreciation expense($6.8m/10yrs) (0.68)
6.12

Receivables: (20X40%) = 8: DR receivables 8 CR sales 8


Payables: [direct cost + operating cost](16X40%+0.5X40%)=6.6
Provision: dismantle costs of 0.8 + 0.8X5%=0.84

Statement of profit or loss:


Sales (20X40%) 8
Expenses (16X40%+0.5X40%) = 6.6
Depreciation expense($6.8m/10yrs) (0.68)
Unwinding of dismantle costs[0.8X5%] 0.04
Profit (transferred to retained earnings W5) 0.68

175
IFRS 13 Fair Value Measurement

In this IFRS 13 we need to know

1, Definition of fair value:

The price that would be received to sell an asset or paid to transfer a liability in an
orderly transaction between market participants at the measurement date.

Here, the price would be the exit price.

2, How to measure fair value

-For financial assets & non-financial assets, liability and equity instrument,
we use this hierarchy.

Level1: quoted price


If there is an active market then the market price from that market on the
measurement date should be used.
Level2: similar quoted price
If level one fails then level two requires that similar market data should
be used to establish the approximated market value.

Level3: unobservable inputs (management best estimate, eg, present value)


If level one and two fails to determine the fair value then you can use
level three where you can use financial model to determine fair value.

-For non-financial assets, we should use highest and best use value
e.g., to determine the fair value of land you need to consider which way that
the land may generate into a highest value. Either for industrial use or residential use.

176
Scope

The IFRS 13 does not cover IFRS2 and IAS 17.

Also the net realizable value in IAS2 and value in use in IAS36 are not covered by
IFRS13 as well.

177
IAS 41 Agriculture

This applies to:


Biological assets(animals and plants);
Agricultural procedure at the point of harvest;
Government grants related to biological assets.

Examples of agricultural activities:


Raising livestock
Cropping
Forestry
Cultivating orchards and plantations
Floriculture
Aquaculture (fish farming)

178
Biological assets(animals and plants):
Initial and y/e: FV-estimated point-of-sale costs
Ie, FV of cattle at a farm is the price for the cattle in the relevant
market-transport&other costs of getting the cattle to that market.
If a fair value cannot be determined because market determined prices or
values are not available. Then the biological asset can be measured at cost less
accumulated depreciation and impairment losses.

Recognition of biological assets


The recognition criteria are very similar to those for other assets, in that animals or
plants should be recognised as assets in the following circumstances.

The entity controls the asset as a result of past events


It is probable that the future economic benefits associated with the asset will flow to
the entity and this can be determined by formal ownership records, eg land title,
branding.
The fair value or cost of the asset to the entity can be measured reliably

Agricultural produce harvested from biological assets:


FV-costs to sell at the point of harvest.
Cost may sometimes approximate fair value when:
Little biological transformation has taken place since initial cost has incurred:
For fruit tree seedlings planted immediately before the reporting period end;
Gains/losses---recognise in statement of profit or loss.

Presentation and disclosure


In the statement of financial position biological assets should be classified as a separate
class of assets falling under neither current nor non-current classifications. This reflects the
view of such assets as having an unlimited life on a collective basis; it is the total exposure
of the entity to this type of asset that is important.

Biological assets should also be sub-classified (either in the statement of financial position
or as a note to the accounts).
Class of animal or plant
Nature of activities (consumable or bearer)
Maturity or immaturity for intended purpose
Agricultural produce should be classified as inventory in the statement of financial position
and disclosed separately either in the statement of financial position or in the notes.

179
Government grants
An unconditional government grant related to a biological asset measured at its fair
value less estimated point-of-sale costs should be recognised as income when, and
only when, the grant becomes receivable, ie, DR cash CR income

If a government grant requires an entity not to engage in specified agricultural


activity (eg the EU's set aside grant), an entity should only recognise the grant as
income when, and only when, the conditions are met, ie, DR cash CR income

IAS 20 does not apply to a government grant on biological assets measured at fair
value less estimated point-of-sale costs, ie, no deferred income method allowed.

However if a biological asset is measured at cost less accumulated depreciation and


accumulated impairment losses then IAS 20 does apply, ie, DR cash CR deferred
income and release deferred income into actual income over the life of biological
assets.

180
Q Luck Dairy(IAS 41)

The Lucky Dairy, a public limited company, produces milk for supply to various customers. It is
responsible for producing twenty five per cent of the country's milk consumption. The company owns
150 farms and has a stock of 70,000 cows and 35,000 heifers which are being raised to produce milk
in the future. The farms produce 2.5 million kilograms of milk per annum and normally hold an
inventory of 50,000 kilograms of milk (E1tracts from the draft accounts to 31 May 2012).

The herds comprise at 31 May 2012:

70,000 – 3 year old cows (all purchased on or before 1 June 2011)


25,000 – heifers (average age 1½ years old – purchased 1 December 2011)
10,000 – heifers (average age 2 years – purchased 1 June 2011)

There were no animals born or sold in the year. The per unit values less estimated point of sale costs
were as follows.

$
2 year old animal at 1 June 2011 50
1 year old animal at 1 December 2011 40
3 year old animal at 31 May 2012 60
1½ year old animal at 31 May 2012 46
2 year old animal at 31 May 2012 55
1 year old animal at 31 May 2012 42

The company has had a difficult year in financial and operating terms. The cows had contracted a
disease at the beginning of the financial year which had been passed on in the food chain to a small
number of consumers. The publicity surrounding this event had caused a drop in the consumption of
milk and as a result the dairy was holding 500,000 kilograms of milk in storage.

The government had stated, on 1 April 2012, that it was prepared to compensate farmers for the drop
in the price and consumption of milk. An official government letter was received on 6 June 2012,
stating that $1.5 million will be paid to Lucky on 1 August 2012. Additionally on 1 May 2012, Lucky had
received a letter from its lawyer saying that legal proceedings had been started against the company
by the persons affected by the disease. The company's lawyers have advised them that they feel that it
is probable that they will be found liable and that the costs involved may reach $2 million. The lawyers,
however, feel that the company may receive additional compensation from a government fund if
certain quality control procedures had been carried out by the company.

However, the lawyers will only state that the compensation payment is 'possible'.

181
The company's activities are controlled in three geographical locations, Dale, Shire and Ham. The only
region affected by the disease was Dale and the government has decided that it is to restrict the milk
production of that region significantly. Lucky estimates that the discounted future cash income from the
present herds of cattle in the region amounts to $1.2 million, taking into account the government
restriction order. Lucky was not sure that the fair value of the cows in the region could be measured
reliably at the date of purchase because of the problems with the diseased cattle. The cows in this
region amounted to 20,000 in number and the heifers 10,000 in number. All of the animals were
purchased on 1 June 2011. Lucky has had an offer of $1 million for all of the animals in the Dale region
(net of point of sale costs) and $2 million for the sale of the farms in the region. However, there was a
minority of directors who opposed the planned sale and it was decided to defer the public
announcement of sale pending the outcome of the possible receipt of the government compensation.
The board had decided that the potential sale plan was highly confidential but a national newspaper
had published an article saying that the sale may occur and that there would be many people who
would lose their employment. The board approved the planned sale of Dale farms on 31 May 2012.
The directors of Lucky have approached your firm for professional advice on the above matters.

Required
Advise the directors on how the biological assets and produce of Lucky should be accounted
for under IAS 41 Agriculture and discuss the implications for the published financial
statements of the above events.

(Candidates should produce a table which shows the changes in value of the cattle stock for the year
to 31 May 2012 due to price change and physical change e1cluding the Dale region, and the value of
the herd of the Dale region as at 31 May 2012. Ignore the effects of ta1ation. Heifers are young female
cows.) (25 marks)

182
Answer to Luck Dairy(IAS 41)

Biological asset
The dairy herd is a biological asset as defined by IAS 41 Agriculture.

It states that a biological asset should be measured at fair value less costs at
estimated point of sale.

If a fair value cannot be determined because market determined prices or values


are not available. Then the biological asset can be measured at cost less
accumulated depreciation and impairment losses.

Gains and losses arising from a change in fair value should be included in profit or
loss for the period.

In this case, fair value is based on market price and point of sale costs are the costs
of transporting the cattle to the market.

IAS 41 encourages companies to analyse the change in fair value between the
movement due to physical changes and the movement due to price changes (see
the table below).

It also encourages companies to provide a quantified description of each group of


biological assets. Therefore the value of the cows and the value of the heifers should
be disclosed separately in the balance sheet.

Valuing the dairy herd for the Dale Region is less straightforward as its fair value
cannot be measured reliably at the date of purchase.

So IAS 41 requires the herd to be valued at cost less any impairment losses.

The standard also requires companies to provide an explanation of why fair value
cannot be measured reliably and the range of estimates within which fair value is
likely to fall.

183
Valuation of cattle stock, excluding Dale region:
Cow Heifers Total
FV @1.6.2011 2,500 1,000 3,500
50,000 X$50(2yrs)
25,000X$40(1yr)

Physical change: 250 100 350


50,000X($60-$55)
25,000X($46-$42)

Price Change 250 50 300


50,000X($55-$50)
25,000X($42-$40)

FV @31.5.2012 3,000 1,150 4,150


50,000X$60(3yrs)
25,000X$46(1.5yr)

Valuation of cattle stock in Dale Region

Cows (20,000 @ 50) 1,000


Heifers (10,000 @40) 400
Cost at 1 June 2011 1,400

- impairment loss (200)


1,200(higher of VIU($1.2m) and NRV($1m))

Milk
The milk is agricultural produce as defined by IAS 41 and should normally be
measured at fair value less estimated point of sale costs at the time of
milking(produce).

Because of the disease, the company is holding 500,000kg milk and it is probable
that much of this milk is unfit for consumption.

The company should estimate the amount of milk that will not be sold and write
down the inventory accordingly.

The write down should be disclosed separately in the income statement as required
by IAS 1 Presentation of financial statements.

184
Government grant
Under IAS 41, the government grant should be recognised as income when it
becomes receivable.

As it was only on 6 June 2012 that the company received official confirmation of the
amount to be paid, the income should not be recognised in the current year.

This is a non-adjusting event per IAS 10 and should be disclosed in the FS.

Legal proceedings and additional compensation


The lawyers have indicated that the company will probably be found liable for
passing on the disease to consumers.

There is a present obligation as the result of a past obligating event and therefore a
provision for $2 million should be recognised, as required by IAS 37 Provisions,
contingent liabilities and contingent assets.

IAS 37 states that reimbursement should only be recognised when it is virtually


certain to be received. It is only possible that the company will receive
compensation for the legal costs and therefore this cannot be recognised.

However, the compensation should be disclosed as a contingent asset in the


financial statements.

Planned sale of Dale farms


The Board of Directors has approved the planned closure, but there has not yet been
a public announcement.

Despite the fact that a local newspaper has published an article on the possible sale,
the company has not created a valid expectation that the sale will take place and in
fact it is not certain that the sale will occur. Therefore there is no 'constructive
obligation' and under IAS 37 no provision should be made for redundancy or any
other costs connected with the planned sale.

Under IFRS 5 Non-current assets held for sale and discontinued operations Dale
must be treated as a continuing operation for the year ended 31 May 2012 as the
sale has not taken place. As management are not yet fully committed to the sale
neither the operation as a whole nor any of the separate assets of Dale can be
classified as 'held for sale'.

185
IFRS for SME’s

IFRS is for the use of capital market which means it’s designed for listed companies.

Small and medium sized entities are companies which are not listed.

So they can follow the accounting standard designed primarily for SME’s.

What is the difference between IFRS and IFRS for SME’s?


-less complexity: lots of treatment in IFRS which are complicated but they’re
simple for SME’s.
-less disclosure: because for most SME’s the owners of business may be the
same as the ones operating the company so no need to
disclose lots of things.
-different treatment (e.g.): IAS38 intangible asset: R&D should be expensed in
SMEs;
Revaluation model under IAS16 & IAS38 is not
allowed;
IAS 33 EPS; 34 interim reporting; IFRS1 (comparable
of transition); IFRS5+8 are not required for SMEs.

Why do we need IFRS for SME’s?

Comparability
For some SME’s their accounting system is so simple so this gives a set of guidance of
how to prepare the FS for SME’s so that users can compare information for 1 SME with
another SME to make their economic decisions.

Relevance
Information would be relevant if users can use this information to make their economic
decisions. e.g., IAS33 is not relevant for SME’s because they are not listed companies
so a removal of the IAS 33 requirement will make information of SME’s FS more
relevant.

Faithful Presentation
If SME’s use the set of accounting standards to prepare for their account then it will
give a more faithful presentation to the users of the FS.

186
Management Commentary

If you have a look at the FS for listed companies there is huge information that is
flowing around especially for non-financial background users they will have no idea of
what is going on with the company.

So IASB issued an IFRS practice statement management commentary in DEC2010 to


give guidance for company to disclose financial statements to users with more vivid
pictures and simple words rather than just boring figures.

So what information is included in the management commentary?

1, Management's view of the entity's performance, position and risks;


2, Supplement and complement information presented in the financial statements;
3, Forward-looking information for future strategy.

The management commentary should make sure that users of FS know:

(i)The nature of the business


(ii)Management's objectives and its strategies for meeting those objectives
(iii)The entity's most significant resources, risks and relationships
(iv)The results of operations and prospects
(v)The critical performance measures and indicators that management uses to
evaluate the entity's performance against stated objectives

Management commentary is not compulsory but it’s quite useful for users of FS
especially they have no financial background.

187
Conceptual and regulatory framework

1, Regulatory framework
Companies prepare financial accounts (statutory accounts) and these are filed at
Companies House. They are available for any interested party to view.
Regulation on the preparation of statutory accounts is governed by two main
areas:
Companies act
International accounting standards

Businesses must comply with the Companies Act, but the international accounting
standards are not legally enforceable. If companies want to be listed on the stock
exchange and have a successful audit then they must comply with the accounting
standards.

Regulation ensures that companies comply in certain areas with-in their financial
statements. This results in good practice and makes statutory accounts more
comparable with other entities. This aids decision making and can lead to the
success of a business.

International accounting standards are extremely important and are issued by the
International Accounting Standards Board (IASB). There are four separate bodies
and the structure is:

1, International Accounting Standards Committee Foundation (IASCF)


This committee is responsible for appointing members in the IASB, SAC and IFRIC.
It is also their responsibility to ensure the three bodies have adequate funding.

2, International Accounting Standards Board (IASB)


The I.A.S.B. is responsible for issuing accounting standards (IAS’s). They are also
called international financial reporting standards (IFRS’s)

3, Standards Advisory Council (SAC)


Their responsibilities include advising the IASB on its priorities and providing the
IASB information about the implications of IAS’s.

3, International Financial Reporting Interpretations Committee (IFRIC)


This body is responsible for providing guidance on any problems that emerge
relating to issued IAS’s.

The members of these bodies are from varying countries and backgrounds, some
are preparers of financial statements and others are users of financial statements.
Accountants are expected to follow IAS.

188
2, Conceptual framework

It covers the following 6 main parts:


*The objectives of financial statements
*The underlying assumptions of financial statements
*Qualitative characteristics of financial statements
*The elements of financial statements
*Recognition of the elements of financial statements
*Measurement of the elements of financial statements

1, objectives of FS

To provide useful information to users

Useful: qualitative characteristics


Information: Financial position
Financial performance
Cash flow
Users: shareholders and investors
Lenders
Employees
Public
Government
Customers
Suppliers

2, The underlying assumptions of FS

*going concern

It assumes they prepare for their financial statements in the foreseeable future,
normally oneyear(going concern).

Non-current assets and liabilities can only be included in financial statements


which are prepared on the going concern basis. This is against the break up basis
which assumes company will not continue into the foreseeable future so no
non-current assets or liabilities should be included.

*accruals

The financial statements are prepared on the basis that transactions are reported
in the period to which they relate, regardless of when cash is received or paid and
this is known as accruals.

189
3, Qualitative characteristics of FS

Qualitative characteristics are:

Fundamental Qualitative characteristics would include RELEVANCE and FAITHFUL


REPRESENTATION.

4 other Enhancing qualitative characteristics: (C)Comparability (d)Verifiability


(e)Timeliness (f)Understandability

(a) Relevance:

Relevant information has predictive value or confirmatory value, or both. It is capable of


making a difference in the decisions made by users.
The relevance of information is affected by its nature and its materiality.

(b) Faithful representation:

Information must be complete, neutral and free from error (replacing 'reliability').

Financial reports represent economic phenomena in words and numbers. To be useful,


financial information must not only represent relevant phenomena but must faithfully
represent the phenomena that it purports to represent.

A complete depiction includes all information necessary for a user to understand the
phenomenon being depicted, including all necessary descriptions and explanations.

A neutral depiction is without bias in the selection or presentation of financial


information. This means that information must not be manipulated in any way in order
to influence the decisions of users.

Free from error means there are no errors or omissions in the description of the
phenomenon and no errors made in the process by which the financial information was
produced. It does not mean that no inaccuracies can arise, particularly where
estimates have to be made.

Substance over form


The IASB says that to do so would be redundant because it is implied in faithful
representation. Faithful representation of a transaction is only possible if it is
accounted for according to its substance and economic reality.

Prudence
Prudence is the concept that specifies, in situations where there is uncertainty,
appropriate caution is exercised in recognising transactions in financial records.

190
(C)Comparability

Comparability is the qualitative characteristic that enables users to identify and


understand similarities in, and differences among, items. Information about a reporting
entity is more useful if it can be compared with similar information about other entities and
with similar information about the same entity for another period or another date.

Consistency, although related to comparability, is not the same. It refers to the use of
the same methods for the same items (ie consistency of treatment) either from period to
period within a reporting entity or in a single period across entities.

The disclosure of accounting policies is particularly important here. Users must be able
to distinguish between different accounting policies in order to be able to make a valid
comparison of similar items in the accounts of different entities.

Comparability is not the same as uniformity. Entities should change accounting policies
if those policies become inappropriate.

Corresponding information for preceding periods should be shown to enable comparison


over time.

(d)Verifiability

Verifiability helps assure users that information faithfully represents the economic
phenomena it purports to represent. It means that different knowledgeable and
independent observers could reach consensus that a particular depiction is a faithful
representation.

It recognises two methods of verification:


 direct; and
 indirect

Direct verification can be achieved by direct confirmation, such as cash counts. Indirect
verification is achieved by verifying the inputs to a model or other technique and
re-calculating the outputs using the same methodology.

(e)Timeliness

Information may become less useful if there is a delay in reporting it. There is a balance
between timeliness and the provision of reliable information.

If information is reported on a timely basis when not all aspects of the transaction are
known, it may not be complete or free from error.

191
Conversely, if every detail of a transaction is known, it may be too late to publish the
information because it has become irrelevant. The overriding consideration is how best to
satisfy the economic decision-making needs of the users.

(f)Understandability

Financial reports are prepared for users who have a reasonable knowledge of business and
economic activities and who review and analyse the information diligently. Some
phenomena are inherently complex and cannot be made easy to understand. Excluding
information on those phenomena might make the information easier to understand, but
without it those reports would be incomplete and therefore misleading. Therefore matters
should not be left out of financial statements simply due to their difficulty as even
well-informed and diligent users may sometimes need the aid of an adviser to understand
information about complex economic phenomena.

4, The elements of FS (definitions)

*Assets
Asset is a resource controlled by the entity as a result of past events and from
which future economic benefits are expected to flow to the entity.

*Liabilities
Liability is an obligation by the entity as a result of past events and from which
future economic benefits are expected to flow out from the entity.

*Equity
Equity is the residual amount found by deducting all liabilities of the entity from all
of the entity’s assets.

*Income
Income is increases in economic benefits during the accounting period in the form
of inflows or enhancements of assets or decreases in liabilities that result in
increases in equity, other than those relating to contributions from equity
participants.

*Expense
Expense is are decreases in economic benefits during the accounting period in the
form of outflows or depletions of assets or incurrences of liabilities that result in
decreases in equity, other than those relating to distributions to equity
participants.

192
5, Recognition of the elements of FS

 Meet Element definition


 Probable cash flow(inflow or outflow)
 Reliably measured Cost or Value

6, measurement of the elements in FS

 Historical cost: Cash price or fair value at acquisition or obligation. Most


commonly used but widely criticised
 Current cost: What would be the cash price today
 Realizable value: What would be the price if it’ssold today
 Present value: Discounted future cashflows
The Framework does not state which of the four should be used

How IAS & IFRS are developed? [Adept]

Step1: Advisory council gives advice on the issues and IASB will consult with this.

Step2: IASB then develops and publishes a discussion paper for public comment.

Step3: After receiving the comment, an Exposure Draft is produced for public
comment together with the IASB review of feedback received from public.

Step4: Publish the final standard.

193
Integrated reporting

The aim is to create transparency of the company and also consistency (applying
integrated reporting to companies from all around the world).

Content:

 Organizational overview
 Governance
 Opportunities and risks
 Strategy and resource allocation
 Business model
 Performance
 Future outlook

194
Chapter 3 Consolidation

Content:
REASONS OF CONSOLIDATION ...................................................................................................................196

HOW CONSOLIDATION WORKS ..................................................................................................................197

CONTROL AND SIGNIFICANT INFLUENCE ....................................................................................................203

COST OF INVESTMENT CALCULATION.........................................................................................................205

CONSOLIDATION ADJUSTMENTS................................................................................................................214

1, INTRA GROUP BALANCES.......................................................................................................................214

2, CASH IN TRANSIT...................................................................................................................................215

3, GOODS IN TRANSIT ...............................................................................................................................215

4, PROVISION FOR UNREALIZED PROFIT (PUP) ...........................................................................................217

5, FAIR VALUE ADJUSTMENTS ....................................................................................................................221

GOODWILL ................................................................................................................................................224

ASSOCIATE (IAS 28 INVESTMENT IN ASSOCIATES).......................................................................................236

BASIC GROUP CONSOLIDATION IN P2 [Q BRAVADO] ..................................................................................239

CHANGES IN OWNERSHIP ..........................................................................................................................243

COMPLEX GROUP ......................................................................................................................................253

IAS 21 THE EFFECTS OF CHANGES IN FOREIGN EXCHANGE RATES ...............................................................260

IAS 7 STATEMENT OF CASH FLOW ..............................................................................................................273

GROUP STATEMENT OF PROFIT OR LOSS AND OTHER COMPREHENSIVE INCOME .......................................285

195
Reasons of consolidation

*Get synergy
-Secure supply of materials
-Diversify product lines
-Expand into new markets

*Remove directors

*Eliminate competition

196
How consolidation works

Consolidated Statement of Financial Position as at XX/XX/XXXX

Consolidation of statement of financial position as at XX/XX/XXXX


Step1: structure
Step2: proforma
Step3: workings

Step1: establish the holding percentage (control or


influence)

Step2: Proforma:

Consolidation of financial statement of Age Ltd as at 31 DEC 2015

$000 $000
Non-current assets
Property, plant & equipment (100% P+S) 100
Goodwill (W) 20
Investment in associate(W) 20
140
Current assets
Inventory (100%P+S) 10
Receivables(100%P+S) 10
Bank & cash (100%+S) 5
25
Total assets 165

Equity and liabilities


Equity
Share capital(parent only) 50
Share premium(parent only) 25
Retained earnings(W) 20
95

Non-controlling interest(W) 10
105

Non-current liabilities (100%P+S)


30
Current liabilities(100%P+S) 35
Total equity and liabilities 165

197
Step3: key workings (5 Workings)

W1: Group Structure (Control or Significant Influence)

W2: Net assets list of Subsidiary


Acquisition date SOFP date
$ $
Share capital X X
Share premium X X
Retained earnings X X
Fair value adjustments X/(X) X/(x)
Provisions for unrealized - (x)
profits(PUP) S sells to P
X1 X2

Used for goodwill calculation

And the difference between X2 and X1 goes to consolidated reserves and


Non-controlling interest (NCI)

W3: Goodwill
$
Investment at cost X
Fair value of NCI at acquisition X
X
Less: fair value of net assets at acquisition (X1)
Goodwill at acquisition X

W4: non-controlling interest (NCI)


$
Fair value of NCI at acquisition X
NCI% post acquisition reserves of subsidiary X
NCI to CSOFP X

198
W5: consolidated reserves
$
100% parent at year end X
Group % post acquisition reserves of subsidiary X
Less: PUP adjustment (P sells to S) (X)
Total to CSOFP X

W6: Associate
$
Investment at cost X
+group share of post-acquisition reserves of associate X
Investment in associate in CSOFP X

W7: any other adjustments given in Q

199
Consolidation of statement of profit or loss and other comprehensive income for
the year ended XX/XX/XXXX

Step1: structure
Step2: Proforma
Step3: workings

Consolidation of statement of profit or loss and other comprehensive


income for the year ended XX/XX/XXXX

Parent Subsidiary Adjustments Group


(post acq only)
Revenue X X (X) X

Less:
Cost of sales (X) (X) X
PUP (X) (X) - (X)
(who sells?) P-S S-P
Gross profit X X - X
Other income X X X

Less:
Distribution costs (X) (X) - (X)
Administration expenses (X) (X) - (X)
Finance Cost (X) (X) - (X)
Profit before tax X X - X

Less: tax (X) (X) - (X)


Profit for the year X X* - X

Attributable to:
NCI (NCI% X profit for X
the year)
Equity holders of the parent Remaining figure X
X

200
Mid-year acquisition:

We must only include the part of the subsidiary’s results that arose after acquisition, i.e.
whilst under the control of the parent. If the acquisition occurred in the middle of the
year, we should only include the second half of the subsidiaries results for the year.

Inter-company trading:

DR Group sales (Consolidated P/L)


CR Group cost of sales (Consolidated P/L)

201
Technical Consolidation Languages

Parent: An entity that has one or more subsidiaries.

Subsidiary: An entity controlled by another entity (the parent).

Trade (simple) investment:

An investment in the shares of another entity that is held for the accretion of
wealth, and is not an associate or a subsidiary.

Associate: An entity that the parent has significant influence over.

Non-controlling interest Shareholding in the subsidiary that is not owned by the


parent.

Control: The power to govern the financial and operating policies of an entity.

Significant influence: The power to participate in the financial and operating


policies of an entity.

202
Control and Significant Influence

Control:

For consolidation of statement of financial position:


Assets & Liabilities:
based on control definition (100% of Parent and 100% of subsidiary)

Equity:
based on ownership (consider Parent only)

This is based on the control definition according to IFRS 10 Consolidated Financial


Statements, 3 criteria need to be met:

P: Power instrument:

 Voting rights and potential voting rights; power to appoint directors on the board.

A: Activities (relevant).

 I like to think about relevant activities which are based on the purpose of the
organization. Such as if your company is going to manufacture high fashion clothes
then a relevant activity would be to determine the selling price of the high fashion
clothes in the whole market etc. An participation in preparation of accounts for the
company is just an irrelevant activity.

R:Returns (either positive or negative)

 The returns here could be positive or negative which means through the direction
of the activity within the company then the investor may be exposed to or have
right to profit or loss not necessarily benefit(profit.)
 Also the returns are variable which means that if the company is doing a good job
so it earns a larger amount of profit then it will distribute larger amount of dividend
to shareholders and this is called variable which is against “fixed”.
 Eg, a preference shareholder is just exposed to the fixed dividend received from
the company so it does not necessarily have control over the entity.

For consolidation of statement of profit or loss and other comprehensive income:

Revenue + expenses (100% of Parent and subsidiary) but if subsidiary is


acquired on mid-year: pro-rata the subsidiary figure.

203
Significant influence:

For consolidation of statement of financial position: (IAS28)


One line figure (Equity accounting)
Show the investment under non-current assets as a one line figure instead of
line by line adding P+S

For consolidation of statement of profit or loss and other comprehensive income:


One line figure
Show the Associate percentage of PAT before PBT in the consolidation.

204
Cost of Investment Calculation

Cash consideration =no. of share acquired X cash amount/share

Share exchange (3 steps)


 -No. of shares acquired
 -Shares issued from P in return
 -Value of P’s shares

The consideration can be the cash paid right now by the parent company, or it can
be deferred consideration or contingent consideration.

Deferred Consideration:

This is the share or cash to be issued or paid in the future.

For cash: We need to discount the total future cash payment to be included in the
fair value of consideration.

For shares: We need to use:


Number of shares to be issued in the future X Share price (Now)

Contingent Consideration:

This is cash or shares to be paid or issued in the future depending on whether


company can meet with targets.

So we need to use:
Probability to meet with targets X Fair value of contingent consideration (Cash or
Shares)

205
Q DC Ltd

BG plc bought 80% shares in DC ltd. The net assets of DC ltd are $11,400. The NCI
at the date of acquisition is $2,500.

The arrangement of the consideration is as follows:

Now:
1. BG plc agrees to pay $500 cash
2. BG plc agrees to issue 1m shares with $3.5/share

In 2 years time:
BG plc agrees to pay $2,000 in 2 years time;
BG plc agrees to issue an additional 2million shares in 2 years time.

The effective interest rate is 12%.

Required:
Calculate the goodwill at the date of acquisition.

206
Answer to DC Ltd:

FV of consideration: 12,594
Cash consideration 500
Shares (1m X $3.5/share) 3,500
Deferred Cash consideration
$2,000/1.12^2 1,594
Deferred Shares Issued
2m X $3.5/share 7,000
+NCI 2,500
-Net Assets (11,400)
Goodwill at acquisition: 3,694

W:
Deferred cash consideration:

At acquisition:
DR Goodwill 1,594
CR Liability 1,594

1st Year:
DR Finance Cost 191 (1,594X12%)
CR Liability 191

2nd Year:
DR Finance Cost 215 (1,785 X 12%)
CR Liability 215

Shares issued now:

DR Goodwill 3,500
CR Share Capital (1m X$1/share) 1,000
CR Share Premium 2,500

Shares to be issued in 2 years time:


DR Goodwill (2m X $3.5/share) 7,000
CR Share Capital (2m X $1/share) 2,000
CR Share Premium 5,000

207
Q PA & SA

PA agrees to pay:

 $2,000 cash in 2 years’ time;


 2 m shares to be issued in 2 years’ time.

The condition is that if SA’s profit will increase by $2m after the acquisition.

Management estimates there would be 60% chance to meet with this target.

NCI at the date of acquisition is $2,500. Net assets=$11,400.

The effective interest rate is 12%.

Required:
Calculate goodwill at acquisition.

Answer to PA & SA Ltd:

FV of consideration 5,156
Contingent Cash (2,000/1.12^2 X60%) 956
Contingent shares issued (2mX$3.5/shareX60%) 4,200
+ NCI 2,500
-Net Assets (11,400)
Bargain Purchase= (3,744)

For Contingent Cash (in 1 years’ time):

DR Finance Cost (956 X 12%) 115


CR Liability 115

208
Example: Consolidated SFP (Parodo Ltd)

Parodo specializes in high fashion clothes manufacturing industry. And recently the
prices in the woolen market have increased dramatically and there’s a shortage in
supply of these material as well.

And so it has pushed up the prices of the high fashion clothes. In order to secure
the woolen material supply and reduce its prices down, Parodo decided to acquire
Scan which is a woolen material supplier.

The financial statements of Parodo and Scan as at 31 DEC 2015 are as follows:

Parodo Scan
$000 $000 $000 $000
Non-current assets
Property, plant and equipment 1,000 500
Investments 450 0
1,450 500
Current assets
Inventory 300 250
Trade receivables 200 150
Bank 300 200
800 600
Total assets 2,250 1,100

Equity and liabilities


Equity
Share capital($1 each) 500 200
Retained earnings 650 400
1,150 600
Non-current liabilities
Loan notes 100 50

Current liabilities
Trade payables 1,000 450
Total equity and liabilities 2,250 1,100

Parado has the following issues:


1. Parado acquired 75% of Scan two years ago for $450,000 cash. At the date of
acquisition Scan retained earnings were $100,000.

2. The directors have deemed fair value of non-controlling interest at acquisition to


be $150,000

209
Required:
Prepare the consolidation statement of financial position for Parodo Group as at 31
December 2015.

210
Answer to Parodo

211
212
213
Consolidation adjustments

1, Intra group balances


Consolidated financial statements treat the parent and subsidiary as one single
entity. If the parent sold goods to the subsidiary there could be receivable and
payable balances between them at the end of the year.

If this is the case and we are treating the group as one single economic entity we
only want to show cash receivable and payable from external entities.

Therefore these balances between group companies are not true outstanding
balances from a group point of view and need to be cancelled or receivables and
payables would be overstated.

Journal:
DR Group payables (Consolidated SOFP)
CR Group receivables (Consolidated SOFP)

Example:

After the acquisition by Parodo, Scan owes $5,000 to Parodo.


So how can this be treated in the consolidation of statement of financial position?

Answer:

DR Group Payable $5,000


CR Group Receivable $5,000

214
2, Cash in transit

Cash has been sent by one group entity but has not been received and so it not
recorded in the books of the other group entity.

So we need to make this adjustment:

DR Bank (cash in transit)


CR Receivables (in seller’s book)

3, Goods in transit
Goods have been sent by one entity but have not been received and so are not
recorded in the books of the other group entity.

So we need to make this adjustment:


DR inventory (goods in transit)
CR Payable (in buyer’s book)

215
Q: PAPA & SASA(cash, goods in transit)

The following are the statement of financial position of PAPA and SASA as at 31st
March 2015.

PAPA SASA
Current assets
Inventory 100 50
Cash and cash 270 80
equivalents
Current liabilities
Payables 160 90

Before the year end, SASA sent a cheque for $4,000 to PAPA. PAPA didn't receive
this cheque until after the year end. Also, PAPA had dispatched goods on credit to
SASA with a value of $6,000 but SASA had not received them by the year end.

Required:
Prepare the consolidated statement of financial position for PAPA group as at 31
March 2015.

Answer to PAPA&SASA:

CSFP as at 31st march 2015:

$
Current assets
Inventory(100+50+6) 156
Cash and cash equivalents(270+80-4) 346
Current liabilities
Payables(160+90+6) 256

Adjustments:
1. Cash in transit:
DR Bank 4
CR Receivable 4 (in seller-PAPA’s book)

2. Goods in transit:
DR inventory 6
CR Payable (in buyer-SASA’s book) 6

216
4, Provision for unrealized profit (PUP)

1. Inventory (PUP)

When goods are sold by one company in a group to another in the same group a
cancellation would be required to remove, according to the single entity concept,
the receivable/payable amount on the group statement of financial position.

But these inter-company transfers are usually done at a marked-up price to reflect
the fact that as separate legal entities they sell to one another at a profit. Since we
view the group as a single entity this profit must be identified and then eliminated.

We have to consider:
Direction of sale & inventory left in warehouse

P-S: DR Consolidated reserve


CR Inventory

S-P: DR subsidiary retained earnings (net assets list)


CR Inventory

217
Example PUP:

1, Parodo sells $100,000 worth of goods to Scan (an 80% subsidiary) at cost plus
25% (25% mark-up). Scan had not sold any of the goods outside the group by the
end of the year.

Required:
Calculate the provision for unrealized profit (PUP) and show the double
entry for this transaction.

218
2, In the post-acquisition period Scan’s sales to Parodo were $50 million on which
Scan had made a margin of 5% on these sales. Of these goods, $7 million (at
selling price to Parodo) were still in the inventory of Parodo at its year-end of 31
December 2015. Parodo holds a controlling interest of 70% in Scan.

Required:
Calculate the provision for unrealized profit (PUP) and show the double entry for
this transaction

219
2. Non-Current Assets (PUP)

Situation1: parent sells to subsidiary.

Parent sells the NCA to subsidiary for $5,000 where its carrying value is $4,000.
The remaining useful life of the NCA at the date of sale is 5 years.
Parent owns 80% of subsidiary.

Treatment:

Step1: remove profit and PPE:


DR Group retained earnings ($5,000-$4,000) $1,000
CR PPE $1,000

Step2: remove additional depreciation:


DR PPE (Accumulated depreciation) ($5,000-$4,000)/5 years $200
CR Group retained earnings (depreciation expense) $200

Situation2: subsidiary sells to parent:

Subsidiary sells the NCA to parent for $5,000 where its carrying value is $4,000.
The remaining useful life of the NCA at the date of sale is 5 years.
Parent owns 80% of subsidiary.

Treatment:

Step1: remove profit and PPE:


DR Group retained earnings 80%X($5,000-$4,000) $800
DR NCI $200
CR PPE $1,000

Step2: remove additional depreciation:


DR PPE (Accumulated depreciation) ($5,000-$4,000)/5 years $200
CR Group retained earnings (depreciation expense) 80%X$200 $160
CR NCI $40

220
5, Fair value adjustments
Sometimes, at the point of acquisition, the recorded book value of an item in the
subsidiary statement of financial position is different to its fair value.

Eg a property may be carried on the statement of financial position at a depreciated


historic cost but it is worth 8 times that on the open market. (Or conversely, 8
times less!).

A subsidiary’s net assets should be adjusted to fair value before goodwill is


calculated and the fair value adjustments included in the group figures.

Fair value (market value) is the amount for which an asset could be exchanged,
or a liability settled, between knowledgeable, willing parties in an arm’s length
transaction (normal commercial transaction with no discounts).

221
Example: (SJ Ltd) (fair value adjustments)

In order to remove its major competitor, Parodo decided to acquire another


company SJ on 1 Jan 2016.

SJ has the following equity balances:

At 1 Jan 2016 At 31 DEC 2016


$000 $000
Equity ($1) share capital 300,000 300,000
Share premium 150,000 150,000
Retained earnings 100,000 160,000
550,000 610,000

Parodo acquired 80% of the equity share capital of S on 1 January 2016 for an
agreed cash amount of$600 million.

At the date of acquisition the carrying values of SJ’s assets were approximately
equal to their book values with the exception of some land that has a fair value of
$50 million but it is currently carried at $40 million in SJ’s financial statements.

The fair value of the non-controlling interest at acquisition is $200 million.

Required:
1, Calculate the goodwill from the acquisition of SJ Ltd
2, show the double entry for the fair value adjustments

222
Answer to SJ Ltd:

223
Goodwill

Goodwill calculation

Before we have a look at goodwill impairment, I would like to remind you of how to
calculate goodwill.

There are basically two methods in goodwill calculation per IFRS3. (Q: Good)

Full goodwill method:


FV of consideration X
FV of NCI(full goodwill)* X
FV of business X
FV of net assets (X)
Goodwill X

*Full goodwill method (commonly used in exam):


Where the NCI is valued at fair value and is deemed to have some ownership of
goodwill.

NCI +some goodwill

Fair value

Partial goodwill method:


FV of consideration X
proportionate share of net asset(partial goodwill) * X
FV of business X
FV of net assets (X)
Goodwill X

*Partial goodwill method (rarely used):


Where the NCI is valued at its proportionate share of net assets and is deemed to
have no ownership of goodwill.

NCI +no goodwill

Proportionate share of net assets

224
Goodwill impairment

After buying this subsidiary and you have paid more than its net asset and the
excess amount is called goodwill which represents future economic benefit that
company might generate and for its good reputation and some other intangible
assets such as customer database. But is it really as good as you think it should be?
Well, that’s why we have to carry out an impairment review at the year-end for the
subsidiary to see whether what we bought is actually as good as we think it should
be.

We do not amortize goodwill simply because goodwill has an infinite life which
means we will never know when the company goes burst in the future and also we
can’t measure reliably about the future benefit that company can generate and
amortization is like depreciation for tangible non-current asset which match the
future revenue that company generates using this asset against its cost.

So instead we carry out an impairment review for the goodwill at the end of each
year for subsidiary and associate we bought.

225
Goodwill is impaired if its

Carrying amount > Recoverable amount (always given in Q).


Goodwill (recorded) Higher of -VIU (value in use)
+Net asset (year-end) -NRV (net realizable value)

For carrying amount of goodwill:


Full goodwill method: G/W recorded in W3
Partial goodwill method: G/W recorded in W3+notional goodwill

Impairment of goodwill:

Full goodwill:

DR NCI
DR consolidated reserve
CR goodwill

Rules for partial goodwill impairment:

Impairment<gross up(Parent G/W+NCI Impairment > Gross up(Parent G/W+NCI


G/W[notional G/W]) G/W[notional G/W])

Decrease P% of impairment Decrease NCI impairment


Decrease NCI% of impairment Decrease all Parent G/W impairment
Decrease remaining assets and split between P and S.

226
Goodwill impairment in subsidiary

*The goodwill impairment in SOFP:

Full goodwill:
DR NCI
DR consolidated reserve
CR goodwill

FV of consideration X
FV of NCI(full goodwill) X
FV of business X
FV of net assets (X)
Goodwill X
Goodwill impairment(CSOFP date)* (X)
Goodwill net book value X
*The goodwill impairment will be split into NCI & Consolidated Reserve

NCI:
FV of NCA at acq X
NCI% post acquisition reserves of subsidiary X
-NCI% of goodwill impairment (X)
NCI in CSOFP X

Consolidated Reserve
100% parent at year end X
Group % post acquisition reserves of subsidiary X
- PUP adjustment (P sells to S) (X)
- P% of goodwill impairment (X)
Total to CSOFP X

Partial goodwill:
DR consolidated reserve
CR goodwill

*The goodwill impairment in SOCI:


DR impairment
CR accum impairment
As an expense before PBIT (operating profit)

(Within the consolidated statement of profit or loss and other comprehensive


income any expenses that will decrease the NCI figure for subsidiary.)

227
Impairment for Associate

Investment at cost X
+Group% post acq reserve X
-impairment loss(same logic behind (X)
goodwill, take the total figure)
Valuation of associate X

228
Q: Good (full & partial goodwill calculation)

Good Ltd bought Bad Ltd last year for $260 for 80% of shares. The fair value of the
NCI at that date was $60. The net assets of Bad Ltd at that point was $250.

Required:
Calculate the goodwill using full goodwill method and partial goodwill method.
(6marks)

Answer:
Full goodwill:
FV of consideration 260
FV of NCI 60
FV of business 320
FV of N/A (250)
Goodwill 70

Partial goodwill method:


FV of consideration 260
Proportionate share of NA(20%X250) 50
FV of business 310
FV of NA (250)
Goodwill 60

229
Q Digi, Cici and Kuki (goodwill impairment)

Digi
Parent company owns 80% of share in Digi.

Goodwill at the data of acquisition is $60m.

Net assets at the consolidated financial statement date is $1,089m.

Goodwill is impaired tested and the recoverable amount of Digi’s net asset is
$1,099m.

Full goodwill method is used in Digi.

Cici [partial goodwill rule 1]


Parent company owns 80% of share in Cici.

Goodwill at the data of acquisition is $60m.

Net assets at the consolidated financial statement date is $1,089m.

Goodwill is impaired tested and the recoverable amount of Digi’s net asset is
$1,099m.

Partial goodwill method is used in Digi.

Kuki [partial goodwill rule 2]


Parent company owns 60% of share in Kuki.

Goodwill at the data of acquisition is $80m.

Net assets at the consolidated financial statement date is $2,255m.


Goodwill is impaired tested and the recoverable amount of Digi’s net asset is
$2,088m.

There was no impairment of the net assets of Kuki before this date and any
impairment loss has been determined to relate to goodwill and property, plant and
equipment.

230
Answer

Digi Cici Kuki


G/W @acq 60 60 80
Notional 60/80%X20%=15 80/60%
G/W X40% =
53.3
Net assets 1,089 1,089 2,255
@SFPdate
Recoverable (1,099) (1,099) (2,088)
amount
Impairment 50 65 300.3
X80% -notional G/W (53.3)
52 247

Digi:
DR consolidated reserve (80%X50) 40
DR NCI (20%X50) 10
CR Goodwill 50

Cici:
DR consolidated reserve 52(parent)
CR Goodwill 52

Kuki: (impairment relates to goodwill and PP&E)


Goodwill: DR consolidated reserve 80
CR goodwill 80
PP&E:
DR consolidated reserve (60% X (247-80) = 100.2)
DR NCI (40% X (247-80) = 66.8)
CR PP&E 167

231
Q Associal (impairment for associate)

On 1 February 2013, Paro, who already has a subsidiary Subso, acquired 25% of the
equity shares of Associal paying $10m in cash. Associal retained earnings for the year
ended 30 September 2013 is $1.2m, much lower than in previous years. Due to this
the value of the investment in Associal was impaired by $2.5m

Required:
Calculate the value of Associal shown in the consolidated statement of financial
position as at 31 Sep 2013 and consolidated statement of profit or loss and other
comprehensive income for the year ended 31 Sep 2013.

232
Example: (PG group) Consolidation statement of profit or loss and
other comprehensive income:

PG acquired 80% of Su on 1 October 2014 for $10million because it thinks after the
acquisition then PG can expand its market in India because Su is based in India. The
following statements of comprehensive income for both companies for the year ended
31 DEC 2014.

PG Su
$000 $000
Sales revenue 110,000 80,000

Cost of sales (50,000) (30,000)

Gross profit 60,000 50,000

Administrative expenses (20,000) (35,000)

Profit before tax 40,000 15,000

Income tax expense (10,000) (5,000)

Profit for the year 30,000 10,000

The following notes are relevant:


1. During the post-acquisition period Su sold $5 million of goods to PG at a mark-up of
25% on cost. A quarter of these goods are in inventory at the year-end.

2. Assume all profits accrue evenly over the year.

Required:
1, point out what is mid-year acquisition of subsidiary and state the relevance in this
example.

2, prepare the consolidation statement of profit or loss and other comprehensive


income for the year ended 31 DEC 2014.

233
Answer:

234
235
Associate (IAS 28 investment in associates)

The investor will have significant influence if he has invested 20%-50% of shares in
another company (associate).

The significant influence is the power to participate into the decision making
process of the company.

The 20%-50% is just a subjective test and in reality even if company fails this test,
maybe it is still having/ having no significant influence over another company:

-if you have 19% shares of another company but there are remaining shareholdings
around from 0.5%-1% and if this is the case, you have significant power to participate into
the decision making process regardless of the failure of the test.

-If you have 25% shares of another company but there’s a very big shareholder who is
holding 70% of shares in the company and in this case you are too small and even though
you comply with the test(20%-50%) but you have no significant influence.

236
How to account for an associate: (Cost + Growth)
[Equity Accounting]

$
Investment at cost X
+group share of post-acquisition of associate (Growth) X
Investment in associate for CSOFP X

Presentation in CSOFP:
Non-current assets
Property, plant and equipment X
Goodwill X
Investment in associate X

Presentation in CP/L:
Parent Subsidiary Adjustments Group
Gross profit X X - X
Expenses (X) (X) - (X)
Profit in Associate (45% of associate profit after tax) X -
Profit before tax X X - X

PUP for associate:

PUP X group share

Parent/Subsidiary sells to associate:

DR RE/ Net assets


CR investment in associate

Associate sells to parent:


DR RE
CR inventory

237
Example: (Plogo Co)

Plogo Co, acquires 25,000 of the 100,000 $1 ordinary shares in Aim Co for $60,000 on 1
January 2015. In the year to 31 December 2015, Aim Co earns profits after tax of $24,000,
from which it pays a dividend of $6,000.

How will Aim Co's results be accounted for in the individual and consolidated accounts of
Plogo Co for the year ended 31 December 2015?

Answer:

Individual:
DR Cash $1,500
CR Dividends received-other income (6,000 × 25%) $1,500

Consolidation:
1, statement of financial position:

Non-current asset:
Investment in associate (W) 64,500

W:
Cost of investment in associate 60,000
Share of A’s profit for the year 6,000
Less dividend received (1,500)
Investment in associate 64,500

2, statement of profit or loss and other comprehensive income:


Presentation in CSOCI:
Parent Subsidiary Adjustments Group
Gross profit X X - X
Expenses (X) (X) - (X)
Profit in Associate (25% of associate profit after tax) 6,000 - 6,000
Profit before tax X X - X

238
Basic Group consolidation in P2 [Q Bravado]
Bravado, a public limited company, has acquired two subsidiaries and an associate.
The draft statements of financial position are as follows at 31 May 2013.

The following information is relevant to the preparation of the group financial statements.
(a) On 1 June 2012, Bravado acquired 80% of the equity interests of Message, a private
entity. The purchase consideration comprised cash of $300 million. The fair value of the
identifiable net assets of Message was $400 million, including any related deferred tax
liability arising on acquisition. The owners of Message had to dispose of the entity for tax
purposes by a specified date, and therefore sold the entity to the first company to bid for it,
which was Bravado. An independent value has stated that the fair value of the
non-controlling interest in Message was $86 million on 1 June 2012. Bravado does not wish
to measure the non-controlling interest in subsidiaries on the basis of the proportionate
interest in the identifiable net assets, but

239
Message were $136 million and other components of equity were $4 million at the date of
acquisition. There had been no new issue of capital by Message since the date of acquisition
and the excess of the fair value of the net assets is due to an increase in the value of
non-depreciable land.

(b) On 1 June 2011, Bravado acquired 6% of the ordinary shares of Mixed. Bravado had
treated this as an as investment in equity instruments at fair value in the financial
statements to 31 May 2012, and had made an irrevocable election (see note (d)) to
recognize changes in fair value in other comprehensive income. There were no changes in
the fair value of Mixed in the year to 31 May 2013. On 1 June 2012, Bravado acquired a
further 64% of the ordinary shares of Mixed and gained control of the company. The
consideration for the acquisitions was as follows.

Holding Consideration
1 June2007 6% 10
1June2008 64% 118
70% 128

Under the purchase agreement of 1 June 2012, Bravado is required to pay the former
shareholders 30% of the profits of Mixed on 31 May 20Y0 for each of the financial years to
31 May 2013 and 31 May 20Y0. The fair value of this arrangement was measured at $12
million at 1 June 2012 and at 31 May 2013 this value had not changed. This amount has not
been included in the financial statements.

At 1 June 2012, the fair value of the equity interest in Mixted held by Bravado before the
business combination was $15 million, and the fair value of the non-controlling interest in
Mixted was $53 million.

The fair value of the identifiable net assets at 1 June 2012 of Mixted was $170 million
(excluding deferred tax assets and liabilities), and the retained earnings and other
components of equity were $55 million and $7 million respectively. There had been no new
issue of share capital by Mixted since the date of acquisition and the excess of the fair value
of the net assets is due to an increase in the value of property, plant and equipment (PPE).

The fair value of the PPE was provisional pending receipt of the final valuations for these
assets. These valuations were received on 1 December 2012 and they resulted in a further
increase of $6 million in the fair value of the net assets at the date of acquisition. This
increase does not affect the fair value of the no controlling interest. PPE is depreciated on
the straight -line basis over seven years. The tax base of the identifiable net assets of
Mixted was $166 million at 1 June 2012. The tax rate of Mixted is 30%.

(c) Bravado acquired a 10% interest in Clarity, a public limited company, on 1 June 2011
for $8 million. The investment was accounted for as an investment in equit y instruments
and at 31 May 2012, its value was $9 million. On 1 June 2012, Bravado acquired an

240
additional 15% interest in Clarity for $11 million and achieved significant influence. Clarity
made profits after dividends of $6 million and $10 million for the years to 31 May 2012 and
31 May 2013. An irrecoverable election was made to take changes in fair value through
other comprehensive income (items that will not be reclassified to profit or loss).

(d) Bravado has made an irrevocable election to hold its investments in Message, Mixted
and Clarity at fair value with changes in fair value recognised in other comprehensive
income. There were no changes in fair value during the year ended 31 May 2013.

(e) On 1 June 2011, Bravado purchased an equity instrument of 11 million dinars which was
its fair value. On that date an election was made to hold it at fair value through other
comprehensive income. The relevant exchange rates and fair values were as follows:
$ to dinars FV of instrument-Dinars
1 June 2011 4.5 11
31 may 2012 5.1 10
31 may 2013 4.8 7

Bravado has not recorded any change in the value of the instrument sinc e 31 May 2012.
The reduction in fair value as at 31 May 2013 is deemed to be as a result of impairment.

(f) Bravado manufactures equipment for the retail industry. The inventory is currently
valued at cost. There is a market for the part completed product at each stage of production.
The cost structure of the equipment is as follows.
Cost per unit Sellig price per unit
st
Production process:1 stage 1,000 1,050
Conversion costs: 2nd stage 500
Finished product 1,500 1,700

The selling costs are $10 per unit, and Bravado has 10,000 units at the first stage of
production and 20,000 units of the finished product at 31 May 2013. Shortly before the year
end, a competitor released a new model onto the market which caused the equipment
manufactured by Bravado to become less attractive to customers. The result was a
reduction in the selling price to $1,450 of the finished product and $950 for 1st stage
product.

(g) The directors have included a loan to a director of Bravado in cash and cash equivalents
of $1 million. The loan has no specific repayment date on it but is repayable on demand.
The directors feel that there is no problem with this accounting entry as there is a choice of
accounting policy within International Financial Reporting Standards (IFRS) and that
showing the loan as cash is their choice of accounting policy as there is no IFRS which says
that this policy cannot be utilized.

(h) There is no impairment of goodwill arising on the acquisitions.


Required

241
(a) Prepare a consolidated statement of financial position as at 31 May 2013 for the
Bravado Group. (35 marks)
(b) Calculate and explain the impact on the calculation of goodwill if the non-controlling
interest was calculated on a proportionate basis for Message and Mixted. (8 marks)
(c) Discuss the view of the directors that there is no problem with showing a loan to a
director as cash and cash equivalents, taking into account their ethical and other
responsibilities as directors of the company.
(5 marks)
Professional marks will be awarded in part (c) for clarity and expression of your discussion.
(2 marks)
(Total = 50 marks)

242
Changes in Ownership

243
Situations

1. Simple investment to subsidiary

2, Associate to subsidiary

3, >50%-<50%: step acquisition: calculate goodwill using 1st and 2nd investment.

SOFP SOCI
FV of 1st investment X
FV of 2nd investment X FV of 1st investment X
FV of NCI X Cost of 1st investment (X)
FV of business X Gains/losses(P/L) X
FV of Net asset (X)
Goodwill at acq X

4, <50%-100%: still control Equity adjusting: (acquisition of NCI)


DR NCI (decrease)
CR Cash
DR/CR retained earnings(equity)(transaction between owner controlled)
-CR side means there’s increase in controlling interest
-DR side means there’s decrease in controlling interest
-NCI: decrease NCI%X net assets @ disposal date

5, 100%-<50%: still control (Disposal of NCI)


DR Cash
CR NCI
DR/CR retained earnings(equity)(transaction between owner controlled)
-CR side means there’s increase in controlling interest
-DR side means there’s decrease in controlling interest

6, 100%->50%: loss control: deem sale of whole group

Cash in X1
NCI Reduction(reduce benefit to others so it’s benefit to us) X2
Remaining(either investment/associate) X3
Total business @disposal X
Total things left @disposal Y
Unimpaired Goodwill at disposal Y1
Net assets at disposal (growth?) Y2

Gain/losses(X-Y)-P/L [gain/loss on disposal of sub]

244
Q Simple 1 (from simple investment to subsidiary)

Simple1 acquired 5% of shares in Sub1 on 1st Sep 2013 for $5,000.

Asso1 acquired another 55% of shares in Sub1 on 1st Sep 2014 for $12,000. And at this
date, the value of the original investment was remeasured at $8,000. And the fair value
of NCI is $3,250(full goodwill). And the net asset was $13,000 at the date of acquisition.
The reserve of parent at 31st March 2015 is $49,000.

Net asset at the date that Simple1 gained control was $13,000.
Net assets changes since acquisition were $8,000.

Required:
Show the value of:
1. Goodwill
2. NCI
3. Reserves
4: simple investment.

Answer:

Group structure:
Before becoming a sub: own 5% shares
After becoming a sub: own (5%+55%=60%)shares

W3 Goodwill:
5% investment: 8,000(updated)
55% investment: 12,000
+ NCI 3,250
- Net assets (13,000)
Goodwill at acquisition 10,250

W4 NCI:
NCI 3,250
+ growth (40%X$8,000) 3,200
6,450

W5 Reserves:
Parent: 49,000
+ growth (60%X$8,000) 4,800
Revaluation of simple investment(W6) 2,100
58,900

245
W6 simple investment

Cost of 30% investment 5,000


Revaluation gain (bal)-P/L 3,000
Value at control 8,000

In effect, we cancel the associate:


DR Goodwill (W3) 8,000
CR simple investment 8,000

246
Q Asso 1 (from Associate to subsidiary)

Asso1 acquired 30% of shares in Sub1 on 1st Sep 2013 for $5,000.

Asso1 acquired another 45% of shares in Sub1 on 1st Sep 2014 for $12,000. And at this
date, the value of the original investment was remeasured at $8,000. And the fair value
of NCI is $3,250(full goodwill). And the net asset was $13,000 at the date of acquisition.
The reserve of parent at 31st March 2015 is $49,000.

Net assets changes from 1st Sep 2013 to 1st Sep 2014 were $3,000.
Net assets changes from 1st Sep 2014 to current year end(31st March 2015) is $8,000.

Required:
Show the value of:
1. Goodwill
2. NCI
3. Reserves
4: Associate.

Answer:

Group structure:
Before becoming a sub: own 30% shares
After becoming a sub: own (30%+45%=75%)shares

W3 Goodwill:
30% investment: 8,000(updated)
45% investment: 12,000
+ NCI 3,250
- Net assets (13,000)
Goodwill at acquisition 10,250

W4 NCI:
NCI 3,250
+ growth (25%X$8,000) 2,000
5,250

W5 Reserves:
Parent: 49,000
+ growth (75%X$8,000) 6,000
+ growth (30%X$3,000) 900
Revaluation of associate(W6)-P/L 2,100
58,000

247
W6 Associate

Cost of 30% investment 5,000


+ Growth (30%X $3,000) 900
5,900
Revaluation gain (bal) 2,100
Value at control 8,000

In effect, we cancel the associate:


DR Goodwill (W3) 8,000
CR investment in associate 8,000

248
Q More Ltd (buying more shares with control)
More Ltd acquired 70% of less ltd on 1 Jan 2015 FV of NCI was $46m.

More Ltd acquires a further 10% of less ltd on 1 May 2015 for $19m. Less ltd has made
profits from 1 Jan 2015 to 1 May 2015 for $7m and $5m from 1 May 2015 to 31 DEC 2015.

Required:
Show the treatment of the above process.

Answer:
W:
FV of NCI 46
+ growth (30% X 7) 2.1
48.1
X 10%/30% (48.1 X1/3) (16)
+ growth (20% X 5) 1
49.1
NCI revised= 33

Adjustment:
DR NCI (w) 16
CR Cash 19
DR (balancing) Equity (OCE) 3

249
Q Distort Ltd (sell of shares but remains control)

Distort Ltd Bought Innocent Ltd on 1 March 2014 and the FV of NCI at that time was
$7,500 and the group values the NCI at its fair value.

The retained earnings on 1 March 2015 is $74,000.

Distort Ltd disposes of 15% of shares of Innocent Ltd for $16m and so reduces its
ownership to 60%.

Net asset of Innocent at the disposal date was $54,000.

Net assets changes from acquisition date up to disposal date are $24,000.
Net assets changes from disposal date to Year end are $6,000.

Required:
Show the treatment of the above process.

Answer:

W4: NCI
FV of NCI 7,500
Growth(acq-disposal): 25%X24,000 6,000
Growth(Disposal to year end): 40%X6,000 2,400
Increase in NCI at disposal: 15%X54,000(N/A) 8,100
24,000

W5: Group reserve


Parent 74,000
Growth: (from acq to disposal): 75%X24,000 18,000
Growth: (from disposal to year end): 60%X6,000 3,600
NCI adjustment (W) 7,900
103,500

W:
DR cash 16,000
CR NCI 8,100
CR Equity (balancing) 7,900

250
Q Complete Ltd (disposal of subsidiary)
On 1st Jan 2014 Nothing Ltd, Complete Ltd acquired 60% of Nothing Ltd for $360m. Nothing
Ltd had identifiable net assets with a fair value of $400m at acquisition and the fair value of
the NCI was $210m. Complete Ltd uses FV to value NCI.

On 31st Dec2014, Complete Ltd sells 15% of Nothing Ltd for $150m and loses control, but
retains influence through its remaining 45% ownership. The fair value of the associate
retained is measured at $400m.

On 31st DEC2014 Nothing Ltd had identifiable net assets of $440m. The growth of Nothing
Ltd is $30m.

Required:
Show the treatment of the above process.

Answer:

Total business at disposal 772


Cash in 150
NCI(210+40%X30) 222
Remaining(investment/associate) 400
Total things left at disposal 600
Goodwill at disposal(W1) 160
Net assets at disposal 440

Disposal of sub 172

W1: Goodwill:
FV of consideration 360
FV of NCI 210
FV of business 570
FV of NA (400)
Goodwill 160

251
Q NCI in P/L:

QQ plc bought 80% of shares in AA plc on 1 Jan 2014. It bought another 10% of shares
in AA Plc on 1 April 2014.

AA Plc has made profits of $1.8m evenly throughout the year.

Required:
What share of profits should be allocated to the NCI in the group P/L?

Answer:
NCI:
1. Jan 2014-1 April 2014: (20% X 1,800 X3/12)=90
1. April 2014-31 DEC 2014: (10%X1,800X9/12)=135
225

252
Complex Group

In the p2 exam the only complex group that examiner is going to test you
is the simple vertical group structure.

1, Vertical group (simple)

70%

70%

SS

The only difference between the simple group and the vertical group
consolidation will be reflected in:

1, date of consolidation of SS: date when SS comes into group (in p2unlikely to be
examined.)
2, Growth: effective percentage (SS): used in NCI & Consolidated reserve
3, Goodwill-FV of consideration in SS: P%X investment (S-SS)
NCI in S: NCI% X investment (S-SS)

253
2, Dee-group

The difference of this and the above is when trying to calculate the
goodwill then the FV of consideration will need to add direct investment
by P to SS.

70%

10%
S

70%

SS

254
Q Pal plc (Effective control)

Senario1:
Paf plc acquired 70% of Saf which owns 70% of Surf.

Senario2:
Paf plc acquired 70% of Saf and 20% of Surf.
Saf owns 70% of Surf.

Required:
Calculate the effective controlling interest and non-controlling interest of Paf to
Surf.

Q: Pal plc (Date of acquisition of SS)

Senario1:
Paf plc acquired 70% of Saf on 1/1/2013.
Saf owns 70% of Surf since 1/1/2012.

Senario2:
Paf plc acquired 70% of Saf on 1/1/2013.
Saf owns 70% of Surf on 31/3 2013.

Required:
What is the acquisition date of Surf by Paf in the above 2 scenarios?

255
Q Patrick plc

Extract from statement of financial position as at 1/1/2014:

Patrick Super Sole


$000 $000 $000
Non-current assets
Investment in Super(14,000 shares) 60
Investment in Sole(6,000 shares) 50

Net assets list of Super and Sole:

Super Sole
Acq date Y/E Acq date Y/E
$000 $000 $000 $000
Share capital 20 20 10 10
Reserves 60 230 30 90
80 250 40 100

It is the group’s policy to use partial goodwill.


Parent’s reserve at the SOFP date is $230,000.

Required:
Calculate goodwill arising from Super and Sole. (no impairment)
Calculate NCI for Super and Sole.
Calculate Consolidated Reserve.

256
Q Patrick plc (modified)

Extract from statement of financial position as at 1/1/2014:

Patrick acquired 75% of Super and 25% of Sole.


Super owns 40% of Sole.

Patrick Super Sole


$000 $000 $000
Non-current assets
Investment in Super 60
Investment in Sole 20 50

Net assets list of Super and Sole:

Super Sole
Acq date Y/E Acq date Y/E
$000 $000 $000 $000
Share capital 20 20 10 10
Reserves 60 230 30 90
80 250 40 100

It is the group’s policy to value NCI at fair value.


Fair value of NCI for Super was $24,000 and for Sole was $23,000.
Parent’s reserve at the SOFP date is $230,000.

Required:
Calculate goodwill arising from Super and Sole. (no impairment)
Calculate NCI for Super and Sole.
Calculate Consolidated Reserve.

257
Past exam question (GLOVE) (June2007)

The following draft statements of financial position relate to Glove, Body and Fit, all public
limited companies, as at 31 May 2007.
Glove Body Fit
$m $m $m
Assets
Non-current assets
Property, plant and equipment 260 20 26
Investment in body 60
Investment in Fit 30
Investment in equity instruments 10
Current assets 65 29 20
Total assets 395 79 46

Ordinary shares 150 40 20


Other reserves 30 5 8
Retained earnings 135 25 10
Total equity 315 70 38

Non-current liabilities 45 2 3
Current liabilities 35 7 5
Total liabilities 80 9 8
Total equity and liabilities 395 79 46

The following information is relevant to the preparation of the group financial statements.
(a) Glove acquired 80% of the ordinary shares of Body on 1 June 2005 when Body's other
reserves were $4 million and retained earnings were $10 million. The fair value of the net
assets of Body was $60 million at 1 June 2005. Body acquired 70% of the ordinary shares
of Fit on 1 June 2005 when the other reserves of Fit were $8 million and retained earnings
were $6 million. The fair value of the net assets of Fit at that date was $39 million. The
excess of the fair value over the net assets of Body and Fit is due to an increase in the value
of non-depreciable land of the companies. There have been no issues of ordinary shares in
the group since 1 June 2005.

(b) Body owns several trade names which are highly regarded in the market place. Body
has invested a significant amount in marketing these trade names and has expensed the
costs. None of the trade names has been acquired externally and, therefore, the costs have
not been capitalised in the statement of financial position of Body. On the acquisition of
Body by Glove, a firm of valuation experts valued the trade names at $5 million and this
valuation had been taken into account by Glove when offering $60 million for the
investment in Body. The valuation of the trade names is not included in the fair value of the
net assets of Body above. Group policy is to amortize intangible assets over ten years.
(c) On 1 June 2005, Glove introduced a defined benefit retirement plan. During the year to

258
31 May 2007, loss on measurement on the defined benefit obligation was $1m, and gain on
measurement on the plan assets were $900,000. These have not yet been accounted for
and need to be treated in accordance with IAS 19, as revised in 2011. The net defined
benefit liability is included in non-current liabilities.

(d) Glove has issued 30,000 convertible bonds with a three year term repayable at par. The
bonds were issued at par with a face value of $1,000 per bond. Interest is payable annually
in arrears at a nominal interest rate of 6%. Each bond can be converted at any time up to
maturity into 300 shares of Glove. The bonds were issued on 1 June 2006 when the market
interest rate for similar debt without the conversion option was 8% per annum. Glove does
not wish to account for the bonds at fair value through profit or loss. The interest has been
paid and accounted for in the financial statements. The bonds have been included in
non-current liabilities at their face value of $30 million and no bonds were converted in the
current financial year.

(e) On 31 May 2007, Glove acquired plant with a fair value of $6 million. In exchange for
the plant, the supplier received land, which was currently not in use, from Glove. The land
had a carrying value of $4 million and an open market value of $7 million. In the financial
statements at 31 May 2007, Glove had made a transfer of $4 million from land to plant in
respect of this transaction.

(f) Goodwill has been tested for impairment at 31 May 2006 and 31 May 2007 and no
impairment loss occurred.

(g) It is the group's policy to value the non-controlling interest at acquisition at its
proportionate share of the fair value of the subsidiary's identifiable net assets.

(h) Ignore any taxation effects.

Required
Prepare the consolidated statement of financial position of the Glove Group at 31 May 2007
in accordance with International Financial Reporting Standards (IFRS). (25 marks)

259
IAS 21 The effects of changes in foreign exchange rates

(i) Foreign transaction


(ii) Foreign subsidiary

(i) Foreign transaction:


When you purchase/sell goods from/to other company in other countries
Step1: You need to firstly translate this transaction in functional currency at spot rate.
Step2: You need to retranslate the monetary item (Bank, receivable, payable, NCL, CL)
at the year end and leave non-monetary items(NCA, CA).

How to determine your functional currency?


Mainly this is the currency that when you’re trying to prepare your trial balance.
Factors to consider:
1, Which currency you use to determine selling price and costs?
2, Which currency is your competitor use to determine selling price or costs?
3, Which currency you use to finance your business?

260
(ii) Foreign subsidiary

Statement of financial position: closing rate


Statement of profit or loss and other comprehensive income: average rate

261
Q KIKKO (Foreign Transaction)

KIKKO buys a device on 5 months credit from France for €60,000 just before the year end
of 31/12/2013. It takes delivery on 10/12/2013. The functional currency of KIKKO is $.
KIKKO pays payable balance on 31/5/2014.

The rates are as follows:

Date Rate
Delivery (10/12/2013) $1: €1.250
Y/E (31/12/2013) $1: €1.300
Payment(31/5/2014) $1: €1.100

Required:
Show the journal relating to the above transaction.

Answer:

Step1: Translate to functional currency:


€60,000/€1.250 = $48,000
DR PPE $48,000
CR Payable $48,000

Step2: Retranslate the monetary item as at the year end:


€60,000/€1.300 =$46,154

DR Payable ($48,000-$46,154) $1,846


CR P/L (Gain) $1,846

Step3: Payment
DR payable $46,154
CR Bank (€60,000/€1.100) $54,545
DR P/L (Loss) $8,391

262
Foreign exchange gains/losses on Statement of cash flow:

Q Amanda (forex gain/losses)

The opening and closing balance of working capital of Amanda are as follows:

Inventory Receivable Payable


Opening balance $400 $200 $320
Closing balance $500 $450 $300

The exchange gains and losses regarding the working capital are as follows:

Inventory Gain $10


Receivables Gain $20
Payables Loss $10

Required:
Calculate the cash movements for the working capital for the year.

Answer to Amanda:

Opening- Closing net


Inventory 400- 500 +10 =-90
Receivable 200- 450 +20 =-230
Closing Opening net
Payable 300- 320 + (-10) =-30

Inventory: instead of spending $100 ($400-$500) but with a gain of $10 so net spent
is only $90.

Receivable: instead of increase in receivable of $250(in effect this is like cash out) but
because with $20(gain) so that the net cash out is only $230.

Payable: instead of decrease in payable of $20(in effect this is like cash out) and
because with the loss of $10 so the net cash out is $30.

263
Foreign Subsidiary Consolidation (Full Approach)

Q: Lukas and Gaga

Statement of financial position:

Lukas Gaga
Property, plant and 218,000 100,000
equipment
Financial assets 22,000 -
Investment in Gaga 100,000 -
Current Assets 90,000 209,600
430,000 309,600
Ordinary share capital($1/ 175,000 50,000
DN1 share)
Share Premium 50,000 10,000
Revaluation Reserve 40,000 -
Retained earnings 125,000 150,000
Total Equity 390,000 210,000

Non-current liabilities 15,000 88,000


Current liabilities 25,000 11,600
430,000 309,600

Statement of profit or loss and other comprehensive income:


Lukas Gaga
Sales revenue 500,000 300,000
Operating Costs (400,000) (150,000)
Operating profit 100,000 150,000
Finance Cost (6,100) (3,300)
Interest receivable 1,100 -
Profit before tax 95,000 146,700
Tax Expense (20,000) (46,700)
Profit after Tax 75,000 100,000

Other comprehensive income


Revaluation Reserve 10,000 -
Total Comprehensive income 85,000 100,000

264
Additional information:

1. Lukas acquired 40,000 shares in Gaga on the first day of the accounting period for
DN400,000 when the retained earnings were DN50,000 and the fair value of the NCI
was DN30,000.

2. At the date of acquisition, the fair value of Gaga’s non-current assets, which at that
time had a remaining useful life of 10 years, exceeded their book value by DN10,000.

3. The annual impairment review in respect of the goodwill arising indicated that no
impairment losses have arisen.

4. Since the date of acquisition no dividends have been paid and no shares issued.

5. Exchange rates to $1:


DN
Start of the year 4
Average Rate 3
End of the year 2

Required:
Prepare the consolidated statement of financial position and the consolidated
statement of profit or loss and other comprehensive income for Lukas.

265
Answer to Lukas and Gaga:

Consolidated statement of financial position

Group
Property, plant and equipment 272,500
218,000+[100,000+10,000(W2)-1,000(W2)]/2
Financial assets 22,000
22,000+0
Goodwill(W3) 155,000
Current Assets 194,800
90,000+209,600/2
644,300
Ordinary share capital($1/ DN1 share) 175,000
Share Premium 50,000
Revaluation Reserve 40,000
40,000+0
Retained earnings(W5) 151,400
Other components of equity(W5) 99,200
Non-Controlling Interest(W4) 38,900
Total Equity 554,500

Non-current liabilities 59,000


15,000+88,000/2
Current liabilities 30,800
25,000+11,600/2
644,300

266
Consolidated statement of profit or loss and other comprehensive income

Lukas Gaga Adj Total


Sales revenue 500,000 300,000/3 600,000
Operating Costs (400,000) (150,000+1,000(W2))/ (450,333)
3
Operating profit 100,000 149,667
Finance Cost (6,100) (3,300)/3 (7,200)
Interest receivable 1,100 - 1,100
Profit before tax 95,000 143,567
Tax Expense (20,000) (46,700)/3 (35,567)
Profit after Tax 75,000 108,000

Profit attributable to:


Parent: 101,400
NCI: (20%X$33,000)W4 6,600

Other comprehensive income


Revaluation Reserve 10,000 - 10,000
Group exchange gain 124,000
($46,500+$77,500)
Total Comprehensive income 85,000 100,000 242,000

Profit Attributable to:


Parent: 210,600
NCI:(20%X(33,000+46,500+77,500)W4 31,400

267
W1 Group Structure

W2 Net assets list of Gaga

At Realized Not realized At


acquisition consolidation
date(DN) date(DN)
Share capital 50,000 50,000
Share premium 10,000 10,000
Retained earnings 50,000 150,000
Fair value 10,000 10,000
adjustments
Depreciation (1,000)
10,000 / 10 X1
120,000 99,000 219,000
/4 /3 /2
$30,000 $33,000 Bal:$46,500 $109,500
80%X$33,000=$26,400(RE W5) 80%X$46,500=$37,200(W5 OCE)
20%X$33,000=$6,600(NCI W4) 20%X$46,500=$9,300(W4 NCI)

W3 Goodwill
DN
FV of consideration 400,000
FV of NCI 30,000
-FV of sub’s net assets (120,000)
Goodwill at acquisition 310,000 /4=$77,500
-impairment expense - Bal: $77,500:
80%X$77,500=$62,000 OCE W5
20% X$77,500=$15,500 NCI W4
Goodwill at y/e 310,000 /2=$155,000

268
W4: NCI

$
FV of NCI(DN30,000/4) 7,500
+NCI% of post-acquisition profit 6,600
20% X$33,000
+NCI%X group exchange gain on N/A(W2) 9,300
+NCI% X group exchange gain on g/w(W3) 15,500
38,900

W5: consolidated reserve

Other components of Retained earnings $


equity $
Parent 125,000
+ P% of post acquisition profits (W2) 26,400
Retranslation on N/A(W2) 37,200
Retranslation on G/W(W3) 62,000
99,200 151,400

269
Past Exam question Ribby (Exam Suggested Approach)
The following draft statements of financial position relate to Ribby, Hall, and Zian, all
public limited companies, as at 31 May 2008.
Ribby Hall Zian
$m $m Dinars
m
Assets
Non-current assets
Property, plant & equipment 250 120 360
Investment in Hall 98
Investment in Zian 30
Financial assets 10 5 148
Current assets 22 17 120
Total assets 410 142 628
Equity
Ordinary shares 60 40 209
Other components of equity 30 10 -
Retained earnings 120 80 307
Total equity 210 130 516
Non-current liabilities 90 5 40
Current liabilities 110 7 72
Total equity and liabilities 410 142 628

The following information needs to be taken account of in the preparation of the group
financial statements of Ribby.
(a) Ribby acquired 70% of the ordinary shares of Hall on 1 June 2006 when Hall’s other
components of equity were $10 million and retained earnings were $60 million. The fair
value of the net assets of Hall was $120 million at the date of acquisition. Ribby
acquired 60% of the ordinary shares of Zian for 330 million dinars on 1 June 2006 when
Zian’s retained earnings were 220 million dinars. The fair value of the net assets of Zian
on 1 June 2006 was 495 million dinars. The excess of the fair value over the net assets
of Hall and Zian is due to an increase in the value of non-depreciable land. There have
been no issues of ordinary shares since acquisition and goodwill on acquisition is not
impaired for either Hall or Zian.

(b) Zian is located in a foreign country and imports its raw materials at a price which is
normally denominated in dollars. The product is sold locally at selling prices
denominated in dinars, and determined by local competition. All selling and operating
expenses are incurred locally and paid in dinars. Distribution of profits is determined by
the parent company, Ribby. Zian has financed part of its operations through a $4
million loan from Hall which was raised on 1 June 2007. This is included in the financial
assets of Hall and the noncurrent liabilities of Zian. Zian’s management have a
considerable degree of authority and autonomy in carrying out the operations of Zian
and other than the loan from Hall, are not dependent upon group companies for

270
finance.
(c) Ribby has a building which it purchased on 1 June 2007 for 40 million dinars and
which is located overseas.

The building is carried at cost and has been depreciated on the straight-line basis over
its useful life of 20 years. At 31 May 2008, as a result of an impairment review, the
recoverable amount of the building was estimated to be 36 million dinars.

(d) Ribby has a long-term loan of $10 million which is owed to a third party bank. At 31
May 2008, Ribby decided that it would repay the loan early on 1 July 2008 and formally
agreed this repayment with the bank prior to the year end. The agreement sets out
that there will be an early repayment penalty of $1 million.

(e) The directors of Ribby announced on 1 June 2007 that a bonus of $6 million would
be paid to the employees of Ribby if they achieved a certain target production level by
31 May 2008. The bonus is to be paid partly in cash and partly in share options. Half of
the bonus will be paid in cash on 30 November 2008 whether or not the employees are
still working for Ribby. The other half will be given in share options on the same date,
provided that the employee is still in service on 30 November 2008. The exercise price
and number of options will be fixed by management on 30 November 2008. The target
production was met and management expect 10% of employees to leave between 31
May 2008 and 30 November 2008. No entry has been made in the financial statements
of Ribby.

(f) Ribby operates a defined benefit pension plan that provides a pension of 1·2% of
the final salary for each year of service, subject to a minimum of four years service. On
1 June 2007, Ribby improved the pension entitlement so that employees receive 1·4%
of their final salary for each year of service. This improvement applied to all prior years
service of the employees. As a result, the present value of the defined benefit
obligation on 1 June 2007 increased by $3.5 million as follows:

$m
Employees with more than four years’ service 3.0
Employees with less than four years’ service (average service of two 0.5
years)
3.5

Ribby had not accounted for the improvement in the pension plan.

(g) Ribby is considering selling its subsidiary, Hall. Just prior to the year end, Hall sold
inventory to Ribby at a price of $6 million. The carrying value of the inventory in the
financial records of Hall was $2 million. The cash was received before the year end, and
as a result the bank overdraft of Hall was virtually eliminated at 31 May 2008. After the
year end the transaction was reversed, and it was agreed that this type of transaction

271
would be carried out again when the interim financial statements were produced for
Hall, if the company had not been sold by that date.

(h) The following exchange rates are relevant to the preparation of the group financial
statements:

Dinars to $
1 June 2006 11
1 June 2007 10
31 May 2008 12
Average for year to 31 May 2008 10.5

(i) It is the group’s policy to value the non-controlling interest at acquisition at fair
value. The fair value of thenon-controlling interest in Hall on 1 June 2006 was
$42million. The fair value of the non-controlling interest in Zian on 1 June 2006 was
220 million dinars.

Required
(a) Discuss and apply the principles set out in IAS 21 The effects of changes in foreign
exchange rates in order to determine the functional currency of Zian. (8 marks)

(b) Prepare a consolidated statement of financial position of the Ribby Group at 31 May
2008 in accordance with International Financial Reporting Standards. (35 marks)

(c) Discuss how the manipulation of financial statements by company accountants is


inconsistent with their responsibilities as members of the accounting profession,
setting out the distinguishing features of a profession and the privileges that society
gives to a profession. (Your answer should include reference to the above scenario.) (7
marks)

Note: requirement (c) includes 2 marks for the quality of the discussion.
(Total = 50 marks)

272
IAS 7 statement of cash flow

In the p2 exam you are required to prepare for a group statement of cash flow.

This builds on the knowledge that you have learnt from your ACCA F3&7 about single
company statement of cash flow.

Before we look at the group statement of cash flow we would like to revise the single
company statement of cash flow in the following:

273
1, IAS7 cash flow statement introduction

Sample cash flow pro forma:

Julia Plc statement of cash flows for the year ended 31 DEC 2014:
$ $
Cash flow from operating activities

Profit before tax 44,400


non-cash items Adjustments:
Depreciation/amortization charge 17,600
Loss/(profit) on disposal of non-current assets (6,320)
Finance cost 2,000
Investment income (80)
Working capital adjustments:
(Increase)/decrease in inventory 2,000
(Increase)/decrease in receivables (800)
Increase/(decrease) in payables 200
Cash generated from operations 59,000

Interest paid (1,900)


Taxation paid (17,850)
Net cash from operating activities 39,250

Cash flows from investing activities


Purchase of non-current assets (41,600)
Proceeds from the sale of non-current assets 27,320
Interest received 40
Dividends received 40
Net cash used in investing activities (14,200)

Cash flows from financing activities


Proceeds from the issue of share capital 10,000
Proceeds from long-term borrowings 3,000
Repayments of long term borrowings (25,000)
Dividends paid (20,000)
Net cash from financing activities (32,000)

Movement in cash and cash equivalents (6,950)


Cash and cash equivalents b/f 6,450
Cash and cash equivalents c/f (500)

274
Q: JYC Ltd

JYC Ltd.’s statement of financial position as at 31 DEC 2014:


2014 2013
$ $ $ $
Non-current assets
Property plant and equipment cost 220,000 180,000
Property. Plant & equipment (92,000) (78,000)
accumulated depreciation
128,000 102,000

Current assets
Inventory 17,000 12,000
Trade receivables 10,000 2,000
Government bonds 10,000 10,000
Bank 16,000 3,000
53,000 27,000
Total assets 181,000 129,000

Equity and liabilities


Share capital($1) 65,000 45,000
Share premium 12,000 10,000
Retained earnings 68,000 24,000
145,000 79,000
Non-current liabilities
10% loan note 20,000 30,000

Current liabilities
Trade payables 13,000 19,000
Taxation 3,000 1,000
16,000 20,000
Total equity and liabilities 181,000 129,000

275
JYC’s statement of profit or loss and other comprehensive income for the year
ended 31 DEC 2014:
$
Profit before interest and tax 52,000
Finance cost (2,000)
Taxation expense (6,000)
Profit for the year 44,000

Requirement:
Prepare the statement of cash flow for JYC Ltd for the year ended 31 DEC2014.

276
Note:

1, cash flow from operating activities:

Cash from day to day operating (trading) of the business.

The cash generated from operations can be calculated using either


*Direct method: cash in from customers-cash out to suppliers
*Indirect method: start from profit before tax and adjust Non cash and Working
capital items (turn accounting profit into cash profit)

Example for direct method:

Cash sales $55,000


Cash received from customers $44,000
Cash purchases $43,000
Cash paid to suppliers $12,000
Cash expenses $11,000
Cash wages and salaries $20,000

Q: What is the cash generated from operations?

Answer:

Total cash received=55,000+44,000=99,000

Total cash paid=43,000+12,000+11,000+20,000=86,000

Cash generated= total cash received-total cash paid=13,000

2, cash flows from investing activities:


Cash from buying and selling internal (PP&E) and external(shares in other company)
investments.

3, cash flows from financing activities:


Cash in and out from business finance either from equity finance and debt finance.

277
Group statement of cash flow:
$ $
Cash flow from operating activities

Group Profit before tax


non-cash items Adjustments:
Depreciation/amortization charge
Loss/(profit) on disposal of non-current assets
Finance cost
Investment income
*Impairment(goodwill)
*Loss/(profit) on sale of subsidiary
*Share of Associate profit

Working capital adjustments:


(Increase)/decrease in inventory (include acq/disposal of sub)
(Increase)/decrease in receivables(include acq/disposal of sub)
Increase/(decrease) in payables(include acq/disposal of sub)
Cash generated from operations

Interest paid
Taxation paid
Net cash from operating activities

Cash flows from investing activities


Purchase of non-current assets
Proceeds from the sale of non-current assets
Interest received
Dividends received
*dividend received/acquisition from/of associate
*acquisition/disposal of Sub
Net cash used in investing activities

Cash flows from financing activities


Proceeds from the issue of share capital
Proceeds from long-term borrowings
Repayments of long term borrowings
Dividends paid to Parent Co
*Dividends paid to NCI
Net cash from financing activities

Movement in cash and cash equivalents


Cash and cash equivalents b/f
Cash and cash equivalents c/f

278
Q Ding Ding (Dividend Received from Associate)

The year end is 30 Apr 2013. On this date the investment in associate in Ding Ding’s FS was
$750m (2012 $600m). During the year income from the associate was $200m.

Required:
Calculate the dividend received from the associate for the year to 30 Apr 2013.

Q Da Da (Dividend Paid to Non Controling Interest)

The opening Non-Controlling Interest (NCI) for DaDa was $24m. Closing NCI was $150m.

During the year Da Da acquired a 75% share in a subsidiary cha-cha.

The value of the net assets at acquisition of cha-cha were $480m.

The Non-Controlling Interest charge in the income statement was $22m.

Required:
Calculate the dividend paid to Non-Controlling Interest for the year.

279
Past Exam question: (DEC2010 Q1 updated)

The following draft group financial statements relate to Jocatt, a public limited company.
JOCATT GROUP
STATEMENT OF FINANCIAL POSITION AS AT 30 NOVEMBER
2013 2012
$m $m

280
JOCATT GROUP STATEMENT OF PROFIT OR LOSS AND OTHER COMPREHENSIVE INCOME
FOR THE YEAR ENDED 30 NOVEMBER 2013

281
JOCATT GROUP STATEMENT OF CHANGES IN EQUITY FOR THE YEAR ENDED 30 NOVEMBER 2013

$m Share Retained Investment Revaluation Total Non-controlling Total


capital earnings in equity surplus interest equity
instrument (PPE)
Balance at 1 275 324 4 16 619 36 655
DEC 2012
Share capital 15 15 15
issued
Dividends (5) (5) (13) (18)
Right issue 2 2
acquisitions 20 20
Total 32 2 (7) 27 10 37
comprehensive
income for the
year
Balance at 290 351 6 9 656 55 711
30NOV 2013

282
The following information relates to the financial statements of Jocatt.
(i) On 1 December 2011, Jocatt acquired 8% of the ordinary shares of Tigret. Jocatt
had treated this as an investment in equity instruments in the financial statements to
30 November 2012 with changes in fair value taken to profit or loss for the year. There
were no changes in fair value in the year to 30 November 2012. On 1 January 2013,
Jocatt acquired a further 52% of the ordinary shares of Tigret and gained control of the
company. The consideration for the acquisitions was as follows.

Holding Consideration($m)
1 DEC 2011 8% 4
1 JAN 2013 52% 30
60% 34

At 1 January 2013, the fair value of the 8% holding in Tigret held by Jocatt at the time
of the business
combination was $5 million and the fair value of the non-controlling interest in Tigret
was $20 million. The purchase consideration at 1 January 2013 comprised cash of $15
million and shares of $15 million.

The fair value of the identifiable net assets of Tigret, excluding deferred tax assets and
liabilities, at the date of acquisition comprised the following.

$m
PP&E 15
Intangible assets 18
Trade receivables 5
Cash 7

The tax base of the identifiable net assets of Tigret was $40 million at 1 January 2013.
The tax rate of Tigret is 30%.

(ii) On 30 November 2013, Tigret made a rights issue on a 1 for 4 basis. The issue was
fully subscribed and raised $5 million in cash.

(iii) Jocatt purchased a research project from a third party including certain patents on
1 December 2012 for $8 million and recognized it as an intangible asset. During the
year, Jocatt incurred further costs, which included $2 million on completing the
research phase, $4 million in developing the product for sale and $1 million for the
initial marketing costs. There were no other additions to intangible assets in the period
other than those on the acquisition of Tigret.

283
(iv) Jocatt operates a defined benefit scheme. The current service costs for the year
ended 30 November 2013 are $10 million. Jocatt enhanced the benefits on 1 December
2012.The total cost of the enhancement is $2 million. The interest on plan assets was
$8 million for the year and Jocatt recognizes measurement gains and losses in
accordance with IAS 19 as revised in 2011.

(v) Jocatt owns an investment property. During the year, part of the heating system of
the property, which had a carrying value of $0·5 million, was replaced by a new system,
which cost $1 million. Jocatt uses the fair value model for measuring investment
property.

(vi) Jocatt had exchanged surplus land with a carrying value of $10 million for cash of
$15 million and plant valued at $4 million. The transaction has commercial substance.
Depreciation for the period for property, plant and equipment was $27 million.

(vii) Goodwill relating to all subsidiaries had been impairment tested in the year to 30
November 2013 and any impairment accounted for. The goodwill impairment related
to those subsidiaries which were 100% owned.

(viii) Deferred tax of $1 million arose in the year on the gains on investments in equity
in the year where the irrevocable election was made to take changes in fair value
through other comprehensive income

(ix) The associate did not pay any dividends in the year.

Required
(a) Prepare a consolidated statement of cash flows for the Jocatt Group using the
indirect method under IAS 7 Statements of cash flows.

Note:Ignore deferred taxation other than where it is mentioned in the question. (35
marks)

284
Group statement of profit or loss and other comprehensive income

285
Group statement of profit or loss and other comprehensive income
pro forma
Parent Subsidiary Adjustments Group
(post acq only)

Revenue X X (X) X
Less:

Cost of sales (X) (X) X


PUP (X) (X) - (X)
(who sells?) P-S S-P
Gross profit X X - X
Other income X X X
Less:

Distribution costs (X) (X) - (X)


Administration expenses (X) (X) - (X)
Finance Cost (X) (X) - (X)
Share of profit of associate X X
Profit before tax X X - X

Less: tax (X) (X) - (X)


Profit for the year X X* - X
Other comprehensive income

Exchange differences on

translating foreign operations

Cash flow hedges

Gains on property revaluation

Actuarial gains/(losses) on

defined benefit pension plans

Other comprehensive income X

for the year, net of tax

Profit Attributable to:

NCI (NCI% X profit for the X


year)
Equity holders of the parent Remaining figure X
X

Total comprehensive income

attributable to:

NCI (NCI% X total X

comprehensive income for

the year)

Equity holders of the parent Remaining figure X

286
June2010 Q1 Ashanti (Group P/L)
The following financial statements relate to Ashanti, a public limited company.
Consolidated statement of profit or loss and other comprehensive income for the
year ended 30 April 2014

The following information is relevant to the preparation of the group statement of


profit or loss and other comprehensive income:

(i) On 1 May 2012, Ashanti acquired 70% of the equity interests of Bochem, a
public limited company. The purchase consideration comprised cash of $150
million and the fair value of the identifiable net assets was $160 million at that date.
The fair value of the non-controlling interest in Bochem was $54 million on 1 May
2012. Ashanti wishes to use the ‘full goodwill’ method for all acquisitions. The share
capital and retained earnings of Bochem were $55 million and $85 million
respectively and other components of equity were $10 million at the date of
acquisition. The excess of the fair value of the identifiable net assets at acquisition
is due to an increase in the value of plant, which is depreciated on the straight-line
method and has a five year remaining life at the date of acquisition. Ashanti
disposed of a 10% equity interest to the non- controlling interests (NCI) of Bochem
on 30 April 2014 for a cash consideration of $34 million. The carrying value of the
net assets of Bochem at 30 April 2014 was $210 million before any adjustments on
consolidation. Goodwill has been impairment tested annually and as at 30 April
2013 had reduced in value by 15% and at 30 April 2014 had lost a further 5% of its
original value before the sale of the equity interest to the NCI. The goodwill
impairment should be allocated between group and NCI on the basis of equity
shareholding.
(ii) Bochem acquired 80% of the equity interests of Ceram, a public limited

287
company, on 1 May 2012. The purchase consideration was cash of $136 million.
Ceram’s identifiable net assets were fair valued at $115 million and the NCI of
Ceram attributable to Ashanti had a fair value of $26 million at that date. On 1
November 2013, Bochem disposed of 50% of the equity of Ceram for a
consideration of $90 million.

Ceram’s identifiable net assets were $160 million and the consolidated value of the
NCI of Ceram attributable to Bochem was $35 million at the date of disposal. The
remaining equity interest of Ceram held by Bochem was fair valued at $45 million.
After the disposal, Bochem can still exert significant influence. Goodwill had been
impairment tested and no impairment had occurred. Ceram’s profits are deemed to
accrue evenly over the year.

(iii) Ashanti has sold inventory to both Bochem and Ceram in October 2013. The
sale price of the inventory was $10 million and $5 million respectively. Ashanti sells
goods at a gross profit margin of 20% to group companies and third parties. At the
year-end, half of the inventory sold to Bochem remained unsold but the entire
inventory sold to Ceram had been sold to third parties.

(iv) On 1 May 2011, Ashanti purchased a $20 million five-year bond with semi
annual interest of 5% payable on 31 October and 30 April. The purchase price of
the bond was $21·62 million. The effective annual interest rate is 8% or 4% on a
semiannual basis. The bond is held at amortized cost. At 1 May 2013 the amortized
cost of the bond was $21.046 million. The issuer of the bond did pay the interest
due on 31 October 2013 and 30 April 2014, but was in financial trouble at 30 April
2014. Ashanti feels that as at 30 April 2014, the bond is impaired and that the best
estimates of total future cash receipts are $2·34 million on 30 April 2015 and $8
million on 30 April 2016. The current interest rate for discounting cash flows as at
30 April 2014 is 10%. No accounting entries have been made in the financial
statements for the above bond since 30 April 2013. (You should assume the annual
compound rate is 8% for discounting the cash flows.)

(v) Ashanti sold $5 million of goods to a customer who recently made an


announcement that it is restructuring its debts with its suppliers including Ashanti.
It is probable that Ashanti will not recover the amounts outstanding. The goods
were sold after the announcement was made although the order was placed prior
to the announcement. Ashanti wishes to make an additional allowance of $8 million
against the total receivable balance at the year end, of which $5 million relates to
this sale.

(vi) Ashanti owned a piece of property, plant and equipment (PPE) which cost $12

288
million and was purchased on 1 May 2012. It is being depreciated over 10 years on
the straight-line basis with zero residual value. On 30 April 2013, it was revalued to
$13 million and on 30 April 2014, the PPE was revalued to $8 million. The whole of
the revaluation loss had been posted to other comprehensive income and
depreciation has been charged for the year. It is Ashanti’s company policy to make
all necessary transfers for excess depreciation following revaluation.

(vii) The salaried employees of Ashanti are entitled to 25 days paid leave each year.
The entitlement accrues evenly over the year and unused leave may be carried
forward for one year. The holiday year is the same as the financial year. At 30 April
2014, Ashanti has 900 salaried employees and the average unused holiday
entitlement is three days per employee. 5% of employees leave without taking
their entitlement and there is no cash payment when an employee leaves in
respect of holiday entitlement. There are 255 working days in the year and the
total annual salary cost is $19 million. No adjustment has been made in the
financial statements for the above and there was no opening accrual required for
holiday entitlement.

(viii) As permitted by IFRS 9 Financial instruments all group companies have made
an irrecoverable election to recognize changes in the fair value of investments in
equity instruments in other comprehensive income (items that will not be
reclassified to profit or loss).

(ix) Ignore any taxation effects of the above adjustments and the disclosure
requirements of IFRS 5 Non-current assets held for sale and discontinued
operations.

Required

(a) Prepare a consolidated statement of profit or loss and other comprehensive


income for the year ended 30 April 2014 for the Ashanti Group. (35 marks)

289

Das könnte Ihnen auch gefallen